Vous êtes sur la page 1sur 141

8. CM. What concentrations of anesthetic solution are used for local tumescent anesthesia?

a) 0,1-0,2%
b) 2-5%
c) 1-5%
d) 5-10%
e) 0,2-0,5%

9. CM. Technique of local tumescent anesthesia by Vishnevsky’s method includes:


a) Infiltration of tissues, layer by layer, with low concentration solution (0,25%) of anesthetic solution
b) Infiltration of tissues, layer by layer, with 1-2% solution of anesthetic
c) Perineural or endoneural administration of anesthetic solution
d) Hydraulic preparation of tissue due to tumescent infiltration
e) Alternated use of syringe with anesthetic and scalpel

10. CS. What substances are used for loco-regional (conductive) anesthesia?
a) 0,25-0,5% solution of lidocaine
b) 1-2% solution of lidocaine
c) 10% solution of lidocaine
d) 1-5% solution of cocaine
e) 0,5% solution of dicaine

11. CM. What local anesthetics refer to the etheric type group?
a) Procaine
b) Cocaine
c) Dicaine
d) Lidocaine
e) Trimecaine

12. CM. Select the local anesthetics that can be administrated without previous skin test for allergy.
a) Procaine
b) Cocaine
c) Lidocaine
d) Trimecaine
e) Ketamine

13. CM. Specify the contraindications for local anesthesia.


a) Known allergy to local anesthetic
b) Psychiatric disorders
c) Refuse of patient from this type of anesthesia
d) Severe nervous excitation
e) Severe concomitant somatic diseases

14. CM. What local anesthetics refer to the amide type group?
a) Procaine
b) Trimecaine
c) Cocaine
d) Lidocaine
e) Dicaine

15. CS. Epidural anesthesia represent a variant of:


a) Tumescent anesthesia
b) Regional anesthesia
c) Superficial anesthesia
d) Vishnevsky’s anesthesia
e) Intraoseous anesthesia

16. CM. What types of local anesthesia refers to the regional anesthesia?
a) Spinal anesthesia
b) Oberst-Lukashevich’s anesthesia
c) Epidural anesthesia
d) Vishnevsky’s anesthesia
e) Anesthesia of mucous membran by local application of anesthetics

17. CM. Select the correct statements regarding regional anesthesia.


a) Anesthetic substance is administrated into the nerve or perineural space
b) Anesthetic substance comes in the direct contact with nerve receptors
c) Technique requires exact information regarding the course of nervous trunk
d) Localization of nervous trunks close to the major vessels increases the risk of bleeding
e) Anesthesia is achived in a few minutes

18. CM. Select the typical complications of spinal anesthesia.


a) Multiple vomiting
b) Involuntary urination
c) Involuntary defecation
d) Hypotension
e) Prolonged hypertension

19. CS. During spinal anesthesia the puncture usually performed:


a) At the level of cervical spine
b) At the level of thoracic spine
c) At the level of lumbar spine
d) At the level of sacral spine
e) At all levels mentioned above

20. CS. For prevention of spinal cord injury puncture during spinal anesthesia should be performed at the level
of:
a) XII thoracic intervertebral space
b) I lombar intervertebral space
c) II lombar intervertebral space
d) III lombar intervertebral space
e) IV lombar intervertebral space

21. CS. Select the correct order of sensation loss during the spinal anesthesia.
a) Pain, thermal, tactile
b) Tactile, thermal, pain
c) Thermal, tactile, pain
d) Pain, tactile, thermal
e) All are loosed at the same time

22. CS. During the spinal anesthesia the main volume of anesthetic substance is introduced in:
a) Epidural space
b) Subdural space
c) Spinal cord
d) Intervertebral space
e) Subcutaneous tissue

23. CM. Specify the causes of hypotension during spinal anesthesia.


a) Cerebral edema
b) Cardio-toxicity of anesthetic solution
c) Relaxation of diaphragm
d) Blocking of sympathetic part of autonomic nervous system
e) Peripheral vascular dilation

24. CM. Select surgical interventions that may be performed under spinal anesthesia.
a) Lower limb amputation
b) Surgery for phlegmon of hand
c) Gastric resection
d) Surgery for varicose veins disease
e) Surgery for hemorrhoid disease

25. CM. Specify the contraindications for spinal anesthesia:


a) Traumatic shock
b) Arterial hypertension
c) Skin infection in the lumbar region
d) Spinal column deformation
e) Hypovolemic shock

26. CS. Specify the most frequent complication of spinal anesthesia.


a) Meningitis
b) Hypotension
c) Cardiac arrest
d) Respiratory arrest
e) Paralysis of lower limbs

27. CM. Specify complications characteristic for spinal anesthesia.


a) Headache
b) Meningial irritation
c) Hypotension
d) Urinary retention
e) Mental disorders

28. CM. Specify the correct statements regarding epidural anesthesia.


a) This type of anesthesia refers to loco-regional anesthesia
b) Rate of complications is lower than in spinal anesthesia
c) Anesthetic substance is mixed with cerebrospinal fluid
d) This type of anesthesia may be used during lower limbs surgery
e) The contraindications for this type of anesthesia are similar with spinal anesthesia

29. CM. Epidural anesthesia can be used for:


a) Prolonged postoperative analgesia
b) Surgical hemostasis of massive obstetrical bleeding
c) Lower limb surgery
d) Upper limb surgery
e) Surgical intervention at the patients with arterial hypotension

30. CS. What is antisepsis?


a) It is a complex of measures used for sterilization of air in the operating room
b) It is a complex of measures used for destruction of microorganisms in the wound
c) It is a complex of measures used for preventing entering of microorganisms into the wound
d) It is a complex of measures used for sterilization of surgical instruments
e) It is a complex of measures used for destruction of microorganisms by physical methods

31. CM. Specify historical periods in the development of antisepsis.


a) The period of Lister’s antisepsis
b) The period of classical antisepsis
c) The period of empirical antisepsis
d) The period of contemporary antisepsis
e) The period of the antisepsis of the XIX-th century before Lister’s antisepsis

32. CS. Who is the founder of modern antisepsis?


a) Ernest Bergmann
b) Kurt Schimmelbusch
c) Hippocrate
d) Louis Pasteur
e) Joseph Lister

33. CS. Which antiseptic was used by Joseph Lister to destroy microorganisms in the wound?
a) Chloramine
b) 96% ethyl alcohol
c) Carbolic acid
d) Corrosive sublimate
e) Furacilin

34. CM. Why does Joseph Lister remained in the history of surgery as the founder of antisepsis?
a) Based on the discovery of Louis Pasteur he made a conclusion, that the cause of purulent complications in
surgery was bacteria
b) For the first time he elaborated a system of measures to fight with infection in surgery
c) He discovered microorganisms and established their relationship with infectious surgical diseases
d) He synthesized penicillin and first applied it in surgery
e) For the first time he developed a method to sterilize dressing material and operating field in autoclave

35. CM. Specify current types of antisepsis.


a) Mechanical antisepsis
b) Endogenous antisepsis
c) Biological antisepsis
d) Preventive antisepsis
e) Antimicrobial antisepsis

36. CM. Specify the existing types of antisepsis.


a) Exogenous antisepsis
b) Aerogenic antisepsis
c) Physiological antisepsis
d) Physical antisepsis
e) Chemical antisepsis

37. CS. What type of antisepsis is most frequently used in surgery?


a) Physical antisepsis
b) Biological antisepsis
c) Mechanical antisepsis
d) Combined antisepsis
e) Chemical antisepsis

38. CS. Purulent wound is drained by gauze dressing imbibed with Betadine. What type of antisepsis was used?
a) Chemical antisepsis
b) Combined antisepsis
c) Mechanical antisepsis
d) Physical antisepsis
e) Biological antisepsis

39. CS. Wound irrigation through the drain with Dioxidine solution is referred:
a) To chemical antisepsis
b) To physical antisepsis
c) To mechanical antisepsis
d) To biological antisepsis
e) To combined antisepsis

40. CS. How is wound toilet performed?


a) The wound is dissected, the edges and bottom of wound are excised within the limits of healthy tissues
b) The wound is treated by water jet under a high pressure, washing off foreign bodies, pus and microorganisms
c) Purulent exudate, infected clots, necrotized tissue are removed using a forceps (pincett) with a gauze globule
d) A permanent lavage of wound is produced: an antiseptic solution is introduced through one tube, and it flows
out through another one
e) The wound is closed with a gauze dressing, imbibed with solution of hydrogen peroxide

41. CM. Which of these surgical measures are related to mechanical antisepsis?
a) Puncture and evacuation of pus in sinus abscess
b) Tamponade of wound with a gauze dressing
c) Installation of the active drainage
d) Primary surgical debridement (processing of wound)
e) Drainage of postinjectional abscess

42. CS. Which of the following refers to mechanical antisepsis?


a) Irrigation of wound with a Dioxidine solution
b) Tamponade of wound with gauze dressing
c) Excision of necrotic tissue
d) Administration of interferon
e) Applying of a sterile bandage to the wound

43. CM. What characterizes correctly the primary surgical processing of wound?
a) Is performed, when purulent procesess is already developed in the wound
b) Purulent exudate, infected clots, necrotized tissue are removed using a forceps (pincett) with a gauze globule
c) Consists of dissection of wound, and excision of wound edges and bottom within the limits of healthy tissues
d) Is performed not later than 12 hours after injuring
e) Consists of hemostasis and closure of wound with aseptic bandage to prevent its infection

44. CS. Edges and bottom of the infected wound were excised. What type of antisepsis was used?
a) Chemical
b) Physical
c) Biological
d) Mechanical
e) Combined

45. CM. Which of the following surgical measures are related to mechanical antisepsis?
a) Secondary surgical processing of wound
b) Packing of wound with a sterile gauze
c) Drainage of purulent collections
d) Primary surgical processing of wound
e) Lavage of wound

46. CM. To the methods of physical antisepsis in treatment of wounds refers:


a) Use of hypertonic saline solution
b) Drainage of wound
c) Primary surgical processing of wound
d) Use of hydrogen peroxide solution
e) Use of gauze dressing

47. CM. To the methods of physical antisepsis refers:


a) Irradiation of infected wound with high-energy (surgical) laser
b) Ultrasound cavitation of purulent wound
c) Chemotherapy of purulent focus
d) Radiotherapy of purulent focus
e) Exploration of purulent wound with probe

48. CS. What is common in the principles of action of hygroscopic dressing material and hypertonic saline
solution?
a) Drying the wound, which creates unfavorable conditions for bacterial multiplication
b) Bactericidal effect on microorganisms in the wound
c) They remove mechanically foreign bodies, pus and bacteria from a wound
d) They improve wound flow-out
e) They improve oxygenation of wound tissue

49. CM. Which of the following are the methods of wound drainage?
a) Flow-irrigative drainage
b) Passive drainage
c) Fractional drainage
d) Active drainage
e) Permanent drainage

50. CS. An active drainage of a purulent focus involves:


a) Creation of negative pressure at the outer end of drainage tube
b) Creation of positive pressure at the outer end of drainage tube
c) Introduction of sorbents into the wound, which actively absorb toxins
d) Drainage is installed in the lower part of the wound
e) Continuous flow irrigation of pathological focus

51. CM. A flow-irrigative drainage of purulent focus involves:


a) Drainage is installed in the lower part of the wound
b) Instalation of two drainage tubes into the wound
c) Creation of negative pressure at the outer end of drainage tube
d) Continuous flow irrigation of pathological focus with antiseptics
e) Periodical aspiration of contents from pathological focus through the drain

52. CM. Specify additional methods of physical antisepsis.


a) Ultrasound cavitation
b) Use of bacteriophage
c) Wound processing with water jet
d) Use of carbon-containing substances (sorbents)
e) High-energy surgical laser

53. CM. What is characteristic of the method of wound treatment into isolator with abacterial environment?
a) Formation of scab, under which cleaning and healing of wound occur
b) It leads to vaporization of tissue structures
c) Jet of air under high pressure removes foreign bodies, pus and microorganisms
d) Drying of wound occurs
e) Installation for treatment consists of a compressor, bacterial filter and isolator

54. CS. The effect of cavitation, having a devastating effect on microorganisms in the wound, is caused by the
action of:
a) High-energy surgical laser
b) Carbon-containing substances (sorbents)
c) Ultrasound
d) Ultra-violet rays
e) Isolator with abacterial environment

55. CM. What is effect of high-energy surgical laser, as an additional method of wound’s processing?
a) Mechanically removes microorganisms from the walls and bottom of infected wound
b) Causes evaporation of tissues
c) Causes the effect of cavitation
d) Causes drying of wound
e) After processing the wound becomes sterile, covered with a burn scab

56. CM. Irradiation with ultraviolet rays is effective in:


a) Abscess and phlegmon
b) Deep purulent wounds
c) Superficial wounds
d) Erythematous form of erysipelas
e) Trophic ulcers of different ethiology

57. CM. To chemical antisepsis refers administration of:


a) Toxoids (anatoxins)
b) Antibiotics
c) Dyes
d) Analgesics
e) Oxidants
58. CM. What is a chemical antiseptic Cidex?
a) It is used for preparing of surgeon’s hands and an operating field
b) It belongs to a group of heavy metal salts
c) It belongs to a group of aldehydes
d) It is used for lavage of the abdominal cavity in peritonitis
e) It is used for sterilization of flexible endoscopes and instruments from plastic mass

59. CM. Specify chemical antiseptics from the group of haloids.


a) Iodopirone
b) Lysoform
c) Solution of Chloramine
d) Alcoholic solution of Iodine
e) Betadine

60. CM. Specify the chemical antiseptics, used for local irrigation of wounds.
a) Metronidazole
b) Cidex
c) Chlorhexidine
d) Manganese crystals solution
e) Betadine

61. CS. In what kind of infection the local use of boric acid is particularly effective?
a) In a specific tuberculous infection
b) In staphylococcal infection
c) In gram-negative infections caused by Escherichia coli
d) In Pseudomonas aeruginosa infection
e) In anaerobic non-clostridial infections

62. CS. Which antiseptic agent has both bactericidal effect and mechanical action, cleaning the wound of pus
and foreign bodies in local application?
a) Solution of Furacilin
b) Manganese crystals solution
c) Solution of Dioxidine
d) Solution of hydrogen peroxide
e) Solution of ethyl alcohol

63. CM. Specify the correct statements that characterize metronidazole.


a) It is a broad-spectrum antibiotic
b) It is effective against protozoa and anaerobic bacteria
c) It is applied locally into purulent wounds and cavities
d) It is administered intravenously and orally
e) It is a chemical antiseptic

64. CM. Which of the antiseptics can be used both locally and systemically (orally or parenterally)?
a) Dioxidine
b) Metronidazole
c) Alcoholic solution of iodine
d) Derivates of nitrofurane
e) Chlorhexidine

65. CM. Unlike antiseptics, disinfectants are used for:


a) Decontamination of surgical instruments
b) Decontamination of surgeon’s hands before operation
c) Decontamination of operated field
d) Decontamination of hospital placements
e) Irrigation of purulent cavities

66. CM. Specify the antibiotics which are used in surgery.


a) Ceftriaxone
b) Biseptol
c) Gentamicin
d) Metronidazole
e) Ampioks

67. CM. Specify the principles of rational antibiotic therapy.


a) An enteral way of antibiotics administration is preferable
b) It is necessary to determine the sensitivity of microorganisms to antibiotics
c) It is preferred to administer antibiotics with different spectrum of action
d) Antibiotics should be administered in minimal therapeutic dosage
e) Antibiotics should be used based on strict indications

68. CM. How to administrate antibiotics correctly at admission of patient with severe surgical infection?
a) Antibiotics should not be prescribed to patients with underlying liver and kidney diseases due to risk of toxic
effects on internal organs
b) Bacteriological analysis from the wound or from the site of infection should immediately be obtain in order to
determine the sensitivity of microflora to antibiotics
c) Antibiotics should be prescribed only after receiving the results of antibioticogramm, taking into consideration
sensitivity and resistance of microorganisms
d) Standard doses of broad spectrum antibiotics should be administered immediately on empirical basis
e) While the sensitivity of microflora to antibiotics is not known, it is necessary to administrate medications in
minimal therapeutic doses

69. CS. If prolonged antibiotic therapy is required, in order to avoid resistance of microorganisms to antibiotics,
they should be replaced every:
a) 1-2 days
b) 5-7 days
c) 10-12 days
d) 14-21 days
e) Month

70. CM. What complications may occur during antibiotic therapy?


a) Hormonal disorders
b) Dysbacteriosis
c) Allergic reactions
d) Formation of resistant strains of microorganisms
e) Toxic effects on inner organs

71. CM. Which of the following methods relate to biological antisepsis?


a) Local application of bacteriophages
b) Treating of wounds with solution of hydrogen peroxide
c) Wound drainage
d) Enteral administration of sulfonamides
e) Intramuscular administration of antibiotics

72. CS. What are the effects of curative serums in patients with surgical infections?
a) Introduction of antibodies to the infectious agent for passive immunization of patients
b) Exibit bactericidal and bacteriostatic effect on microorganisms
c) They contain viruses, which can reproduce in bacterial cells and destroy them
d) Introduction of minimal quantity of microorganisms for stimulation of host own antibodies release
e) They stimulate a nonspecific immunity

73. CS. What is the action of bacteriophages in patients with surgical infections?
a) Introduction of antibodies to the infectious agent for passive immunization of patients
b) They stimulate the mechanisms of nonspecific resistance of human body to infection
c) They contain viruses, which can reproduce in bacterial cells and destroy them
d) Introduction of minimal quantity of microorganisms for stimulation of host own antibodies release
e) They stimulate a nonspecific immunity
74. CS. What are the effects of vaccines and anatoxins in patients with surgical infection?
a) Administration of antibodies to infectious agents for passive immunization of patient
b) Exibit direct bactericidal and bacteriostatic effect on microorganisms
c) Contain a virus which can reproduce in bacterial cells and cause their death
d) The introduction to patient of a minimum dose of microbial antigens in order to stimulate his own antibodies
release
e) Stimulate non-specific immunity

75. CM. What medications are related to biological antiseptics of direct action?
a) Tetanus toxoid (anatoxin)
b) Antigangrenous serum
c) T-activin
d) Antistaphylococcal gamma-globulin
e) Combined bacteriophage

76. CM. What medications are related to biological antiseptics of indirect action?
a) Tetanus toxoid (anatoxin)
b) Antigangrenous serum
c) T-activin
d) Antistaphylococcal gamma-globulin
e) Combined bacteriophage

77. CM. The actions of proteolytic enzymes in the infected wound are the following:
a) Anti-inflammatory
b) Destruction of microorganisms
c) Local analgesic effect
d) Wound cleaning from necrotic tissue and fibrin
e) Anti-edematous

78. CM. The most common types of nosocomial infection among all hospitalized patients are:
a) Urinary infection
b) Acute intestinal infection
c) Respiratory infection
d) Central nervous system infection
e) Surgical site infection

79. CS. The most common type of nosocomial infection among operated patients is:
a) Urinary infection
b) Respiratory infection
c) Central nervous system infection
d) Surgical site infection
e) Hepatitis С

80. CM. Specify the criteria used for definition of term “surgical site infection”.
a) Grade of emergency of surgical intervention
b) Time interval from the moment of surgical intervention
c) Anatomic region, where the surgery was performed
d) Results of bacteriological cultures
e) Presence of implant in surgical site

81. CS. During what time frame development of septic process in the operating wound after appendectomy is
defined as nosocomial surgical site infection?
a) One week
b) Two weeks
c) One month
d) Six months
e) One year
82. CS. During what time frame development of septic process in the operating wound after hernia repair with
synthetic mesh is defined as nosocomial surgical site infection?
a) One week
b) Two weeks
c) One month
d) Six months
e) One year

83. CM. According to anatomical principle the surgical site infections are classified as:
a) Asymptomatic infection
b) Superficial incisional infection
c) Deep incisional infection
d) Generalized infection
e) Organ and space infection

84. CM. According to the anatomical classification, deep surgical site infection involves:
a) Skin
b) Subcutaneous tissue
c) Fascia or aponeurosis
d) Muscles
e) Internal organs

85. CS. In general surgery surgical site infection is most frequently caused by:
a) Staphylococcus
b) Streptococcus
c) Escherichia coli
d) Pseudomonas aeruginosa
e) Klebsiella pneumoniae

86. CS. What is asepsis?


a) A complex of measures, directed to sterilize the air in the operating room
b) A complex of measures, directed to destruction of microorganisms in the wound
c) A complex of measures, directed to prevent the entry of microorganisms into the operating wound
d) A complex of measures, directed to sterilize surgical instruments
e) A complex of measures, directed to destruction of microorganisms by physical methods

87. CM. Endogenous infection can cause suppuration of postoperative wounds, penetrating to the human body in
the following ways:
a) Contact spread
b) Lymphogenous way
c) Airborne spread
d) Hematogenous way
e) Contamination by implantation

88. CM. Which of the following possible sources of infection belong to endogenous?
a) Air in the operating room
b) Surgical instruments
c) Carious teeth in a patient, which will be operated for hip joint replacement
d) Chronic pyelonephritis in a patient, to be operated for uncomplicated inguinal hernia
e) Suture material

89. CM. Specify the possible sources of endogenous infection.


a) Synthetic arterial prostheses
b) Chronic sinusitis in a patient
c) Conditionally pathogenic flora of the colon
d) Surgeon’s hands
e) Suture material
90. CM. Specify the possible sources of exogenous infection in a patient, who is operated on for uncomplicated
inguinal hernia?
a) Synthetic mesh, used for hernia repair
b) Chronic tonsillitis, existing in a patient
c) Chronic caries in a patient
d) Respiratory infection, existing in a surgeon
e) Surgical instruments used for surgery

91. CS. It is planned to perform five surgical interventions during the day in one operating room. What kind of
procedure should be carried out first?
a) Gastric resection for peptic ulcer disease
b) Phlebectomy for varicose veins of the lower extremity
c) Repeated surgical debridement for phlegmon of the soft tissues
d) Cholecystectomy in cholelithiasis
e) Resection of sigmoid colon for malignant tumor

92. CM. Specify necessary actions of a surgeon during preparation for surgery.
a) Decontamination and limiting of the operative field with a sterile drapes
b) Sterilization of suture material
c) Decontamination of hands
d) Use of surgical gloves
e) Putting on a mask

93. CM. Operating bloc (department) is divided into the following areas of sterility:
a) Area of limited regime
b) Area of absolute sterility
c) Area of non-sterile regime
d) Area of relative sterility
e) Area of general hospital regime

94. CS. How often is a general cleaning performed in the operating room?
a) Daily
b) Once in two days
c) Once in three days
d) Once a week
e) Once a month

95. CS. Accepted critical threshold for contamination of air in the operating room is:
a) Less than 100 germs/m3
b) Less than 200 germs/m3
c) Less than 300 germs /m3
d) Less than 400 germs/m3
e) Less than 500 germs/m3

96. CM. The methods of prevention of airborne infection in the operating room are:
a) Irradiation of air with ultraviolet rays
b) Decontamination of the operative field
c) Identification and treatment of health-workers who are carriers of pathogenic germs
d) Using of air bacterial filters
e) Wearing of surgical masks

97. CM. What elements of surgical attire are not sterile?


a) Surgical cap
b) Surgical mask
c) Scrub suite
d) Shoe covers
e) Surgical gown

98. CM. What distinguishes the ideal (super sterile) operating room from the usual operating room?
c) Temperature - 120°C
d) Temperature - 190°C
e) Time - 45 min

107. CS. When closed the Schimmelbusch box keeps contents sterile during:
a) 6 hours
b) 12 hours
c) 24 hours
d) 48 hours
e) 72 hours

108. CM. If the side holes of Schimmelbusch container are open, it can mean the following:
a) The contents of the box is sterile
b) The contents of the box is not sterile
c) The box is in the course of preparation for sterilization
d) The box is in the process of transportation after autoclaving
e) Sterility of the box content is under control

109. CS. How surgeon's hands are decontaminated with alcohol before surgery?
a) With 96% alcohol solution - 1 time during 1 min
b) With 96% alcohol solution - 2 times during 1 min
c) With 70% alcohol solution - 3 times during 1 min
d) With 70% alcohol solution - 4 times during 1 min
e) With 40% alcohol solution - 5 times during 1 min

110. CS. Which antiseptic can guarantee complete sterilization of the surgeon’s hands prior to surgery?
a) Chlorhexidine
b) 70% alcohol solution
c) Iodine
d) Triclosan
e) None of these

111. CM. Sterile surgical gloves are considered contaminated and should be changed during surgery if:
a) Glove becomes torn or punctured
b) Gloved hand touch anything that is not sterile
c) Glove becomes soiled with blood
d) Surgical intervention lasts for more than one hour
e) Gloved hands drops below the waist of surgeon

112. CS. Maximum interval of time from the moment of shaving (depilation) of surgical site till incision of the skin
should not exceed:
a) 30 min
b) 1 hour
c) 2 hours
d) 3 hours
e) 4 hours

113. CM. Specify the rule of patient’s skin preparation before surgery.
a) More contaminated areas are cleaned first
b) Skin is cleaned with antiseptic more widely than area of planed incision
c) Cleaning is performed from the center to periphery
d) Shaving (depilation) of surgical site should be performed immediately before the operation
e) The best method for hair removal is the use of electric epilator

114. CM. Decontamination of the operating field with antiseptic solutions should be repeated in the following
stages of surgery:
a) Before suturing of the skin
b) Before making a skin incision
c) Before dissection of the wall of a hollow organ
a) Laminar (straight) movement of air from the ceiling toward the floor
b) Air is supplied into the operating room through a bacterial filter
c) The operating room is divided into four areas of sterility
d) Strict observance of the aseptic rules
e) Access to the operating room is limited

99. CM. To the scrubbed (sterile) personnel working in the operating room refers:
a) Scrub nurse
b) Nurse
c) Surgeon
d) Surgeon’s assistant
e) Anesthesiologist

100. CM. What is the source of contact infection in surgery?


a) Surgeon’s hands
b) Surface of the operating table
c) Surgical dressings and linens
d) Surgical instruments
e) Suture material

101. CM. What physical methods of sterilization are used in modern asepsis?
a) Sterilization by dry heat
b) Hot steam sterilization under pressure (autoclaving)
c) Sterilization by radiation
d) Sterilization by gas
e) Sterilization with antiseptic solutions

102. CM. What chemical methods of sterilization are used in modern asepsis?
a) Sterilization by dry heat
b) Hot steam sterilization under pressure (autoclaving)
c) Sterilization by radiation
d) Sterilization with antiseptic solutions
e) Sterilization by gas

103. CS. Metallic surgical instruments are usually sterilized by:


a) Hot steam under pressure (autoclaving)
b) Dry heat
c) Boiling
d) Ionizing radiation
e) Placement in a 6% hydrogen peroxide solution

104. CS. Sterilization of metallic surgical instruments by dry heat is performed in the following regime (time +
temperature):
a) 1 hour at 120°С
b) 1 hour at 130°С
c) 1 hour at 150°С
d) 1 hour at 180°С
e) 1 hour at 200°С

105. CS. What method is used for sterilization of optical surgical instruments (laparoscope)?
a) Immersion in Betadine solution
b) Hot steam under pressure
c) Immersion in 3% solution of hydrogen peroxide
d) Immersion in glutaraldehyde “Cidex”
e) Dry heat

106. CM. Standard regime of sterilization of surgical linens in autoclave is:


a) Pressure - 0.5 atm
b) Pressure - 1.5 atm
d) Before limitation of operation field with sterile drapes
e) After suturing of the skin

115. CM. An implantable device is a device, that is placed into a surgically or naturally formed cavity of the human
body, if it is intended to remain there for a period of:
a) More than one week, but less than one month
b) More than two weeks, but less than one month
c) More than 24 hours, but less than one week
d) More than 30 days
e) Life-long

116. CM. Infection by implantation is characterized by the following:


a) Implantation of an infected item always results in development of infection
b) Implantation of an infected item very rare results in development of infection
c) Infection is resistant to antibacterial drugs
d) Clinical diagnosis of infection is not difficult
e) Treatment of infection usually requires the explantation (removal) of infected item

117. CS. Indicate the minimal quantity of pathogenic germs required for development of surgical site infection
(without implant in the wound).
a) 100 germs per 1 g of tissue
b) 1.000 germs per 1 g of tissue
c) More than 10.000 germs per 1 g of tissue
d) More than 100.000 germs per 1 g of tissue
e) More than 1.000.000 germs per 1 g of tissue

118. CM. Specify the sources of contamination by implantation of infected materials in surgery.
a) Surgical gloves
b) Suture material
c) Surgeon’s hands
d) Surface of operating table
e) Synthetic mesh for hernia repair

119. CS. Nowadays, the single acceptable method of sterilization of suture material is:
a) Boiling
b) Autoclaving
c) Placing in alcohol 96°
d) Irradiation with ultraviolet rays
e) Ionizing radiation

120. CM. What characterizes the direct method of sterility control?


a) It is the most rapid
b) It is the most accurate
c) It must be performed once every 7-10 days
d) It is performed using antipyrine
e) It must be performed daily

121. CS. What method of sterility control of surgical instruments and dressing material is the most accurate?
a) Test with phenolphthalein
b) Melting of ascorbic acid
c) Melting of antipyrine
d) Bacteriological study
e) Data of thermometry

122. CM. What is characteristic of indirect method of sterility control?


a) It is the most accurate
b) It indicates the presence or absence of microorganisms
c) It is used only to control the quality of thermal sterilization
d) It allows to determine the degree of bacterial contamination
e) It is used in every sterilization

123. CS. What is a nosocomial infection?


a) Infection acquired during hospitalization or as a result of treatment
b) Specific infections (tuberculosis, actinomycosis, syphilis)
c) A collective term meaning infectious disease, which led to hospitalization
d) Purulent surgical diseases of soft tissues with a typical localization
e) Inflammatory diseases of the upper respiratory tract as a source of endogenous infection

124. CM. When antibacterial prophylaxis is not indicated?


a) Surgery for uncomplicated varicose veins
b) Hip joint replacement
c) Gastrectomy for bleeding ulcer
d) Hernia repair without implant
e) Excision of benign skin tumor (papilloma)

125. CM. When antibacterial prophylaxis is mandatory?


a) Bowel resection for cancer
b) Implantation of synthetic aortic graft
c) Gastrectomy for bleeding ulcer
d) Hernia repair with synthetic mesh
e) Excision of benign skin tumor (lipoma), with installation of a tube for drainage which will be kept in the wound
for 24 hours

126. CS. Indicate the “first-line” drugs for antibacterial prophylaxis in general surgical interventions.
a) Aminoglycoside
b) Fluorchinolone
c) First-generation cephalosporins
d) Third-generation cephalosporins
e) Sulifanilamides

127. CM. How to perform the antibacterial prophylaxis correctly?


a) First dose of drug is administrated immediately after wound closure
b) Single maximal dose of drug should be used
c) Prolonged surgery and significant blood loss require administration of additional doses
d) Preferred way of drug administration is intramuscular
e) Drug should be administrated 1 hour before surgery

128. CМ. Who discovered the blood groups?


a) V.Iurevici
b) K.Landsteiner
c) N.Rosengart
d) J.Jansky
e) A.Wiener

129. CМ. Who discovered the Rh factor?


a) V.Iurevici
b) K.Landsteiner
c) N.Rosengart
d) J.Jansky
e) A.Wiener

130. CМ. The contemporary era in the development of transfusiology is characterized by the following principles:
a) Reduction of the indications for blood transfusion
b) Extension of the indications for blood transfusion
c) Widespread use of blood substitutes
d) Preferential transfusion of whole blood
e) Practically complete renouncement of direct blood transfusion

131. CS. What is the blood group?


a. A complex of leukocyte’s antigens
b. A complex of immunoglobulins
c. A complex of erythrocyte’s antigens
d. A complex of platelet’s antigens
e. Antigens of transplantation

132. CМ. The first blood group is characterized by:


a) The presence of antigen B on the surface of erythrocytes
b) The presence of antigen A on the surface of erythrocytes
c) The absence of antigens A and B on the surface of erythrocytes
d) The presence of antibodies α and β in the plasma
e) The absence of antibodies α and β in the plasma

133. CМ. The second blood group is characterized by:


a) The presence of antigen B on the surface of erythrocytes
b) The presence of antigen A on the surface of erythrocytes
c) The absence of antigens A and B on the surface of erythrocytes
d) The presence of antibodies α in the plasma
e) The presence of antibodies β in the plasma

134. CМ. The third blood group is characterized by:


a) The presence of antigen B on the surface of erythrocytes
b) The presence of antigen A on the surface of erythrocytes
c) The absence of antigens A and B on the surface of erythrocytes
d) The presence of antibodies α in the plasma
e) The absence of antibodies β in the plasma

135. CМ. The fourth blood group is characterized by:


a) The presence of antigen B on the surface of erythrocytes
b) The presence of antigen A on the surface of erythrocytes
c) The absence of antigens A and B on the surface of erythrocytes
d) The presence of antibodies α and β in the plasma
e) The absence of antibodies α and β in the plasma

136. CМ. Ottenberg's rule:


a) Allows transfusion of the second blood group to the recipient with the fourth blood group
b) Allows transfusion of the first blood group to the recipient with the second blood group
c) Approve, that the agglutinins of infused blood are diluted in vascular bed of patient and are not able to
agglutinate the agglutinins of the recipient
d) Approve, that the agglutinins of infused blood are diluted in vascular bed of patient and are not able to
agglutinate the erythrocytes of the recipient
e) Is valid only in transfusion of small (less than 500 mL) volume of blood

137. CМ. Ottenberg's rule:


a) Allows transfusion of the second blood group to the recipient with the third blood group
b) Allows transfusion of the third blood group to the recipient with the first blood group
c) Approve, that the agglutinins of infused blood are diluted in vascular bed of patient and are not able to
agglutinate the agglutinins of the recipient
d) Approve, that the agglutinins of infused blood are diluted in vascular bed of patient and are not able to
agglutinate the erythrocytes of the recipient
e) Is valid in significant (1:20 or more) dilution of donated blood in the blood of the recipient

138. CS. The „universal recipient” (according to the Ottenberg's rule) is called individual with:
a) The first blood group
b) The second blood group
c) The third blood group
d) The fourth blood group
e) The first and fourth blood group

139. CS. The „universal donor” (according to the Ottenberg's rule) is called individual with:
a) The first blood group
b) The second blood group
c) The third blood group
d) The fourth blood group
e) The first and fourth blood group

140. CМ. Specify methods for determination (typing) of blood group according to AB0 system.
a) With standard platelets
b) With standard erythrocytes
c) With standard serums
d) With monoclonal antibodies
e) With monoclonal antigens

141. CМ. Choose methods for determination (typing) of blood group according to AB0 system.
a) With monoclonal Anti-A antibodies
b) With monoclonal Anti-B antibodies
c) With monoclonal Anti-0 antibodies
d) With monoclonal Anti-D antibodies
e) With monoclonal Anti-HLA antibodies

142. CМ. During determination (typing) of blood group with standard serums the following actions are performed:
a) In each drop of serum is added one drop of patient’s blood with 10 times smaller volume
b) In each drop of serum is added one drop of patient’s blood with 10 times larger volume
c) Assess the presence of agglutination no earlier than in 5 minutes
d) It is performed at a temperature of +15-25°C
e) It is performed at a temperature not above +15°C

143. CS. During the determination of blood group by serum-test, agglutination with all standard serums (group I,
group II and group III) was obtained. Indicate the correct blood group.
a. 0(I)
b. A(II)
c. B(III)
d. AB(IV)
e. Error in blood group determination

144. CS. During the determination of blood group by erythrocyte -test, agglutination with standard erythrocytes of
I, II and III blood groups was obtained. Indicate the correct blood.
a. 0(I)
b. A(II)
c. B(III)
d. AB(IV)
e. Error in blood group determination

145. CS. During the determination of blood group with monoclonal antibodies (Tsoliclon-test), agglutinations with
Tsoliclon Anti-A and Tsoliclon Anti-B was obtained. Indicate the correct blood group.
a. 0(I)
b. A(II)
c. B(III)
d. AB(IV)
e. Error in blood group determination

146. CS. During the determination of blood group by serum-test, no agglutination was obtained with all standard
serums (group I, group II and group III). Indicate the correct blood group.
a. 0(I)
b. A(II)
c. B(III)
d. AB(IV)
e. Error in blood group determination

147. CS. During the determination of blood group with monoclonal antibodies (Tsoliclon-test), no agglutination was
obtained with Tsoliclon Anti-A and Tsoliclon Anti-B. Indicate the correct blood group.
a. 0(I)
b. A(II)
c. B(III)
d. AB(IV)
e. Error in blood group determination

148. CS. During the determination of blood group by serum-test, agglutination with standard serums of group II
and group III was obtained and no agglutination with serum of group I was registered. Indicate the correct
blood group.
a. 0(I)
b. A(II)
c. B(III)
d. AB(IV)
e. Error in blood group determination

149. CS. During the determination of blood group by serum-test, agglutination with standard serum of group I and
group II was obtained and no agglutination with serum of group III was registered. Indicate the correct blood
group.
a. 0(I)
b. A(II)
c. B(III)
d. AB(IV)
e. Error in blood group determination

150. CS. During the determination of blood group with monoclonal antibodies (Tsoliclon-test), agglutination was
obtained with Tsoliclon Anti-A and no agglutination was registered with Tsoliclon Anti-B. Indicate the correct
blood group.
a. 0(I)
b. A(II)
c. B(III)
d. AB(IV)
e. Error in blood group determination

151. CS. During the determination of blood group with monoclonal antibodies (Tsoliclon-test), agglutination was
obtained with Tsoliclon Anti-B and no agglutination was registered with Tsoliclon Anti-A. Indicate the correct
blood group.
a. 0(I)
b. A(II)
c. B(III)
d. AB(IV)
e. Error in blood group determination

152. CМ. For determination of the Rh-factor is used:


a) Method with monoclonal antibodies anti-D
b) Coombs test (indirect antiglobulin test)
c) Method with anti-D standard serum
d) Reaction of conglutination using gelatin
e) Method with monoclonal antibodies anti-HLA

153. CS. What method of Rh-factor determination is used only in immunological laboratory?
a. Anti-Rh serum test
b. Test with monoclonal reagent-D
c. Reaction of conglutination in gelatin
d. Indirect antiglobulin test (Coombs test)
e. Test for individual compatibility by Rh-factor

154. CМ. Method of blood autotransfusion is performed as follows:


a) Collecting of blood from patient is done several weeks before elected surgery
b) Several samplings of 400-500 mL of blood with an interval of one week are performed
c) Several samplings of 100-200 mL of blood with an interval of two days are performed
d) Last collecting is made no later than 12 hours prior to surgery
e) Last collecting is made no later than 72 hours prior to surgery

155. CМ. Acute preoperative normovolemic haemodilution means the following measures:
a) Exfusion of 400-500 mL of patient's blood one week before surgery
b) Exfusion of 750-1000 mL of patient's blood immediately before surgery
c) Replacement of collected blood with the blood substitutes solutions
d) Replacement of collected blood with the donor plasma
e) The collected blood is transfused back to the patient during or after surgery

156. CS. In a patient with traumatic rupture of the spleen and hemoperitoneum, the blood from abdominal cavity
was collected using the “Cell Saver” device and transfused intravenously to the patient. How is called this
method?
a) Managed haemodilution
b) Reinfusion
c) Rehydration
d) Plasmapheresis
e) Hemodialysis

157. CМ. Blood reinfusion can be performed in case of:


a) Gastric ulcer bleeding
b) Traumatic rupture of the spleen
c) Ruptured ectopic gestation
d) Traumatic amputation of the lower limb
e) Blunt thoracic trauma with hemothorax

158. CS. Reinfusion of blood collected from the abdominal cavity is impossible in case of:
a. Injury of diaphragm
b. Injury of spleen
c. Injury of abdominal hollow organs
d. Injury of large blood vessels
e. Poor general status of patient

159. CМ. Choose the components of blood used in transfusiology.


a) Albumin solution
b) Packed red blood cells
c) Granulocyte concentrate
d) Fresh frozen plasma
e) Cryoprecipitate

160. CМ. Choose the components of blood used in transfusiology.


a) Platelet concentrate
b) Immunoglobulins and gamma-globulin
c) Lyophilized thrombin
d) Fresh frozen plasma
e) Packed red blood cells

161. CS. Which of the following does not refer to the blood components?
a) Gamma-globulin
b) Packed red blood cells
c) Platelet concentrate
d) Granulocyte concentrate
e) Fresh frozen plasma

162. CМ. Choose the plasma derivates used in transfusiology.


a) Fresh frozen plasma
b) Albumin solution
c) Lyophilized thrombin
d) Immunoglobulins and gamma-globulin
e) Packed red blood cells

163. CS. Choose the plasma derivates used in transfusiology.


a) Fresh frozen plasma
b) Ringer’s solution
c) Platelet concentrate
d) Hartmann's solution
e) Cryoprecipitate

164. CS. Which of the following does not refer to the plasma derivates?
a. Thrombin
b. Fibrinogen
c. Haemostatic sponge
d. Platelet concentrate
e. Gamma-globulin

165. CМ. Erythrocyte concentrate (packed red blood cells) is characterized by the following:
a) The hematocrit of concentrate is about 85-95%
b) The hematocrit of concentrate is about 35-45%
c) One container (1 dose) of concentrate contains 50-100 mL
d) One container (1 dose) of concentrate contains 200-300 mL
e) Transfusion is performed according to compatibility by AB0 and Rh-systems

166. CS. What temperature is required for red blood cells concentrate preservation?
a. 0°C
b. -2-4°C
c. +4+6°C
d. +8+10°C
e. 0+1°C

167. CS. At what level of hemoglobin the transfusion of packed red blood cells is always indicated?
a) Less than 70 g/L
b) Less than 80 g/L
c) Less than 90 g/L
d) Less than 100 g/L
e) Less than 110 g/L

168. CS. At what level of hemoglobin the transfusion of packed red blood cells is not recommended?
a) Over 60 g/L
b) Over 70 g/L
c) Over 80 g/L
d) Over 90 g/L
e) Over 100 g/L

169. CS. At what level of hemoglobin the indications for transfusion of packed red blood cells depend on the
clinical data (general condition of the patient, presence of concomitant diseases, severity of hypoxia)?
a) 40-60 g/L
b) 60-70 g/L
c) 70-100 g/L
d) 100-110 g/L
e) 110-120 g/L

170. CS. After a single dose of transfused packed red blood cells is expected an increase of hemoglobin level by
an average:
a) 5 g/L
b) 10 g/L
c) 15 g/L
d) 20 g/L
e) 25 g/L

171. CS. After a single dose of transfused packed red blood cells is expected an increase of hematocrit level by an
average:
a) 5%
b) 4%
c) 3%
d) 2%
e) 1%

172. CМ. Platelet concentrate is characterized by the following:


a) One dose of concentrate has a volume of 50-60 mL
b) One dose of concentrate has a volume of 200-300 mL
c) It is administered in calculation of one dose of concentrate per 30 kg of body weight
d) It is administered in calculation of one dose of concentrate per 10 kg/body weight
e) It is transfused according to compatibility by AB0 and Rh-system

173. CМ. The platelet concentrate transfusion is indicated if:


a) The number of platelets is less than 100x109/L (in patients with bleeding or risk of bleeding)
b) The number of platelets is less than 50x109/L (in patients with no bleeding or risk of bleeding)
c) The number of platelets is less than 50x109/L (in patients with bleeding or risk of bleeding)
d) The number of platelets is less than 100x109/L (regardless of bleeding risk)
e) The number of platelets is less than 20x109/L (in patients with no bleeding or risk of bleeding)

174. CМ. Transfusion of the fresh frozen plasma:


a) Is used for replacement of circulating blood volume
b) Is used for prophylaxis and treatment of coagulopathies
c) Is performed after its defrosting and warming up to 37°С
d) Is transfused in usual dose of 15 mL per kilogram of patient’s weight
e) Is transfused in usual dose of 5 mL per kilogram of patient’s weight

175. CМ. Which of the following coagulation factors contains cryoprecipitate?


a) The factor VIII
b) The factor II
c) The fibrinogen
d) The factor Willebrand
e) The factor XIII

176. CМ. The plasma derivate thrombin is characterized by the following:


a) Besides thrombin itself it contains the small amount of thromboplastin and calcium chloride
b) It has a hemostatic effect due to convertion of fibrinogen into fibrin and activation of factor XIII
c) It is produced in a lyophilized form, which prior to application is diluted with sterile saline solution
d) It is administered only locally (in the tissue of the source of bleeding)
e) It is administered only systemically (intravenously or intra-arterial)

177. CМ. According to international classification the blood substitutes are divided into:
a) Solutions with rapid action
b) Solutions with slow action
c) Colloidal solutions
d) Crystalloid solutions
e) Plasma derivated

178. CМ. Choose the crystalloid blood substitutes.


a) Ringer's solution
b) Gelatin preparations (Gelofusin, Ghemaxel)
c) Normal saline 0.9% sodium chloride solution
d) Dextrans (Dextran 70 or Poliglucin, Dextran 40 or Reopoliglucin)
e) Hartmann's solution

179. CМ. Choose the colloidal blood substitutes.


f) Starch derivatives (Hydroxyethyl starch 450 or Refortan)
a) Ringer's solution
b) Dextrans (Dextran 70 or Poliglucin, Dextran 40 or Reopoliglucin)
c) Normal saline 0.9% sodium chloride solution
d) Gelatin preparations (Gelofusin, Ghemaxel)

180. CМ. Crystalloid blood substitutes are characterized by the following:


a) After intravenous administration, they leave quickly the bloodstream and penetrate the tissues
b) They are able to remain in the bloodstream up to 8 hours
c) After transfusion of one liter of solution, the circulating blood volume increases with approximately 250 mL
d) In case of hemorrhage, they should be administered in a volume of at least three times higher compared to
the volume of blood lost
e) In case of hemorrhage, they are usually transfused into a 1:1 ratio to the volume of blood lost

181. CМ. Colloidal blood substitutes are characterized by the following:


a) After intravenous administration, they leave quickly the bloodstream and penetrate the tissues
b) They are able to remain in the bloodstream up to 8 hours
c) After transfusion of one liter of solution, the circulating blood volume increases with approximately 250 mL
d) In case of hemorrhage, they should be administered in a volume of at least three times higher compared to
the volume of blood lost
e) In case of hemorrhage, they are usually transfused into a 1:1 ratio to the volume of blood lost

182. CS. Which of the following effect cause transfusion of the Polyglucine (colloidal blood substitute)?
a. Hypervolemic (increase of circulating blood volume)
b. Desaggregation of blood cells
c. Antithrombotic
d. Diuretic
e. Detoxification

183. CM. Quality control of blood before transfusion include the following steps:
a. Bacteriology (culture) of blood for transfusion
b. Check up information from the label of container (group, Rh, component type, date)
c. Determination of Rh-factor of blood from container
d. Visual inspection of blood quality in transparent container
e. Check up of container integrity

184. CS. Test for individual compatibility according to AB0 system, performed before blood transfusion, is based
on agglutination between:
a. Erythrocyte’s antigens of recipient and plasma antibodies of donor
b. Erythrocyte’s antigens of donor and plasma antibodies of recipient
c. Erythrocyte’s antigens of donor and plasma antibodies of recipient, at the temperature of 38-40°C
d. Erythrocyte’s antigens of recipient and plasma antibodies of donor, at the temperature of 38-40°C
e. Erythrocyte’s antigens of recipient and leukocyte’s antigens of donor

185. CS. What test should be done prior of transfusion of fresh frozen plasma?
a. No tests required
b. Test for compatibility by AB0 system
c. Test for compatibility by Rh-factor
d. Biological test
e. Skin-test with plasma

186. CS. What test is not performed during the blood transfusion?
a. Visual inspection of blood quality
b. Blood group determination (typing)
c. Test for compatibility by AB0 system and Rh-factor
d. Test for individual compatibility by HLA system (human leukocyte antigens)
e. Biological test

187. CS. How is performed the biological test during blood transfusion?
a. Bolus transfusion of 25 mL of blood, repeated twice with the interval of 5 minutes
b. Bolus transfusion of 15 mL of blood, repeated twice with the interval of 3 minutes
c. Slow transfusion of blood during 15 minutes, followed by 3-5 minutes of expectation
d. Single bolus transfusion of 25 mL of blood
e. Biological test is performed only in case of plasma transfusion

188. CS. The maximum allowed time from the moment of puncture of container with the blood component until the
end of transfusion is:
a) 1 hour
b) 2 hours
c) 3 hours
d) 4 hours
e) 5 hours

189. CМ. Specify the correct rate (rapidity) for transfusion of blood components.
a) The packed red blood cells are transfused drop by drop, at a rate of 40-60 drops per minute
b) The packed red blood cells are transfused streamly (in jet)
c) The fresh frozen plasma is transfused drop by drop, at a rate of 40-60 drops per minute
d) The fresh frozen plasma is transfused streamly (in jet)
e) The platelet concentrate is transfused streamly (in jet)

190. CМ. From the start of transfusion the patient's condition is assessed:
a) After 5 minutes
b) After 15 minutes
c) After 30 minutes
d) Every hour
e) Every two hours

191. CМ. After completing of transfusion the patient's condition is assessed:


a) After 5 minutes
b) After 15 minutes
c) After one hour
d) After two hours
e) After one day

192. CМ. According to current classification, posttransfusion reactions and complications are divided into:
a) Early (acute)
b) Late
c) Delayed
d) Immune
e) Non-immune

193. CМ. Acute non-immune posttransfusion reactions and complications include:


a) Pyrogenic reactions
b) Febrile non-haemolytic antigenic reactions
c) Transfusional shock
d) TRALI syndrome (Transfusion Related Acute Lung Injury)
e) TACO (Transfusion Associated Circulatory Overload)

194. CМ. Acute non-immune posttransfusion reactions and complications include:


a) Acute sepsis and endotoxic shock
b) Citrate toxicity and hyperpotassemia
c) Allergic reactions (urticaria)
d) Air embolism, thromboembolism
e) Anaphylactic reactions

195. CМ. Acute immune posttransfusion reactions and complications include:


a) Pyrogenic reactions
b) Febrile non-haemolytic antigenic reactions
c) Transfusional shock
d) TRALI syndrome (Transfusion Related Acute Lung Injury)
e) TACO (Transfusion Associated Circulatory Overload)

196. CМ. Acute immune posttransfusion reactions and complications include:


a) Acute sepsis and endotoxic shock
b) Citrate toxicity and hyperpotassemia
c) Allergic reactions (urticaria)
d) Air embolism, thromboembolism
e) Anaphylactic reactions

197. CМ. The mild posttransfusion reactions are characterized by:


a) Elevation of body temperature with 1°C
b) Elevation of body temperature with 1,5-2°C
c) Elevation of body temperature with more than 2°C
d) Headache
e) Chills

198. CМ. The posttransfusion reactions of moderate severity are characterized by:
a) Elevation of body temperature with 1°C
b) Elevation of body temperature with 1,5-2°C
c) Elevation of body temperature with more than 2°C
d) Headache
e) Chills

199. CМ. The severe posttransfusion reactions are characterized by:


a) Elevation of body temperature with 1°C
b) Elevation of body temperature with 1,5-2°C
c) Elevation of body temperature with more than 2°C
d) Headache
e) Chills

200. CМ. Within blood transfusion the recipient can be contaminated by the following infections:
a) Human immunodeficiency virus type 1 and 2
b) Viral hepatitis B and C
c) Viral hepatitis A and E
d) Syphilis
e) Cytomegalovirus infections

201. CM. What periods do not refer to evolution of the hemolytic shock?
a. Period of shock
b. Period of acute renal failure
c. Period of septic toxemia
d. Period of urine output recovery
e. Period of hemodynamic stabilization

202. CS. What does the term “symptom” mean?


a) A complex of signs characteristic of a pathological process
b) A single sign of a pathological process
c) Nosologic unit (disease)
d) A group of similar pathological conditions
e) Morphological changes characteristic of a certain disease

203. CM. Specify which of the following conditions refers to symptoms.


a) Obesity
b) Pain
c) Vomiting
d) Pneumothorax
e) Dyspnea

204. CM. Specify which of the following conditions refers to symptoms.


a) Fever
b) Gangrene
c) Nausea
d) Tachycardia
e) Hemothorax

205. CS. What does the term “syndrome” mean?


a) Morphological changes characteristic of a certain disease
b) A single sign of a pathological process
c) A complex of symptoms which have a common origin and characteristic of a certain pathological process
d) The sequence of appearance of pathological signs
e) Pathology of several organs

206. CM. The examples of pathological syndromes are:


a) Systemic inflammatory response
b) Fever in purulent arthritis
c) Chronic arterial insufficiency of lower extremities
d) Tympanitis
e) Intestinal obstruction

207. CM. Specify the examples of pathological syndromes.


a) Chronic venous insufficiency
b) Acute lower limb ischemia
c) Absent pulsation of the femoral artery
d) Swelling
e) Respiratory failure

208. CM. What parts of a history of disease include subjective information?


a) Complaints of the patient
b) History of present illness
c) History of life
d) Local status
e) Results of instrumental studies

209. CS. What parts of a history of disease include objective information?


a) Complaints of the patient
b) History of present illness
c) History of life
d) Allergological anamnesis
e) Data of physical examination

210. CS. In surgical patients percussion can determine:


a) The presence of tumor
b) The presence of a foreign body in tissues
c) An accumulation of fluid or air in the cavities
d) The presence of inflammatory process
e) The mobility of organ

211. CM. In surgical patients auscultation can determine:


a) The presence of cardiac and vascular sounds
b) The presence of intestinal peristalsis
c) The character of respiratory sounds
d) The presence of tumor
e) The presence of inflammatory process

212. CM. Specify the data from a history of disease relating to the description of local status.
a) On admission the patient complained of abdominal pain
b) In the lower third of the left leg hyperpigmentation of the skin and ulcerative defect are determined
c) The heart rate is 70 beats per minute
d) In the right inguinal region a painless mass of elastic consistency, which can be reduced into the abdominal
cavity is determined
e) According to patient, abdominal pain developed about 18 hours ago

213. CM. Preliminary diagnosis is based on the following data:


a) Complaints of the patient
b) Data of physical examination
c) Results of laboratory tests
d) Results of instrumental studies
e) Results of histomorphological study

214. CM. Clinical diagnosis is based on the following data:


a) Subjective data of patients
b) Data of physical examination
c) Results of laboratory tests
d) Results of instrumental studies
e) Results of histomorphological study

215. CM. Final (definitive) diagnosis is made based on the following components:
a) Diagnosis of basic disease
b) Diagnosis of concomitant illnesses
c) Complications of basic disease
d) Preliminary diagnosis
e) Differential diagnosis

216. CM. Specify characteristic features of torticollis.


a) The patient's face turned to the healthy side
b) The patient's face turned to the affected side
c) The patient's head tilted to the healthy side
d) The patient's head tilted to the affected side
e) Cicatricial shortening of the sternocleidomastoid muscle

217. CS. The diagnosis of torticollis is made on the basis of:


a) Physical examination of a patient
b) Laboratory tests
c) X-ray examination and tomography
d) Biochemical analysis
e) Invasive instrumental studies

218. CM. Specify characteristic features of inflammatory lesions of cervical lymph nodes.
a) Lymph nodes are mobile
b) Lymph nodes are fixed to the surrounding tissues
c) Lymph nodes are painless to palpation
d) Lymph nodes are painful on palpation
e) Lymph nodes have a “stony” density

219. CM. Specify characteristic features of metastatic tumor lesions of cervical lymph nodes.
a) Hyperemia of skin over the lymph nodes
b) Lymph nodes are fixed to the surrounding tissues
c) Lymph nodes are painless to palpation
d) Lymph nodes are painful on palpation
e) Lymph nodes have irregular contours

220. CM. Specify the rules of palpation of the thyroid gland.


a) The physician palpates the gland standing behind the patient
b) The physician palpates the gland situated in front of patient
c) The physician palpates the gland with one hand
d) The physician palpates the gland with two hands
e) The physician palpates the gland during swallowing

221. CM. Specify the most common benign tumors of the head.
a) Melanoma
b) Atheroma
c) Papilloma
d) Adenocarcinoma
e) Hematoma

222. CM. Specify the surgical instruments designed for dissection of tissues.
a) Retractor Finochietto
b) Scissors
c) Gigli wire saw
d) Scalpel
e) Retractor Farabeuf

223. CM. Specify the surgical instruments, designed for hemostasis.


a) Kocher forceps
b) Anatomical forceps (pincettes)
c) Needle holder
d) Pean forceps
e) Surgical forceps (pincettes)

224. CM. Specify the surgical instruments, designed for grasping of tissues.
a) Kocher forceps
b) Anatomical forceps (pincettes)
c) Needle holder
d) Pean forceps
e) Surgical forceps (pincettes)

225. CS. What retractor has teeth at the end of the working blade?
a) Fritch retractor
b) Farabeuf retractor
c) Finochietto retractor
d) Volkmann retractor
e) Rectal dilator

226. CM. Specify mechanical retractors.


a) Dartigues retractor
b) Retractor of Kirschner
c) Farabeuf retractor
d) Volkmann retractor
e) Abdominal (hepatic) retractor

227. CS. Specify the surgical instruments, designed for exploration.


a) Kocher forceps
b) Anatomical forceps (pincettes)
c) Finochietto retractor
d) Probe
e) Rectal dilator

228. CM. Round surgical needles are used for suturing of:
a) Vessels
b) Intestine
c) Skin
d) Periosteum
e) Fascia

229. CM. Triangular (cutting) surgical needles are used for suturing of:
a) Fascia
b) Parenchimatous organs
c) Intestine
d) Vessels
e) Skin

230. CM. What curvature may a surgical needle have?


a) 1/4 of circle
b) 1/3 of circle
c) 4/6 of circle
d) 3/8 of circle
e) 1/2 of circle

231. CM. What are the requirements for suture materials used in surgery?
a) Absence of carcinogenic activity
b) Resistance to infection
c) Possibility of long-term use
d) Simplicity of sterilization
e) Safety of a knot

232. CM. Specify the biological suture material used in surgery.


a) Metallic thread
b) Kapron
c) Lavsan
d) Catgut
e) Silk

233. CM. Specify the synthetic absorbable suture material used in surgery.
a) Dexon
b) Kapron
c) Polypropylene
d) Catgut
e) Vycril

234. CM. Specify the synthetic nonabsorbable suture material used in surgery.
a) Dexon
b) Kapron
c) Polypropylene
d) Catgut
e) Vycril

235. CS. According to classification of suture material, polypropylene refers to:


a) Synthetic monofilament nonabsorbable sutures
b) Synthetic monofilament absorbable sutures
c) Biological multifilament nonabsorbable sutures
d) Biological monofilament absorbable sutures
e) Synthetic multifilament absorbable sutures

236. CS. According to classification of suture material, kapron refers to:


a) Synthetic monofilament nonabsorbable sutures
b) Synthetic monofilament absorbable sutures
c) Biological multifilament absorbable sutures
d) Biological monofilament absorbable sutures
e) Synthetic multifilament nonabsorbable sutures

237. CS. According to classification of suture material, silk refers to:


a) Synthetic monofilament nonabsorbable sutures
b) Synthetic monofilament absorbable sutures
c) Biological multifilament nonabsorbable sutures
d) Biological monofilament absorbable sutures
e) Synthetic multifilament nonabsorbable sutures

238. CS. The most safer knot, used in surgery is considered:


a) Simple (direct) knot
b) Slip knot
c) Surgeon’s knot
d) Instrument tying knot
e) Square knot

239. CM. Specify the types of interrupted sutures, which are used for closure of skin wounds in surgery.
a) Blair-Donati suture
b) Suture in “U”
c) Intracutaneous suture
d) Simple suture
e) Simple running suture

240. CM. Specify the types of continuous sutures, which are used for closure of skin wounds in surgery.
a) Intracutaneous suture
b) Simple running suture
c) Mechanical suture
d) Blair-Donati suture
e) Suture in “U”
241. CM. Specify which of the listed conditions are related to surgical infection.
a. Acute pneumonia
b. Acute hydradenitis
c. Liver abscess
d. Postoperative wound suppuration
e. Chronic pyelonephritis

242. CM. What infectious processes are related to acute specific surgical infection?
a. Anthrax
b. Syphilis
c. Tetanus
d. Actinomycosis
e. Tuberculosis

243. CS. According to classification of surgical infection by clinical evolution, tuberculosis refers to:
a. Chronic non-specific infection
b. Chronic specific infection
c. Acute specific infection
d. Acute putrid infection
e. Acute anaerobic infection

244. CS. According to classification of surgical infection by clinical evolution, tetanus refers to:
a. Chronic non-specific infection
b. Chronic specific infection
c. Acute specific infection
d. Acute putrid infection
e. Acute anaerobic infection

245. CM. Specify the factors, which lead to the development of surgical infection.
a. Presence of virulent pathogenic microorganisms
b. Venous congestion of tissues
c. Response of human body to infection
d. Transient bacteremia
e. Presence of the penetration route for infection

246. CM. Which biological characteristics of microorganisms determine the evolution of surgical infection?
a. Invasiveness
b. Virulence
c. Grade of contamination
d. Toxicity
e. Spore-forming

247. CM. Specify the possible sources of surgical infection.


a. Exogenous
b. Implantation
c. Contact
d. Endogenous
e. Airborne

248. CM. Specify the exogenous source of surgical infection.


a. Dental caries
b. Abrasions
c. Chronic tonsillitis
d. Ducts of the sweat glands
e. Wounds

249. CM. Specify the non-specific mechanisms of the human body response to infection.
a. Reaction of inflammation
b. Saprophytic microflora of the human body
c. Humoral factors contained in plasma
d. Phagocytosis
e. Humoral immune response

250. CM. What is the role of mononuclear phagocytes in the inflammatory focus?
a. Formation of granulation tissue
b. Absorption and destruction of microbial bodies and their fragments
c. Release of cytokines
d. Synthesis of antibodies to foreign antigens
e. Direct action on microbial antigen (killer cells)

251. CS. Which leukocytes provide the humoral immune response to surgical infection?
a. Monocytes
b. B-lymphocytes
c. Neutrophil leukocytes
d. Phagocytes
e. T-lymphocytes

252. CS. Specify the mechanism of skin hyperemia in purulent surgical infection of soft tissues.
a. Local acceleration of catabolic reactions, accompanied by release of energy
b. Vasodilation as a result of histamine effect and acidosis in the focus of inflammation
c. Increased vascular permeability in the focus of inflammation and extravasation of fluid
d. Action of serotonin and histamine on nerve endings in the focus of inflammation and their compression by
tissue edema
e. Blood imbibition of soft tissues

253. CS. Specify the mechanism of skin hyperthermia in purulent surgical infection of soft tissues.
a. Local acceleration of catabolic reactions, accompanied by release of energy
b. Vasodilation as a result of histamine effect and acidosis in the focus of inflammation
c. Increased vascular permeability in the focus of inflammation and extravasation of fluid
d. Action of serotonin and histamine on nerve endings in the focus of inflammation and their compression by
tissue edema
e. Blood imbibition of soft tissues

254. CS. Specify the mechanism of pain in purulent surgical infection of soft tissues.
a. Local acceleration of catabolic reactions, accompanied by release of energy
b. Vasodilation as a result of histamine effect and acidosis in the focus of inflammation
c. Increased vascular permeability in the focus of inflammation and extravasation of fluid
d. Action of serotonin and histamine on nerve endings in the focus of inflammation and their compression by
tissue edema
e. Blood imbibition of soft tissues

255. CS. Specify the mechanism of edema in purulent surgical infection of soft tissues.
a. Local acceleration of catabolic reactions, accompanied by release of energy
b. Vasodilation as a result of histamine effect and acidosis in the focus of inflammation
c. Increased vascular permeability in the focus of inflammation and extravasation of fluid
d. Action of serotonin and histamine on nerve endings in the focus of inflammation and their compression by
tissue edema
e. Blood imbibition of soft tissues

256. CS. Specify the cause of limb function disturbance in acute purulent surgical infection.
a. Purulent-necrotic destruction of the ligaments and joints
b. Plegia due to toxic neuritis
c. Severe pain syndrome
d. Severe disorders of arterial blood supply
e. Distal venous thrombosis

257. CS. In surgical soft tissue infection pulsating pain is characteristic of the:
a. Phase of infiltration
b. Phase of wound contraction
c. Phase of resorption of the inflammatory exudate
d. Phase of abscess formation
e. Phase of spontaneous blow of abscess through the skin

258. CM. When the purulent cavity is formed, unlike infiltrative phase of inflammation, the following symptoms can
be revealed:
a. Hardening
b. Softening
c. Fluctuation
d. Dullness
e. Functional disturbance of affected segment

259. CS. What does the symptom of fluctuations in surgical soft tissue infection suggest?
a. Formation of the cavity filled with pus
b. Presence of inflammatory process in the phase of infiltration
c. Presence of gas in the tissues
d. Anaerobic non-clostridial nature of infection
e. Involvement in the pathological process of deeper lying organs

260. CM. Specify the typical changes in blood analysis during acute surgical infection.
a. Leukocytosis
b. Thrombocytosis
c. Shift of leukocyte formula to the left
d. Increased erythrocyte sedimentation rate
e. Eosinophilia

261. CM. Specify the local complications of inflammatory processes, located on the soft tissues of extremities.
a. Lymphangitis
b. Lymphadenitis
c. Bacterial pneumonia
d. Sepsis
e. Thrombophlebitis

262. CM. What characterizes lymphangitis?


a. Without treatment leads to bacteremia
b. Is a purulent inflammation of lymph nodes
c. Is a primary disease
d. Is a complication of another infectious process
e. Requires surgical drainage

263. CM. Specify the paraclinical methods used for the diagnosis of acute surgical infection of soft tissues:
a. General blood analysis
b. Urinalysis
c. Ultrasound scan of soft tissues
d. Fine needle aspiration
e. Biopsy

264. CM. Specify the areas of the body, on which furuncles arenot formed.
a. Nasolabial triangle
b. Back of neck
c. Palmar surface of hand
d. Anterior abdominal wall
e. Arch of foot

265. CS. In case of furuncle purulent inflammation develops in:


a. Subcutaneous adipose tissue
b. Sebaceous gland
c. Skin
d. Hair follicle
e. Sweat gland

266. CM. Specify the phases of furuncle evolution.


a. Phase of infiltration
b. Phase of necrosis
c. Phase of abscess formation
d. Phase of blow
e. Phase of rejection of the necrotic tissue (core)

267. CS. Phase of healing and scarring in the evolution of furuncle occurs after:
a. Spontaneous discharge of pus
b. Administration of antibiotics
c. Appearance of softening and fluctuation
d. Spontaneous discharge of necrotic core
e. Application of dressing with magnesium sulphate

268. CM. In which cases hospitalization of patients with furuncle is required?


a. In patients with furuncle of hip in a child aged 7
b. In patients with furuncle in phase of abscess formation in a patient with severe diabetes
c. In patients with the localization of furuncle on the foot and inability to support the limb
d. In patients with the localization of furuncle in the nasolabial triangle
e. In patients with history of furuncle disease

269. CS. Which of the following symptoms is indicates reliably, that the furuncle in the phase of abscess
formation?
a. Symptom of fluctuation
b. Hyperthermia of skin
c. Hyperemia of skin
d. Pain in the focus of inflammation
e. Appearance in center of the inflammatory infiltrate of necrotic core

270. CS. Cavernous sinus thrombosis is a rare but very serious (and often fatal) complication of a furuncle in its
location on:
a. On the lateral aspect of the nose or infra-orbital area
b. In the temporal or parietal area
c. At the nape of the neck or on the back
d. In the groin region or perineal area
e. In the axillary or subclavian region

271. CM. Conservative treatment of the patient with furuncle, complicated by cavernous sinus thrombosis, have to
include:
a. Antibiotics
b. Interferon
c. Anticoagulants
d. Insulin therapy
e. Diuretics

272. CS. Purulent inflammation of sweat glands is called:


a. Carbuncle
b. Hydrosalpinx
c. Furuncle
d. Hydradenitis
e. Axillary streptococcal dermatitis

273. CS. In case of hydradenitis a purulent inflammation develops in:


a. Subcutaneous adipose tissue
b. Sweat glands
c. Hair follicle
d. Lymph node
e. Sebaceous gland

274. CM. The typical localization of hydradenitis is:


a. Anterior abdominal wall
b. Skin fold of the breast
c. Axilla
d. Popliteal fossa
e. Inguinal skin fold

275. CM. Hydradenitis is characterized by the following features:


a. In the center of inflammatory infiltrate a hair follicle is located
b. The typical localization is the occipital region and the back of the neck
c. It is located commonly in the axillary area
d. Purulent inflammation begins in the sebaceous gland
e. Tendency to recurrence

276. CM. Which of the statements relating to abscess are correct?


a. The body temperature in this disease usually remains normal
b. The cavity of abscess is formed in the center of the inflammatory infiltrate
c. Surgical opening and drainage of the abscess do not improve the condition of patient
d. Clinical examination reveals the symptom of fluctuation
e. An increased temperature, usually is accompanied by fever

277. CM. Which of the statements relating to phlegmon are correct?


a. Suppurative process spreads along fascial and adipose spaces
b. An accumulation of pus is limited by pyogenic capsule
c. Symptoms of intoxication are usually not marked
d. The condition of patients remains satisfactory
e. Patients require hospitalization and emergency surgery under general or regional anesthesia

278. CS. What differentiates the abscess from the phlegmon?


a. Need in urgent surgery
b. Severe pain in the lesion area
c. Leukocytosis
d. Collection of pus is limited from surrounded tissues
e. High fever

279. CM. What is included in the surgical treatment of purulent focus?


a. A wide opening of purulent cavity
b. Introduction of broad-spectrum antibiotics in the purulent cavity
c. Removal of necrotic tissues
d. Placement of primary sutures
e. Drainage of the purulent cavity

280. CS. What is the causative agent of furuncle and carbuncle?


a. Pseudomonas aeruginosa
b. Streptococcus
c. Staphylococcus aureus
d. Escherichia coli
e. Peptostreptococcus

281. CM. Carbuncle is characterized by the following features:


a. Usually it develops in the occipital and interscapular areas
b. Does not develop in diabetic patients
c. The skin over the focus of inflammation is not altered
d. An abscess has the form of “honeycomb”
e. The symptom of fluctuation is not typical
282. CM. Specify the rules of surgery for carbuncle.
a. Is opened by a single cruciate incision
b. Thorough excision of necrotic tissues
c. Each of the multiple “honeycomb” abscesses is opened by a separate incision
d. Wounds usually leaving open to heal by secondary intention
e. Upon completion of surgery, the wound is sutured for healing by primary intention

283. CM. In cases of suspected abscess of soft tissues the direct indications for surgery are:
a. Severe pain in the focus of inflammation
b. Significant skin hyperemia and edema
c. Positive symptom of fluctuations
d. Increased erythrocyte sedimentation rate in general analysis of blood
e. Obtaining pus during the fine needle aspiration of the inflammatory focus

284. CS. Common causative microbial agent of erysipelas is:


a) Streptococcus pyogenes
b) Staphylococcus aureus
c) Escherichia coli
d) Pneumococcus
e) Pseudomonas aeruginosa

285. CM. Specify the clinical forms of erysipelas.


a. Phlegmonous form
b. Abscess formation form
c. Bullous form
d. Necrotic form
e. Erythematous form

286. CM. Specify the clinical symptoms characteristic of the erythematous form of erysipelas.
a. Formation of blisters on the skin filled with serous fluid
b. Hyperemia of the skin in the form of “tongues of flame”
c. Formation of areas of skin necrosis
d. Hyperemia of the skin in the form of “geographical map”
e. Positive symptom of fluctuations

287. CS. In what area of the body the erysipelas is most common?
a. On the arms
b. On the abdomen
c. On the nape of the neck and on the back
d. Within the intergluteal cleft
e. On the legs

288. CM. What forms of erysipelas require surgical treatment?


a. Phlegmonous form
b. Bullous form
c. Necrotic form
d. Abscess formation form
e. Erythematous form

289. CM. Specify the components of a complex treatment of surgical infection?


a. Rational antibiotic therapy
b. Disintoxication therapy
c. Transfusion of whole blood to stimulate the immune system
d. Surgical debridement of purulent focus
e. Administration of cytostatics

290. CM. What characterizes erysipeloid?


a. Commonly affects the skin of the hands
b. Usually develops against a background of venous or lymphatic insufficiency
c. Disease of people frequent handling fish and meat
d. Infectious process develops as a cellulitis
e. On the skin appear blisters with serous exudate, which may merge

291. CS. Puerperal mastitis develops:


a. In pubertal period
b. In the third trimester of pregnancy
c. In postpartum period
d. In postmenopausal period
e. In the first trimester of pregnancy

292. CM. The following forms of breast abscess are distinguished depending on the location:
a. Intramammary abscess
b. Epifascial abscess
c. Subareolar abscess
d. Subpectoral abscess
e. Retromammary abscess

293. CM. Specify the typical symptoms of acute mastitis.


a. Increased body temperature
b. Nipple retraction on the side of inflammation
c. Pain and bloating in the mammary gland
d. Skin in the form of “orange peel”
e. Serous and hemorrhagic discharge from the nipple

294. CM. Which of the following statements are correct?


a. Mastitis is a multiple times recurrent disease
b. In case of mastitis bacteria penetrates through the fissures of nipples
c. Galactostasia plays an important role in the development of mastitis
d. A typical causative agent of mastitis is streptococcus
e. All forms of mastitis need urgent surgical treatment

295. CS. Paraproctitis is:


a. Acute inflammation of the rectal mucosa
b. Thrombosis of hemorrhoidal node
c. Bleeding from the rectum
d. Bacterial infection of skin in the perianal region
e. Purulent inflammation of the perirectal adipose tissue

296. CM. Specify the forms of paraproctitis.


a. Bullous
b. Subcutaneous
c. Phlegmonous
d. Submucous
e. Pelviorectal

297. CS. To development of pilonidal abscess contributes the accumulation in the cavity of pilonidal cyst of:
a. Lymphoid tissue
b. Feces
c. Fungal microflora
d. Hairs and skin debris
e. Atheromatous masses

298. CM. What characterizes the pilonidal abscess?


a. Disease occurs as a result of the congenital defect
b. Commonly recur
c. Is often multilocular
d. Most common in young men
e. Complete recovery comes after spontaneous discharge of pus on the skin
299. CS. Definitive surgery for pilonidal abscess consists in:
a. Opening of the abscess by two separate incisions at the top and bottom of purulent collection
b. Elliptical wedge excision of the affected skin and subcutaneous tissue
c. Opening of the abscess by cruciate incision, with excision of necrotized tissues
d. Opening of the abscess by longitudinal incision along the intergluteal cleft
e. Elliptical wedge excision of the affected skin and subcutaneous tissue, and removal of the coccyx

300. CM. What characterizes the ingrowing toenail?


a. Congenital disease
b. Nail acts as a “foreign body”, which does not allow to heal inflammation
c. Affects the great toe
d. Affects fingers of hand
e. Is explained by cutting the nails too short at the corners

301. CM. For arterial bleeding is characteristic:


a) Bright red blood
b) Pulsatile jet of blood flow
c) Rapid blood loss with development of acute anemia
d) Non-pulsatile flow of blood
e) Dark-color blood

302. CM. For venous bleeding is characteristic:


a) Bright red blood
b) Pulsatile jet of blood flow
c) Diffuse uniform oozing of blood from whole wound surface
d) Non-pulsatile flow of blood
e) Dark-color blood

303. CM. For capillary bleeding is characteristic:


a) Bright red blood
b) Pulsatile jet of blood flow
c) Diffuse uniform oozing of blood from whole wound surface
d) Non-pulsatile flow of blood
e) Has a mixed arterio-venous character

304. CS. According to anatomical classification, parenchymatous hemorrhage refers to:


a) Arterial
b) Venous
c) Capillary
d) Erosive
e) External

305. CM. Specify the possible mechanisms of bleeding occurrence.


a) Mechanical injury of blood vessel
b) Destruction of vascular wall by pathological process
c) Increased permeability of vascular wall
d) Increase of systemic blood pressure
e) Destruction of vascular wall during thermal exposure

306. CS. On what basis bleeding are classified into “per diabrosin”, “per rhexin” и “per diapedesin”?
a) According to anatomical principles
a) According to mechanism of occurrence
b) According to severity of blood loss
c) According to evolution (intensity)
d) According to time of development

307. CS. The term “haemorrhagia per rhexin” means:


a) Bleeding due to increased permeability of vascular wall
b) Bleeding due to injury of blood vessel
c) Bleeding due to erosion of vessel
d) Bleeding due to decreased blood coagulation
e) Bleeding due increased systemic blood pressure

308. CS. The term “haemorrhagia per diabrosin” means:


a) Bleeding due to slippage of hemostatic ligature
b) Bleeding due to increased permeability of vascular wall
c) Bleeding due to erosion of vessel
d) Bleeding due to decreased blood coagulation
e) Bleeding due to injury of blood vessel

309. CS. The term “haemorrhagia per diapedesin” means:


a) Bleeding due to erosion of vessel
b) Bleeding due to increased permeability of vascular wall
c) Bleeding due to injury of blood vessel
d) Bleeding due to decreased blood coagulation
e) Bleeding due to increased blood coagulation

310. CM. The direct causes of “haemorrhagia per diabrosin” may serve:
a) Purulent inflammation
b) Malignancy
c) Ulcerative destruction
d) Hemorrhagic vasculitis
e) Avitaminosis

311. CM. The direct causes of “haemorrhagia per diapedesin” may serve:
a) Purulent inflammation
b) Malignancy
c) Sepsis
d) Hemorrhagic vasculitis
e) Avitaminosis

312. CM. The internal bleeding include:


a) Bleeding from gastric ulcer
b) Intramuscular hematoma
b) Bleeding from cutting shin wound
c) Pulmonary bleeding
d) Bleeding into the joint cavity

313. CM. To the internal intracavitary bleeding refers:


a) Bleeding from gastric ulcer
b) Bleeding into the bronchus
c) Intramuscular hematoma
d) Hemothorax
e) Bleeding into the joint cavity

314. CM. Which of these bleeding refers to the internal intracavitary?


a) Bleeding into the abdominal cavity in ruptured ectopic pregnancy
b) Bleeding into the pleural cavity
c) Uterine bleeding
d) Bleeding into the urinary bladder
e) Bleeding into the pericardial sac with cardiac tamponade

315. CS. The accumulation of blood in the abdominal cavity is called:


a) Hemothorax
b) Hemopericardium
c) Hemoperitoneum
d) Hemarthrosis
e) Hemobilia

316. CM. To the internal intraluminal bleeding refers:


a) Bleeding into the pleural cavity
b) Bleeding into the pericardial sac with cardiac tamponade
c) Bleeding into the bronchus
d) Intracerebral hematoma
e) Bleeding from duodenal ulcer

317. CM. To the manifestations of internal intratissular bleeding refers:


a) Petechia
b) Purpura
c) Ecchymosis
d) Hematoma
e) Hemarthrosis

318. CS. Purpura, which occurs in the internal intratissular bleeding, is called:
a) Small (with diameter 1-3 mm) red or purple spot on the skin, caused by a minor bleed from broken capillary
blood vessels
b) Red or purple spots on the skin (with diameter 3-10 mm) that do not blanch on applying pressure
c) The subcutaneous or submucous spot of extravasated blood, with diameter larger than 1 cm
d) Limited accumulation of liquid or coagulated extravasated blood in the depth of soft tissue
e) Accumulation of liquid or coagulated blood, outside the blood vessels, into the pericardial sac

319. CS. Ecchymosis, which occurs in the internal intratissular bleeding, is called:
a) Small (with diameter 1-3 mm) red or purple spot on the skin, caused by a minor bleed from broken capillary
blood vessels
b) Red or purple spots on the skin (with diameter 3-10 mm) that do not blanch on applying pressure
c) The subcutaneous or submucous spot of extravasated blood, with diameter larger than 1 cm
d) Limited accumulation of liquid or coagulated extravasated blood in the depth of soft tissue
e) Accumulation of liquid or coagulated blood, outside the blood vessels, into the pericardial sac

320. CS. Hematoma, which occurs in the internal intratissular bleeding, is called:
a) Small (with diameter 1-3 mm) red or purple spot on the skin, caused by a minor bleed from broken capillary
blood vessels
b) Red or purple spots on the skin (with diameter 3-10 mm) that do not blanch on applying pressure
c) The subcutaneous or submucous spot of extravasated blood, with diameter larger than 1 cm
d) Limited accumulation of liquid or coagulated extravasated blood in the depth of soft tissue
e) Accumulation of liquid or coagulated blood, outside the blood vessels, into the pericardial sac

321. CS. According to classification of bleeding by time of their development, the primary bleeding occurs:
c) In the first hours after injury of vessel
a) Immediately after the injury of the vessel
b) After the development of purulent wound infection
c) During transportation of patient into the hospital as a result of sliding of knot from the vessel
d) During transportation of patient into the hospital as a result of expulsion of thrombus from injured vessel

322. CS. According to classification of bleeding by time of their development, early secondary bleeding occurs:
a) In the first hours or days after trauma, as a result of sliding of knot from the vessel
b) At the time of injury
c) After development of the purulent wound infection
d) After decreasing of blood coagulation
e) With the progression of inflammatory changes in the wound

323. CM. The causes of early secondary bleeding are:


a) Expulsion of thrombus from the vascular lumen
b) Injury of vessel
c) Sliding of knot from the vessel
d) Inflammatory process
e) Increased permeability of vascular wall

324. CS. According to classification of bleeding by time of their development, late secondary bleeding occurs:
a) In the first days after injury of vessel
b) Immediately after injury of vessel
c) After development of the purulent wound infection
d) During transportation of patient into the hospital as a result of sliding of knot from the vessel
e) In the first days after injury, due to increased permeability of the vessel wall

325. CM. The causes of late secondary bleeding are:


a) Spontaneous rupture of vessel
b) Inflammatory process in the wound
c) Sliding of knot from the vessel
d) Increased permeability of the vascular wall
e) Arrosion of the vessel wall

326. CS. According to classification of hemorrhage on their evolution (intensity), bleeding are divided into:
a) Acute and chronic
b) Primary and secondary
c) Severe, moderate and mild
d) Arterial, venous and capillary
e) External and internal

327. CS. Acute blood loss of 750-1500 mL in an adult men, corresponds to circulatory blood volume deficits:
a) Up to 10%
b) 10-15%
c) 15-30%
d) 30-40%
e) More than 40%

328. CS. In normal physiological conditions venous system contain:


a) 5-7.5% of circulatory blood volume
b) 15-20% of circulatory blood volume
c) 30-50% of circulatory blood volume
d) 55-60% of circulatory blood volume
e) 70-75% of circulatory blood volume

329. CM. Specify the life-threatening consequences of bleeding.


a) Enhancing of blood oxygenation
b) Hemodilution
c) Decreasing of circulatory blood volume
d) Hypoxia of vital organs
e) Release of the adrenal cortex hormones

330. CM. In case of bleeding, decrease of blood pressure and stimulation the baroreceptors of carotid sinus and
aortic arch lead to:
a) The increase of venous tone
b) The increase of cardiac output volume
c) The appearance of tachycardia
d) The centralization of circulation
e) The blood sequestration

331. CM. Specify the compensatory reaction of human organism in the initial phase of hemorrhage.
a) Increased venous tone
b) Release of red blood cells from the “depot”
c) Tachycardia
d) Blood sequestration
e) Hyperventilation

332. CM. Physiological mechanisms of compensation in case of bleeding are following:


a) Hemodilution
b) Centralization of circulation
c) Decentralization of circulation
d) Brain ischemia and edema
e) Including of renal mechanism of compensation

333. CM. The centralization of circulation in case of bleeding is characterized by:


a) Vasoconstriction of arterioles in the skin, lungs and digestive tract
b) The biggest part of the arterial blood goes through arterio-venous shunts, bypassing the capillary network
c) Circulation in the coronary and cerebral vessels does not suffer
d) Increased rate and volume of respiratory movements
e) Retention of water in organism by reducing of urine output

334. CM. The mechanism of compensatory hyperventilation (increased rate and volume of respiratory
movements) in case of bleeding is caused by:
a) Decrease of mean blood pressure to 60 mm Hg
b) Decrease in blood рН (acidosis development)
c) Decrease of mean blood pressure to 100 mm Hg
d) Increase in blood рН (alkalosis development)
e) Decrease of hydrostatic pressure at the capillary level

335. CM. Compensatory hemodilution in case of bleeding contributes to:


a) Compensation of hypovolemia
d) Spontaneous hemostasis
b) Improvement the rheological properties of blood
c) Water and sodium reabsorption
d) Changes of metabolism character from aerobic to anaerobic

336. CM. Specify the pathological phenomena of decompensation, which develop in the human organism in
hemorrhage.
a) Tissue acidosis
b) Centralization of circulation
c) Multiple organ dysfunction syndrome
d) Decentralization of circulation
e) Hemodilution

337. CM. Complaints of patients with chronic bleeding include:


a) Moderate weakness
b) Rapid fatigue
c) Loss of conciseness
d) Severe dyspnea
e) Oligoanuria

338. CM. Specify the general symptoms of hemorrhage.


a) Tachycardia
e) Melena
b) Pallor of skin
c) Blood vomit
d) Hypotension

339. CM. Specify the local symptoms of hemorrhage.


a) Melena
f) Dizziness
b) Blood vomit
c) Hemoptysis
g) Accelerated breathing (tachypnea)

340. CS. What is hemoptysis?


a) Elimination of blood with stool
b) Vomiting with blood clots
c) Elimination of blood with sputum from respiratory tract
d) Elimination of blood from the nostrils
e) Elimination of blood with urine

341. CS. Which of these symptoms are characteristic for epistaxis?


a) Elimination of blood from the rectum
b) Elimination of blood from the urinary tract
c) Elimination of blood from the ears
d) Elimination of blood from the nose
e) Elimination of blood from the uterus

342. CM. Specify the local symptoms of esophageal or gastric bleeding.


a) Hematemesis
b) Epistaxis
c) Hematuria
d) Hemoptysis
e) Melena

343. CS. For what type of hemorrhage is characteristic melena?


a) Gastrointestinal
b) Uterine
c) Renal
d) Pulmonary
e) Intraperitoneal

344. CS. A “coffee-ground” vomiting may be a manifestation of:


a) Hemorrhoidal bleeding
b) Urinary bladder tumor
c) Gastroduodenal ulcer bleeding
d) Traumatic rupture of spleen
e) Bacterial destruction of lung

345. CM. Which of these symptoms indicate hemorrhage from the large bowel?
a) Bloody stool (hematochezia)
b) Vomiting with blood
c) Black tarry stool
d) Hematuria
e) “Coffee-ground” vomiting

346. CS. What is metrorrhagia?


a) Elimination of blood with stool
b) Elimination of blood through vagina
c) Elimination of blood from the respiratory tract
d) Elimination of blood through nostrils
e) Elimination of blood with urine

347. CM. Specify the possible causes of hemoperitoneum.


a) Injury of liver
b) Hemorrhage from a urinary bladder tumor
c) Ruptured aneurysm of abdominal aorta
d) Bleeding from esophageal varices in liver cirrhosis
e) Ruptured ectopic pregnancy
348. CS. In case of hemothorax is observed:
a) Dyspnea, tachypnea, dullness of percussion sound, decreased respiration
b) Tachycardia, skin cyanosis, distension of neck veins, muffled heart tones
c) Abdominal pain and distention, peritoneal irritation
d) Enlargement of the joint, acute pain, forced position of extremity
e) Bradycardia, face asymmetry, anisocoria

349. CS. In case of hemoperitoneum is observed:


a) Dyspnea, tachypnea, dullness of percussion sound, decreased respiration
b) Tachycardia, skin cyanosis, distension of neck veins, muffled heart tones
c) Abdominal pain and distention, peritoneal irritation
d) Enlargement of the joint, acute pain, forced position of extremity
e) Bradycardia, face asymmetry, anisocoria

350. CS. In case of hemarthrosis is observed:


a) Dyspnea, tachypnea, dullness of percussion sound, decreased respiration
b) Tachycardia, skin cyanosis, distension of neck veins, muffled heart tones
c) Abdominal pain and distention, peritoneal irritation
d) Enlargement of the joint, acute pain, forced position of extremity
e) Bradycardia, face asymmetry, anisocoria

351. CM. The severity of blood loss is reflected by following laboratory parameters:
a) Red blood cells
b) Blood leukocytes
c) Hemoglobin of blood
d) Hematocrit of blood
e) Platelets of blood

352. CS. Specify the normal values of hematocrit.


a) 10-15%
b) 20-25%
c) 30-35%
d) 40-45%
e) 50-55%

353. CM. The following parameters are require for determination of Allgower shock index:
a) Pulse rate per minute
b) The value of systolic blood pressure
c) The value of mean blood pressure
d) Estimated circulatory blood volume
e) The level of red blood cells in peripheral blood

354. CS. The Allgower shock index is calculated by:


a) Dividing of pulse rate on the value of systolic blood pressure
b) Dividing of pulse rate on the value of diastolic blood pressure
c) Dividing of pulse rate on the value of mean blood pressure
d) Multiplication of pulse rate on the value of systolic blood pressure
e) Multiplication of pulse rate on the value of mean blood pressure

355. CM. To estimate the volume of intraoperative blood loss according to Gross’s formula are used:
a) Height of the patient
b) Normal value of hematocrit
c) Weight of the patient
d) Value of hematocrit after surgery
e) Coefficient of blood volume per one kilogram of body mass

356. CS. Formulas of Moore or Nadler are used for:


a) Determining the area of a burn surface
b) Determining the severity of blood loss
c) Determining the nutritional status of surgical patient
d) Determining the estimated circulatory blood volume
e) Determining the grade of perioperative risk

357. CM. What diagnostic methods are helpful for confirmation of hemothorax?
a) Bronchoscopy
b) Chest radiograph
c) Thoracentesis
d) Paracentesis
e) Electrocardiography (ECG)

358. CS. What diagnostic method is indicated to confirm the diagnosis of gastric bleeding?
a) Paracentesis
b) Fibrogastroduodenoscopy
c) Colonoscopy
d) Bronchoscopy
e) Ultrasound scan

359. CM. What diagnostic methods may confirm the hemorrhage into the joint?
a) Arthroscopy
b) Laparoscopy
c) Paracentesis
d) Fine needle aspiration (puncture) of articular cavity
e) Radiograph of joint

360. CM. What diagnostic methods may confirm the intraperitoneal hemorrhage?
a) Bronchoscopy
b) Fibrogastroduodenoscopy
c) Fine needle aspiration (puncture) of posterior vaginal fornix
d) Plane abdominal radiography
e) Laparoscopy

361. CM. The process of blood coagulation includes three main phases:
a) Adhesion and aggregation of erythrocytes
b) Lysis and degradation of the fibrin clot
c) Vasoconstriction (spasm of injured vessel)
d) Activation of plasmatic coagulation cascade
e) Adhesion and aggregation of platelets

362. CM. Platelet clot formation occurs through the action of the following factors:
a) Tissue factor (thromboplastin)
b) Spasm of injured vessel
c) Blood platelets
d) Serum prothrombin
e) Serum fibrinogen

363. CS. The intrinsic pathway of plasmatic coagulation is initiated by activation of:
a) Hageman factor
b) Tissue factor
c) Stuart-Prower factor
d) Fibrin-stabilizing factor
e) Prothrombin

364. CS. The extrinsic pathway of plasmatic coagulation is initiated by activation of:
a) Hageman factor
b) Tissue factor
c) Stuart-Prower factor
d) Fibrin-stabilizing factor
e) Prothrombin

365. CS. What factor of coagulation is activated at the end of both intrinsic and extrinsic pathways of plasmatic
coagulation?
a) Hageman factor
b) Tissue factor
c) Stuart-Prower factor
d) Fibrin-stabilizing factor
e) Prothrombin

366. CM. Which of the following events refer to a common pathway of plasmatic coagulation?
a) Conversion of plasminogen to plasmin
b) Stabilization of fibrin clot
c) Conversion of prothrombin to thrombin
d) Transformation of fibrinogen to fibrin
e) Adhesion and aggregation of platelets

367. CM. Specify the physiological mechanisms for restricting of local coagulation and prevention of generalized
uncontrollable intravascular coagulation.
a) Vascular endothelium has an barrier isolating role
b) Clotting factors are inactive in circulation
c) Existence of physiological anticoagulants
d) Enlargement of vessels (vasodilatation)
e) Effect of fibrin destabilizing factor

368. CS. The disseminated intravascular coagulation syndrome manifests by the following phenomenon:
a) Inflammatory phenomenon
b) Regenerative phenomenon
c) Proliferative phenomenon
d) Hemorrhagic phenomenon
e) Degenerative phenomenon

369. CM. The etiology of disseminated intravascular coagulation syndrome includes:


a) Massive blood transfusion
b) Severe bacterial infections
c) Prolonged intake of anticoagulants
d) Obstetrical complications: placental abruption, intrauterine fetal death
e) Severe diabetes mellitus

370. CS. In pathogenesis of disseminated intravascular coagulation syndrome, the main cause of blood
incoagulability is:
a) Massive consumption of clotting factors and depletion of their reserve
b) Activation of coagulation cascade by endogenous factors
c) Systemic damage of vascular endothelium
d) Direct activation of coagulation system by microbial enzymes
e) Blockage of microcirculation in the vital organs

371. CM. Specify clinical forms of disseminated intravascular coagulation syndrome.


a) Fulminant form
b) Acute form
c) Subacute form
d) Chronic form
e) Asymptomatic form

372. CM. Specify the phases of disseminated intravascular coagulation syndrome.


a) Phase of hypovolemia
b) Intravascular phase
c) Phase of hypercoagulation
d) Phase of hypocoagulation
e) Phase of dissemination

373. CS. Specify the clinical symptoms of the first phase of disseminated intravascular coagulation syndrome.
a) Multiple bleeding from different sources
b) Pronounced jaundice
c) Multiple vascular thrombosis
d) Bleeding into the vital organs
e) Hepatomegaly and splenomegaly

374. CS. Specify the clinical symptom of the second phase of disseminated intravascular coagulation syndrome.
a) Multiple bleeding, at least from three different sources
b) Pronounced jaundice
c) Multiple vascular thrombosis
d) It is difficult to establish symptoms clinically or their are absent
e) Hepatomegaly and splenomegaly

375. CM. What findings in laboratory parameters confirm the disseminated intravascular coagulation syndrome?
a) Severe thrombocytopenia
b) Decreased blood fibrinogen
c) Severe thrombocytosis
d) Increased level of D-dimers
e) Increased blood fibrinogen

376. CM. Specify three main components of complex treatment for disseminated intravascular coagulation
syndrome.
a) Treatment of the diseases, which lead to the development of disseminated intravascular coagulation
b) Massive antibiotic therapy
c) Immediate compensation of clotting factors
d) Administration of heparin
e) Administration of immune serums

377. CM. What medications are indicated for the treatment of patients with disseminated intravascular coagulation
syndrome?
a) Administration of heparin
b) Intravenous administration of vasopressors (adrenalin)
c) Transfusion of fresh frozen plasma
d) Transfusion of thrombin
e) Thrombolytic therapy (urokinase, streptokinase)

378. CM. To the methods of temporary hemostasis refer:


a) Application of compressive bandage
b) Elevated position of extremity
c) Maximal flexion of extremity
d) Ligation of bleeding vessel in the wound
e) Surgical repair of the vessel

379. CM. Specify the methods of temporary hemostasis.


a) Replacement of the injured segment of the vessel
b) Digital compression of bleeding vessel in the wound
c) Application of hemostatic tourniquet
d) Application of compressive bandage
e) Maximal flexion of extremity

380. CM. What methods of hemostasis refer to temporary?


a) Embolization of injured vessel
b) Application of hemostatic forceps to the bleeding vessel
c) Ligation of bleeding vessel in the wound
d) Surgical repair of the vessel
e) Application of compressive bandage
381. CS. What method of temporary hemostasis is the optimal during urgent surgery for ruptured abdominal aortic
aneurysm?
a) Application of hemostatic tourniquet
b) Suturing of aortic defect
c) Intravenous administration of hemostatic drugs
d) Clamping of aorta with forceps above the aneurysm
e) Clamping of aorta with forceps below the aneurysm

382. CM. Specify the principles of correct application of hemostatic tourniquet.


a) Tourniquet is applied only in case of severe arterial bleeding
b) In correct application of tourniquet the pulse on peripheral artery will disappear
c) The exact time of tourniquet application should be indicated on the special tag
d) Tourniquet is applied directly to the skin to enhance compression
e) The extremity distal to correctly applied tourniquet becomes red

383. CS. Where should be applied a hemostatic tourniquet in case of external arterial bleeding from the wound of
a shin?
a) In the lower third of shin
b) In the middle third of shin
c) In the upper third of shin
d) In the middle third of femur
e) On the level of knee joint

384. CS. Where should be applied a hemostatic tourniquet in case of external arterial bleeding from the wound of
a hand?
a) In the distal third of forearm
b) In the middle third of forearm
c) In the proximal third of forearm
d) In the middle third of shoulder
e) On the level of elbow joint

385. CS. To stop bleeding temporary, the hemostatic tourniquet on a limb should be applied for a period no more
than:
a) One hour
b) One and a half hours
c) Three hours
d) Four hours
e) 45 minutes

386. CM. Specify the possible complications of hemostatic tourniquet application on a limb for more than 1.5
hours.
a) Development of paresis (plegia) of limb
b) Increased vascular tone in the limb
c) Limb gangrene
d) Development of early secondary bleeding
e) Gas embolism of affected extremity

387. CM. To the methods of definitive hemostasis refer:


a) Application of hemostatic forceps on the bleeding vessel
b) Prosthetics of the injured vessel
c) Surgical repair of the vessel
d) Application of hemostatic tourniquet
e) Endovascular embolization of the vessel

388. CM. Specify the methods of definitive hemostasis.


a) Application of hemostatic tourniquet
b) Elevated position of extremity
c) Application of hemostatic sponge on basis of collagen
d) Endoscopic clipping of bleeding vessel
e) Digital compression of bleeding vessel in the wound

389. CM. What methods of hemostasis refer to definitive?


a) Application of hemostatic tourniquet
b) Digital compression of bleeding vessel
c) Removing the injured organ
d) Argon-plasma coagulation
e) Ligation of vessel in the wound

390. CS. Which of these types of bleeding require ligation of vessel at the distance from the wound to achieve
definitive hemostasis?
a) Arterial bleeding from fresh cutting hip wound
b) Arterial bleeding from purulent wound
c) Bleeding from duodenal ulcer
d) Bleeding in patient with hemophilia
e) Bleeding hemorrhoids

391. CM. The methods of definitive hemostasis are classified depending on their nature to:
a) Mechanical
b) Physical
c) Chemical
d) Biological
e) Artificial

392. CS. To the methods of definitive mechanical hemostasis refer:


a) Application of fibrin glue
b) Cryosurgical exposure of liquid nitrogen
c) Suturing of bleeding vessel
d) Monopolar diathermocoagulation of vessel
e) Tissue sclerosis with ethanol

393. CM. What methods of definitive hemostasis are indicated for injury of a common femoral artery?
a) Application of hemostatic tourniquet
b) Application of compressive bandage
c) Vascular anastomosis
d) Ligation of iliac artery
e) Implantation of vascular prostheses

394. CM. Specify the methods of definitive physical hemostasis.


a) Ligation of vessel
b) Ligation with rubber bands for hemorrhagic esophageal varices
c) Application of compressive bandage
d) Ultrasound scalpel
e) Diathermocoagulation of vessel

395. CM. Specify the methods of definitive chemical hemostasis.


a) Topical administration of adrenaline solution
b) Radiofrequency coagulation
c) Application of hemostatic sponge
d) Local application of thrombin
e) Endoscopic injection of ethanol

396. CM. Which of the following drugs are used for definitive chemical hemostasis?
a) Collagen
b) Fibrinogen
c) Polidocanol
d) Liquid nitrogen
e) Cyanoacrylate
397. CS. The mechanism of hemostatic action of cyanoacrylate the following:
a) Causes the protein coagulation and blood clotting
b) Causes the sclerosis and dehydration of tissues in a wound
c) In the application produces heat and results in denaturation of proteins
d) Causes polymerization, which converts a substance from a liquid to solid substance
e) In the contact with liquid blood causes the local blood clotting

398. CM. Specify the methods of definitive biological hemostasis.


a) Intravenous administration of calcium chloride solution
b) Endovascular stent-grafting of vessel
c) Application of fibrin glue
d) Bipolar diathermocoagulation
e) Local application of collagen hemostatic sponge

399. CM. Hemostatic sponges for local application contain:


a) Adrenalin
b) Vikasol
c) Fibrinogen and thrombin
d) Collagen
e) Etamsylate

400. CM. Specify the main components of fibrin glue Tissucol and hemostatic sponge Tachocomb, used for
definitive biological hemostasis.
a) Fibrinogen
b) Thrombin
c) Adrenalin
d) Cellulose
e) Aminocaproic acid

401. CM. Which of these factors cause directly tissue necrosis?


a) Mechanical lesion
b) Arterial embolism
c) Burning injury
d) Compression of arterial vessel
e) Electrical injury

402. CM. Which of the following factors causes tissue necrosis indirectly?
a) Mechanical lesion
b) Arterial embolism
c) Burning injury
d) Strangulated hernia
e) Electrical injury

403. CM. What are characteristic feauters of gangrene?


a) It develops only in organs, that have contact with the external environment
b) It develops only in organs, that have no contact with the external environment
c) The organ or anatomical region is affected entirely
d) It develops after prolonged compression of soft tissue between the hard surfaces (bones and bed)
e) It affects only a part of the organ, which receives blood supply from a certain arterial branch

404. CM. Which of the following factors does not influence the development of “circulatory” necrosis?
a) Insufficient flow of arterial blood
b) Impairment of pulmonary gas exchange
c) Disruption of venous blood outflow
d) Impairment of transcapillary metabolism at the level of the microcirculation
e) Disturbances of innervation
405. CM. Specify the components of the Virchow's triad, which represent the pathogenetic mechanism of vascular
thrombosis.
a) Endothelial damage
b) Stasis of blood
c) Atrial fibrillation with the rupture of thrombus from the heart cavity
d) Hypercoagulation
e) Prolonged consumption of the indirect anticoagulants

406. CS. Specify the typical localization of trophic ulcers in the syndrome of chronic ischemia.
a) Middle third of the shin
b) External surface of the lower third of the shin
c) Distal phalanx of the digits of foot, heel area
d) Medial surface of the lower third of the shin
e) Popliteal fossa

407. CM. Dry gangrene is characterized by the following features:


a) It usually develops in acute ischemia
b) It usually develops in chronic ischemia
c) It develops after bacterial contamination
d) There is a line of demarcation
e) The demarcation line is absent

408. CM. Moist gangrene is characterized by the following features:


a) It usually develops in acute ischemia
b) It usually develops in chronic ischemia
c) It develops after bacterial contamination
d) There is a line of demarcation
e) The demarcation line is absent

409. CS. Which of the following is necrosis?


a) Infarction
b) Bedsores
c) Trophic ulcer
d) Gangrene
e) All of the above

410. CM. Specify the organs, in which gangrene can develop.


a) Spleen
b) Lung
c) Pancreas
d) Intestine
e) Lower limbs

411. CM. Specify the organs, in which the development of necrosis is called infarction.
a) Heart
b) Bones
c) Kidney
d) Lower limbs
e) Spleen

412. CM. Specify the common causes of trophic ulcer development.


a) Acute venous thrombosis
b) Chronic ischemia
c) Arterial aneurysm
d) Chronic venous insufficiency
e) Acute ischemia

413. CS. Which of these fistulas are congenital?


a) Purulent fistula in chronic osteomyelitis
b) Suture sinus
c) Gastrostomy
d) Urachal fistula
e) Colostomy

414. CM. Which of these fistulas are acquired?


a) Purulent fistula in chronic osteomyelitis
b) Suture sinus
c) Tracheostome
d) Urachal fistula
e) Colostomy

415. CM. Specify the external fistulas.


a) Perianal fistula
b) Cholecystoenterostomy
c) Colostomy
d) Gastrostomy
e) Gastroenterostomy

416. CM. The following types of fistulas are distinguished depending on the nature of tissue lining the fistulous
tract:
a) Epithelial
b) Muscular
c) Mucosal
d) Granulation
e) Ossified

417. CS. Surgically created artificial communication between two hollow organs is called:
a) Perforation
b) Ostomy
c) Anastomosis
d) Cyst
e) Penetration

418. CS. Artificially created communication between a hollow organ and the external environment is called:
a) Cyst
b) Ostomy
c) Perforation
d) Fistula
e) Anastomosis

419. CS. Pathological communication between two hollow organs is called:


a) Fistula
b) Anastomosis
c) Ostomy
d) Cyst
e) Perforation

420. CS. Pathological communication between a hollow organ and the external environment is called:
a) Perforation
b) Ostomy
c) Anastomosis
d) Fistula
e) Cyst

421. CM. Specify the common location of bedsores?


a) Elbows
b) Lumbar region
c) Sacral region
d) Heels
e) Scapular region

422. CM. Specify the phase of bedsore development?


a) Prereactive phase
b) Phase of circulatory disorders
c) Phase of necrotic lesions
d) Phase of regeneration
e) Reactive phase

423. CS. In all types of surgical pathology preoperative period starts:


a) With admission of the patient to the hospital and case history registration
b) With appearance of first signs of surgical disease
c) When a surgical disease was diagnosed and decision to operate was made
d) At the moment of transportation of patient to the operating room and preparation for anesthesia
e) When skin scrub in surgical site and limitation with sterile drapes are performed

424. CS. Preoperative period in all surgical pathologies is finalized at the moment when:
a) The decision to intervene was done by surgeon and informed consent was signed by patient
b) Anesthesia is initiated
c) Skin incision is performed
d) Patient is transported in the operating room
e) Surgical field is prepared and delimitated

425. CM. Which of the following refers to the main goals of preoperative period?
a) Prevention of venous thromboembolism
b) Patient blood group typing and preparation for blood transfusion
c) Reduction of surgical risk
d) High-dose antibacterial therapy
e) Increasing of curative effectiveness of surgery

426. CM. Preoperative period includes the following stages:


a) Stage of preparation
b) In-patient stage
c) Stage of recovery
d) Diagnostic stage
e) Out-patient stage

427. CS. Shtanghe’s and Ghence’s tests can be used in preoperative period for the assessment of:
a) Grade of compensation of cardio-respiratory function
b) Hepatic function
c) Grade of emergency of surgical intervention
d) State of nutrition of surgical patient
e) Risk of perioperative venous thromboembolism

428. CM. Specify the routine diagnostic tests done before any surgical intervention.
a) Full blood count
b) Tests for HIV-infection and hepatitis B, С, D
c) Urinalysis
d) Electrocardiography
e) Abdominal ultrasound

429. CM. During the physical examination of a patient, admitted to the surgical department for elective surgery,a
surgeon finds the heart bruits and abnormal cardiac rhythm. What actions should be performed in such a
case?
a) Immediate discharge of the patient from the hospital for complex cardiac evaluation in out-patient conditions
b) Immediate transfer of the patient to the department of cardiology for treatment
c) Realization of planned surgical intervention
d) Refer the patient to cardiologist (internal medicine physician) for consultation
e) Review of current ECG results and previous cardiac tests

430. CS. The most often system used for stratification of surgical risk is:
a) Glasgow score
b) POSSUM system
c) Allgower index
d) ASA system
e) Moore formula

431. CS. According to the ASA classification of surgical risk, a patient with life-threatening concomitant disorders
(comorbidities) supposed to emergency intervention should be classified as:
a) ASA III E
b) ASA IV С
c) ASA V С
d) ASA IV E
e) ASA V E

432. CS. Choose the situation that serves as an absolute indication for surgery.
a) Presence of a pathology with risk for patients life, which may be treated either medically or surgically with
similar results
b) Presence of a pathology with risk for patient’s life, which may be treated only surgically
c) Presence of malignant tumor
d) Presence of a pathology with no risk for the patient’s life and functional capacity, which may be treated only
surgically
e) Traumatic injuries

433. CM. Specify the diagnosis which serve as an absolute indications for surgery.
a) Ateroma of occipital region
b) Uncomplicated varicose veins of lower limb
c) Gastric cancer without metastases and surrounding tissue invasion
d) Acute appendicitis
e) Strangulated inguinal hernia

434. CM. Specify the diagnosis which serve as a relative indications for surgery.
a) Slash wound with injury of femoral artery
b) Lypoma of lumbar region
c) Duodenal ulcer with multiple exacerbations
d) Acute intestinal obstruction
e) Strangulated inguinal hernia

435. CS. What diseases or pathological conditions may be a contraindication for surgery in case of severe active
intraperitoneal bleeding?
a) Hypovolemic shock
b) Acute myocardial infarction
c) Septic shock
d) Acute cerebral circulatory impairment (stroke)
e) None of the mentioned above

436. CM. What information should be provided to a patient during the preparation for surgery?
a) Diagnosis
b) Technical aspects of elective surgery (type of anastomosis, type of suture)
c) Possibilities of nonsurgical treatment (if any exist)
d) Results of similar intervention in other patients, currently hospitalized in the department
e) Possible surgery related complications

437. CM. Informed consent for surgery should be signed by:


a) Patient
b) Anesthesiologist
c) Operating surgeon
d) Nurse
e) Director of Hospital

438. CM. Specify the situations when life-saving emergency surgery may be performed without informed consent
for surgery signed by patient.
a) A patient with normal mental state categorically refuses the surgery
b) A patient is unconscious
c) A patient with disturbed mental state, confirmed by psychiatrist
d) Relatives of patient agree with the proposed surgical intervention
e) A patient refuses the surgery due to religious convictions

439. CM. What factors do not increase the risk of postoperative venous thromboembolism?
a) Obesity
b) Early postoperative ambulation
c) Malignancy
d) Traumatic and prolonged surgical interventions
e) Young age

440. CM. Specify the correct statements regarding postoperative pulmonary artery embolism (PE).
a) More frequently the source of PE is the superior cava vein and its branches
b) Usually thrombus formation starts „in-situ” – directly in the pulmonary artery
c) PE may develop either in early and late postoperative period
d) More frequently PE develops in case of free-floating type of deep vein thrombosis
e) PE never results in sudden death of a patient

441. CM. Indicate measures used for prevention of postoperative venous thromboembolism.
a) Administration of direct anticoagulants (heparins) in the pre- and postoperative periods
b) Refuse of antibacterial treatment
c) Strict bed rest after surgery
d) Massive blood transfusion
e) Elastic compression of lower limbs

442. CS. Choose the correct regime of administration of antibiotics to prevent postoperative surgical site infection.
a) One broad-spectrum antibiotic, orally, 7 days before surgery
b) One narrow-spectrum antibiotic, intramuscular, 2-3 days after surgery
c) One broad-spectrum antibiotic, single intravenous injection of maximal therapeutic dose, immediately before
surgery
d) One broad-spectrum antibiotic, single intramuscular injection of minimal therapeutic dose, immediately before
surgery
e) Two or more broad-spectrum antibiotics, intravenous, 5-7 days before surgery

443. CM. Preparation of digestive tract for elective abdominal surgery includes:
a) Nothing by mouth (NPO - Nothing Per Orem) diet during several days before surgery
b) NPO diet during 12 hours before surgery
c) Rigorous gastric lavage through gastric tube
d) Cleaning enemas
e) Siphon enemas

444. CM. Choose the examples of special preoperative care.


a) Depilation of surgical site
b) Sedative medication
c) Skin marking of varicose veins
d) Placement of central venous access line
e) Preoperative wearing of hernia-bandage for huge ventral hernia

445. CS. Preoperative conclusion must be written by:


a) Anesthesiologist
b) Operating surgeon
c) Surgical nurse
d) Family physician
e) Any health-worker

446. CM. Preoperative conclusion must include the following data:


a) Argumentation of the main diagnosis and indications for surgery
b) Short description of past history (anamnesis vitae)
c) Allergy to food and medications
d) Planned volume of intervention and type of anesthesia
e) Grade of risk

447. CS. Trendelenburg’s position of a patient on the operating table is used for:
a) Surgery on the diaphragm
b) Liver and biliary surgery
c) ENT (ear-nose-throat) surgery
d) Pelvic (gynecological) surgery
e) Lower limbs arterial surgery

448. CM. “Anti-Trendelenburg’s position” of a patient on the operating table is used for:
a) Surgery on the diaphragm
b) Liver and biliary surgery
c) ENT (ear-nose-throat) surgery
d) Pelvic (gynecological) surgery
e) Lower limbs arterial surgery

449. CS. Lateral position of the patient on operating table is used for surgical interventions on:
a) Rectum
b) Liver and biliary tract
c) ENT (Ear-Nose-Throat) Surgery
d) Lungs
e) Arteries of lower limbs

450. CM. Choose the steps of surgical intervention.


a) Patient skin scrub and limitation with sterile drapes
b) Induction in anesthesia
c) Surgical access
d) Surgical procedure
e) Recovery from anesthesia

451. CS. Patient was diagnosed with acute appendicitis and decision to operate was done. During surgery the
correctness of diagnosis should be confirmed at stage of:
a) Access
b) Exploration
c) Procedure
d) Check and Closure
e) Appendix removal

452. CM. The last step of surgical intervention includes:


a) Check of hemostasis stability
b) Check for foreign bodies
c) Check of hemodynamic parameters
d) Drainages placement
e) Surgical wound closure

453. CM. Emergency surgery is performed in case of:


a) Uncomplicated inguinal hernia
b) Acute appendicitis
c) Heart tamponade
d) Chronic cholecystitis
e) Perforated peptic ulcer
454. CM. Elective surgery is performed in case of:
a) Mechanical asphyxia due to airway obstruction
b) Acute intestinal obstruction
c) Varicose veins of lower limbs
d) Strangulated umbilical hernia
e) Simple (uncomplicated) inguinal hernia

455. CS. In a patient with advanced unresectable cancer of esophagus, an external artificial fistula of the stomach
(gastrostomy) was created surgically with the aim of enteral feeding. How is this type of surgery classified?
a) Diagnostic surgery
b) Combined surgery
c) Radical surgery
d) Palliative surgery
e) Simultaneous surgery

456. CS. In a patient with inguinal hernia, resection of hernia sac, repair of inguinal channel and implantation of
synthetic mesh were performed. How this type of surgery is classified?
a) Diagnostic surgery
b) Combined surgery
c) Radical surgery
d) Palliative surgery
e) Simultaneous surgery

457. CM. Which of the following refers to diagnostic surgical interventions?


a) Re-implantation
b) Amputation
c) Biopsy
d) Laparoscopy, thoracoscopy
e) Transplantation

458. CS. Two hours after stomach resection a patient was transported back to the operating room due to
abundant leakage of blood from the intraperitoneal tubes. Abdominal cavity was reopened to provide
hemostasis. What is the correct term of surgical intervention?
a) Control laparotomy
b) Relaparotomy
c) Bilaparotomy
d) Secondary laparotomy
e) Multiple laparotomy

459. CM. Which of the following interventions refer to simultaneous surgery?


a) Gastric resection for peptic ulcer and cholecystectomy for gallstone disease were performed during the same
surgical intervention
b) Resection of sigmoid colon with application of colostomy was followed by reconstruction of digestive tract
integrity 6 months later
c) Gastric resection was combined with transection of vagus nerve, both interventions being performed for
peptic ulcer disease
d) Laparoscopic treatment of gastro-esophageal reflux disease was associated with cholecystectomy for
gallstone disease, both performed during the same surgical intervention
e) Bilateral removing of varicose veins of lower limbs

460. CM. Which of the following interventions refer to combined surgery?


a) Resection of sigmoid colon with application of colostomy was followed by reconstruction of digestive tract
integrity 6 months later
b) Femoral-popliteal bypass for atherosclerosis and lower limb ischemia was associated with transection of
lumbar sympathetic chain (sympathectomy) during the same surgical intervention
c) Bilateral removing of varicose veins of lower limbs
d) Gastric resection was combined with transection of vagus nerve, both interventions being performed for
peptic ulcer disease
e) Gastric resection for peptic ulcer and cholecystectomy for gallstone disease were performed during the same
surgical intervention

461. CS. According to the classification by grade of infection, the bowel resection is considered:
a) Absolutely clean
b) Clean
c) Infected
d) Septic
e) Purulent

462. CS. According to the classification by grade of infection, the gastric resection is considered:
a) Absolutely clean
b) Clean
c) Infected
d) Septic
e) Purulent

463. CM. Specify local complications that can develop during surgical procedures in general surgery.
a) Acute cardiac failure
b) Hemorrhage
c) Surgical wound abscess
d) Injury of surrounding organs
e) Disseminated intravascular coagulation syndrome

464. CM. The physiological stages of the postoperative period are:


a) Anabolic stage
b) Reactive stage
c) Pre-reactive stage
d) Catabolic stage
e) Transitory stage

465. CM. Catabolic stage of the postoperative period is characterized by:


a) Activation of sympathetic system
b) Activation of parasympathetic system
c) Disintegration of proteins
d) Tissue acidosis
e) Synthesis of proteins

466. CS. What physiological stage of the postoperative period is characterized by synthesis of proteins?
a) Catabolic stage
b) Anabolic stage
c) Transition stage
d) Early stage
e) Remote stage

467. CS. Duration of the early postoperative period is:


a) 8-12 hours
b) 12-24 hours
c) 3-5 days
d) Up to 14 days
e) Up to one month

468. CM. Specify the complications characteristic of the early postoperative period.
a) Development of incisional hernia
b) Stenosis of intestinal anastomosis
c) Hemorrhage
d) Suture sinus
e) Shock (hypovolemic, toxic, cardiac)
469. CM. Specify the complications characteristic of the late postoperative period.
a) Late secondary hemorrhage
b) Acute cardiovascular insufficiency
c) Development of incisional hernia
d) Recurrence of disease
e) Surgical site infection

470. CS. Specify the complications, characteristic of the delayed postoperative period.
a) Late secondary bleeding
b) Acute cardiovascular insufficiency
c) Urinary tract infection
d) Recurrence of disease
e) Surgical site infection

471. CM. Gastric resection for peptic ulcer and cholecystectomy for gallstone disease were performed during the
same surgical intervention. How may this type of surgery be classified according to various principles?
a) Simultaneous surgery
b) Multiple stage surgery
c) Combined surgery
d) Absolutely clean surgery
e) Clean surgery

472. CM. Femoral-popliteal bypass for atherosclerosis and lower limb ischemia was associated with transection of
lumbar sympathetic chain (sympathectomy) during the same surgical intervention. How may this type of
surgery be classified according to various principles?
a) Simultaneous surgery
b) Multiple stage surgery
c) Combined surgery
d) Absolutely clean surgery
e) Clean surgery

473. CM. The term “osteomyelitis” means:


a) Purulent inflammation of the bone marrow
b) Inflammation of the compact substance of bone
c) Inflammation of the periosteum
d) Inflammation of the synovial bursa
e) Inflammation of the bone marrow and adjacent joint

474. CM. Which bones are most often affected by acute hematogenous osteomyelitis?
a) Ulna
b) Fibula
c) Femur
d) Tibia
e) Radius

475. CS. In children aged 2-3 acute hematogenous osteomyelitis most often affects:
a) Metaphysis of bone
b) Diaphysis of of bone
c) Epiphysis of bone
d) Diaphysis and metaphysis of bone
e) All sections of bone

476. CS. In children older than 3 years acute hematogenous osteomyelitis most often affects:
a) Diaphysis of the femur
b) Metaphysis of the femur
c) Epiphysis of the femur
d) Epiphysis and diaphysis of the femur
e) All sections of the femur
477. CS. In children the most frequent form of osteomyelitis is:
a) Acute non-hematogenous osteomyelitis
b) Postoperative osteomyelitis
c) Brodie abscess
d) Acute hematogenous osteomyelitis
e) Sclerozing osteomyelitis of Garre

478. CM. Specify the factors that predispose to the development of acute hematogenous osteomyelitis.
a) Closed injuries of limbs
b) The presence of chronic foci of infection in the body
c) Increased virulence of microorganisms
d) Avitaminosis (vitamin deficiency) and malnutrition
e) All mentioned factors

479. CM. In the first 3-5 days of illness, the most important symptoms of acute hematogenous osteomyelitis are:
a) General symptoms of intoxication
b) Pathology on X-ray examination
c) Local symptoms of purulent inflammation
d) Anemia
e) Severe leukocytosis (15-30 x 109/L)

480. CS. In what form of osteomyelitis the metastatic abscesses develop most frequently?
a) In toxic form of acute hematogenous osteomyelitis
b) In local form of acute hematogenous osteomyelitis
c) In Brodie’s abscess
d) In septic form of acute hematogenous osteomyelitis
e) In albuminous osteomyelitis Olier

481. CM. What symptoms of inflammation appear in acute hematogenous osteomyelitis first of all?
a) Tumor
b) Dolor
c) Calor
d) Rubor
e) Functio laesa

482. CM. Which of these affirmations relate to acute hematogenous osteomyelitis?


a) It develops more common in childhood
b) It is developed in open fractures of limbs
c) It can develop after surgical repositioning of fracture and osteosynthesis
d) Sometimes it occurs with the formation of multiple metastatic abscesses in bones and inner organs
e) The first radiographic signs of illness appear in 10-14 days after the onset of disease

483. CS. In what clinical form of acute hematogenous osteomyelitis, the patient's death can occur within the first 2-
3 days of the onset?
a) In toxic form
b) In local form
c) In septic form
d) In purulent form
e) In all forms

484. CM. Specify the symptoms, based on which the early diagnosis of acute hematogenous osteomyelitis is
made?
a) Acute onset of disease with a pain in limb
b) Severe intoxication on 3rd -4th day of disease
c) Detachment of periosteum from the bone on radiogram
d) Increase of body temperature up to 39-40°C
e) Edema, redness, and fluctuation in the affected region

485. CM. Which of the following symptoms are observed in acute hematogenous osteomyelitis?
a) Edema of the affected limb segment
b) Hyperemia of skin over the focus of inflammation
c) Disturbances of limb function
d) Absence of pulse on peripheral arteries
e) Pathological fracture of the limb with the formation of a false joint

486. CS. Appearance of fluctuation in acute hematogenous osteomyelitis suggests:


a) Development of concomitant thrombophlebitis
b) Development of intramuscular or subcutaneous phlegmon
c) Development of pathological fracture
d) Appearance of osteomyelitic external fistula
e) Development of bullous form of erysipelas

487. CM. Specify the possible complications of osteomyelitis.


a) Sepsis
b) Gas gangrene
c) Primary purulent arthritis
d) Amyloidosis of the inner organs
e) Secondary purulent arthritis

488. CS. The first radiological signs of acute hematogenous osteomyelitis appear on the:
a) 2-3 days of the onset of disease
b) 4-6 days of the onset of disease
c) 7-9 days of the onset of disease
d) 10-14 days of the onset of disease
e) 15-21 days of the onset of disease

489. CM. Specify the radiographic signs, characteristic of acute osteomyelitis.


a) Detachment of the periosteum
b) Appearance of “sequester box”
c) Thickening of the periosteum
d) Pathological fracture
e) Lysis of bone tissue

490. CM. Specify the radiographic signs, characteristic of chronic osteomyelitis.


a) Narrowing of joint space
b) Appearance of “sequester box”
c) Formation of sequesters
d) Detachment of the periosteum
e) Pathological fracture

491. CM. Specify the operations that are performed for chronic osteomyelitis.
a) Long longitudinal incisions
b) Sequestrectomy
c) Plastic operations, aimed at the elimination of bone and tissue defects
d) Amputation
e) Osteoperforation

492. CM. Specify the operations that are performed for acute osteomyelitis.
a) Disarticulation of limb
b) Opening and drainage of the soft tissues phlegmon
c) Osteoperforation
d) Amputation
e) Sequestrectomy

493. CM. Complex treatment of acute hematogenous osteomyelitis includes:


a) Immobilization of the affected limb
b) Antibacterial therapy
c) Detoxication therapy
d) Hormone therapy
e) Surgical treatment

494. CS. The absence of movements in joint is called:


a) Osteoarthritis
b) Arthrosis
c) Rigidity
d) Ankylosis
e) Scoliosis

495. CM. Which of the following should be performed in the presence of positive symptom of fluctuation in the joint
region and the general symptoms of intoxication?
a) Arthrocentesis, aspiration of exudate and intraarticular introduction of antibiotic
b) Amputation of the limb
c) Resection of the articular surfaces with formation of ankylosis
d) Immobilization of limb
e) Systemic administration of antibiotics

496. CM. Specify the clinical symptoms of bursitis.


a) Edema of tissues in the region of joint capsule
b) Pain in the joint
c) Local hyperthermia
d) Hyperemia of the skin
e) Severe limitation of joint mobility

497. CM. Conservative treatment of bursitis includes:


a) Immobilization of the limb
b) Removal of periarthric bursa
c) Administration of anti-inflammatory medicaments
d) Needle aspiration and drainage of periarthric bursa
e) Local hypothermia (ice)

498. CM. In serous bursitis of the elbow joint should be perform:


a) Immobilization of the limb
b) Opening and drainage of periarthric bursa
c) Removal of periarthric bursa
d) Administration of anti-inflammatory medicaments
e) Needle punction of periarthric bursa with aspiration of exudate and introduction of antibiotic

499. CS. According to what principle are fractures divided into open and closed ones?
a) By origin
b) By localization
c) By integrity of the overlying skin
d) By features of the fracture line
e) By position of the bone fragments

500. CM. Depending on features of the fracture line, the bone fractures are classified into:
a) Impacted
b) Incomplete
c) Closed
d) Comminuted
e) Diaphyseal

501. CM. Depending on features of the fracture line, the bone fractures are classified into:
a) Open
b) Transverse
c) Pathological
d) Metaphyseal
e) Oblique
502. CM. Relative symptoms of bone fractures are:
a) Local pain and tenderness
b) Disturbance of limb function
c) False or unnatural motion
d) Bony crepitus
e) Deformation of extremity segment

503. CM. Absolute symptoms of bone fractures are:


a) False or unnatural motion
b) Pain in the region of injury
c) Deformation of extremity segment
d) Disturbance of limb function
e) Bony crepitus

504. CM. Pathological fracture may be caused by:


a) Bone tumor
b) Bone form of gas gangrene
c) Chronic osteomyelitis
d) Osteoporosis
e) Purulent arthritis

505. CS. Which type of fractures is characterized by highest probability of bacterial contamination?
a) Intrauterine
b) Comminuted
c) Open
d) Closed
e) Pathological

506. CM. What cells take part in the formation of callus in fractures?
a) Cells of endoosseous
b) Cells of Haversian cannals
c) Cells of periosteum
d) Cells of adipose tissue around the fracture
e) Cells of connective tissue around the fracture

507. CS. The formation of primary callus takes on average:


a) 2-4 weeks
b) 4-6 weeks
c) 6-8 weeks
d) 8-10 weeks
e) 10-12 weeks

508. CS. The greatest regenerative capacity in formation of callus have:


a) Cells of endoosseous
b) Cells of Haversian canals
c) Cells of connective tissue around the fracture
d) Cells of cambial layer of periosteum
e) All mentioned above

509. CM. Which of the following fractures are considered complicated:


a) “Greenstick” fracture
b) Fracture combined with injury of the major artery
c) Fracture developed due to bone tumor
d) Fracture combined with nerve injury (paresis, paralysis)
e) Fracture developed due to chronic osteomyelitis

510. CM. Specify the possible local complications of fractures.


a) Traumatic shock
b) Abnormal consolidation of bone fragments
c) Damage of vessels and nerves
d) Fat embolism
e) Posttraumatic (nonhematogenous) osteomyelitis

511. CS. Specify the most common complication of open fractures.


a) Fat embolism
b) Shortening of the damaged limb
c) Paralysis of limb
d) Osteomyelitis
e) Amyloidosis

512. CM. Specify the possible general complications of fractures.


a) Traumatic shock
b) Disseminated intravascular coagulation
c) Acute anemia
d) Formation of pathological bone callus
e) Paralysis of limb

513. CM. In fractures the first medical aid includes:


a) Administration of analgesics
b) Transport immobilization with special (medical) or improvised devices
c) In open fractures – hemostasis, application of aseptic bandage on the wound
d) Reduction of bone fragments in open fractures (perforation of skin by the bone fragment)
e) Elimination of displacement of bone fragments and limb deformities

514. CM. In order to transport immobilization in fractures are used:


a) Improvised splints from different materials
b) Cramer’s splint
c) Beller’s splint
d) Pneumatic splint
e) Dieterich’s splint

515. CS. The Dieterich’s splint is used for:


a) Transport immobilization of the upper limbs
b) Transport immobilization of the lower extremities
c) Treatment of fractures of the upper extremity
d) Treatment of fractures of the lower extremity
e) Treatment of spinal fractures

516. CM. In open fractures the first medical aid does not include:
a) Performing of temporary hemostasis
b) Administration of analgesics
c) Reposition of bone fragments
d) Transport immobilization
e) Placement of primary sutures on the wound

517. CM. Specify the possible causes of the delay in formation of callus.
a) Incomplete reposition of bone fragments
b) Partial or total interposition of soft tissues between the bone fragments
c) Inadequate immobilization
d) Diabetes mellitus in patient
e) Intramedullary osteosynthesis

518. CM. In fractures a prolonged immobilization of limb can lead to:


a) False joint
b) Hemarthrosis
c) Epiphysiolysis
d) Contractures of the limb
e) Muscular atrophy

519. CM. Penetration of pathogenic organisms in the soft tissues of hands most often occurs:
a. By hematogenous route
b. Through calluses, cracks of the skin
c. By lymphogenous route
d. With small foreign bodies (splinters, pieces of metal)
e. From infected phalangeal and wrist bones

520. CS. In most cases, purulent processes on fingers and hands are caused by:
a. Streptococcus pyogenes
b. Staphylococcus aureus
c. Escherichia coli
d. Anaerobic non-clostridial organisms
e. Candida albicans

521. CM. Specify the anatomical features of fingers and hand, which determine special manifestations of the
inflammatory process.
a. Skin of the palmary surface is strong, thick and not elastic, which explains the low probability of spontaneous
rupture of purulent focus
b. Flexor tendon sheath of the I and V fingers communicate with cellular spaces of forearm, which can lead to
spread of infection to the forearm
c. Flexor tendon sheath of the II, III and IV fingers communicate with cellular spaces of forearm, which can lead
to spread of infection to the forearm
d. Adipose tissue of the palmary surface is divided by fibrous septa in separate cells, therefore abscess tends to
spontaneous rupture on the skin
e. Adipose tissue of the palmary surface is divided by fibrous septa in separate cells, therefore abscess tends to
spread in depth

522. CM. Tenosynovitis of which fingers can lead to the spread of infection into Pirogov-Parona’s cellular space of
forearm?
a. The first
b. The second
c. The third
d. The fourth
e. The fifth

523. CM. Specify the reasons of particularly expressed pain in felon, which is not comparable with pains in
purulent processes of other localization.
a. Compression of nerves by inflammatory exudate
b. Compression of vessels by inflammatory exudate and tissue ischemia
c. Rapidly occurring secondary necrosis of the tendons and bone
d. Effect of inflammatory mediators, released in large amounts
e. The rich sensitive innervations of the fingers palmar surface

524. CS. In purulent tenosynovitis necrosis of the tendons is caused by:


a. Direct damaging effect of bacterial enzymes
b. Restriction of active movements
c. Compression by inflammatory exudate and thrombosis of blood vessels, which supply the tendon
d. Compression of nerves and tendons by inflammatory exudate
e. Iatrogenic vascular injury during surgical treatment for felon

525. CM. What location of abscess is defined as a felon?


a. Palmar surface of nail phalanx of hand finger
b. Plantar surface of nail phalanx of feet finger
c. Pirogov-Parona’s cellular space of forearm
d. Nail area of hand finger
e. Dorsal surface of proximal phalanx of hand finger
526. CM. Specify the superficial forms of felon.
a. Subcutaneous
b. Tenosynovitis
c. Pandactylitis
d. Paronychia
e. Cutaneous

527. CS. Paronychia is defined as a localization of purulent process:


a. Under the nail plate
b. Under the nail proximally
c. On the palmar surface of the nail phalanx
d. On the lateral nail fold
e. Ingrown nail in the nail fold of toe with inflammation

528. CM. Specify the deep forms of felon.


a. Tenosynovitis
b. Bone
c. Articular
d. Subungual
e. Pandactylitis

529. CM. What characterizes the cutaneous felon?


a. Flexed resting posture of affected finger
b. Pain is not characteristic
c. Inflammatory process may spend to underlying bone, joint or flexor tendons
d. In most cases inflammatory process is localized on palmar surface of distal finger phalanx
e. Is diagnosed as a superficial vesicle with purulent contents

530. CS. What form of panaritium is most likely, if on the palmar surface of thumb is determined a small vesicle
filled with a purulent contents?
a. Cutaneous
b. Subcutaneous
c. Tenosynovitis
d. Subungual
e. Pandactylitis

531. CS. Specify the optimal method of anesthesia for the treatment of cutaneous panaritium.
a. Local infiltrative
b. Locoregional
c. General intravenous
d. General inhalational
e. Anesthesia is not used

532. CM. What includes surgical treatment of cutaneous felon?


a. Immobilization of affected finger after drainage of purulent focus
b. Removing the affected epidermis and drainage of pus
c. Application of antiseptic dressings after drainage of purulent focus
d. Purulent focus is excised within the healthy tissues
e. After proliferation of granulation tissue, secondary sutures are applied to the wound

533. CM. What characterizes the subcutaneous felon?


a. Is the most common form of felon
b. Inflammation is always limited to subcutaneous tissue
c. May complicate by spread of inflammation on the bone, joint and tendons
d. In most cases inflammation is located on palmar surface of nail phalanx
e. Is diagnosed at appearance of fluctuation

534. CM. How to determine the most painful site and, therefore, location of purulent collection in case of
subcutaneous felon?
a. With probe
b. With finger of examiner
c. With tip of forceps
d. During active and passive movements of finger
e. Fine needle aspiration of the purulent cavity

535. CS. Specify the most common method of anesthesia used for surgical treatment of subcutaneous felon.
a. Local infiltrative
b. Locoregional of Oberst-Lukashevich
c. General intravenous with myorelaxation
d. General inhalational with myorelaxation
e. Anesthesia is not used

536. CM. How to drain the purulent focus in subcutaneous felon?


a. Perform parallel incisions on the anterolateral surface of the nail phalanx
b. Wide excision of inflamed and necrotic tissues on palmar surface of nail phalanx
c. Incisions do not extend to interphalangeal folds
d. Perform Clapp’s incision (arcuate, parallel to the edge of the nail)
e. The entire nail or a part of nail is removed

537. CM. What characterizes the paronychia?


a. Process can become chronic
b. On pressing from under nail fold is released a drop of pus
c. Infection results from trauma to the eponychial or paronychial region
d. In the nail fold appear mild pain, swelling and redness
e. Abscess can spread to the palmar surface of nail phalanx in the form of “hourglass”

538. CS. Clapp’s incision (arcuate, parallel to the edge of the nail) is used for the treatment of:
a. Cutaneous felon
b. Purulent tenosynovitis
c. Paronychia
d. Subungual felon
e. Bone felon

539. CM. How to drain the purulent focus in paronychia?


a. Perform parallel incisions on the anterolateral surface of the nail phalanx
b. Evacuate pus by simple pressure on the nail fold
c. It is always necessary to remove the entire nail
d. Perform Clapp’s incision (arcuate, parallel to the edge of the nail)
e. It is sometimes necessary to remove the part of nail

540. CM. What characterizes the purulent tenosynovitis?


a. Tenosynovitis of V-th finger can spread to the deep cellular spaces of forearm
b. Flexed resting posture of affected finger
c. Infection penetrates in the tendon through the cracks of skin
d. Pain is not characteristic
e. Tenosynovitis of IV-th finger can spread to the deep cellular spaces of forearm

541. CM. Select the classic signs of Kanavel, described in pyogenic flexor tenosynovitis.
a. Tenderness along the course of the flexor tendon
b. Exquisite pain on slightest attempt of passive extension
c. Symmetrical fusiform swelling of the entire finger
d. Flexed resting posture of finger
e. Sharp increasing of pain when patient lowers his hand down

542. CS. What incision is recommended for purulent tenosynovitis?


a. Semilunar incision on the distal phalanx
b. Longitudinal incisions on both lateral surfaces of the distal phalanx
c. Longitudinal incisions on both lateral surfaces of the middle and proximal phalanxes
d. Longitudinal incision on the midline line of the finger palmar surface along the full length of the tendon sheath
e. Multiple transverse incisions on all phalanxes

543. CM. How to drain the purulent focus in purulent tenosynovitis?


a. Flexor tendon sheaths are opened with parallel incisions on the anterolateral surface of the distal phalanx
b. Flexor tendon sheaths are opened with parallel incisions on the anterolateral surface of the distal, middle and
proximal phalanxes
c. Removal of flexor tendon is an obligatory component of primary surgery
d. It is only necessary to remove the flexor tendon in case of necrosis
e. Flexor tendon sheaths are opened with parallel incisions on the anterolateral surface of the middle and
proximal phalanxes

544. CM. What characterizes the bone felon?


a. Process can become chronic
b. Bone felon of I and V fingers can spread to the deep cellular spaces of forearm
c. As a rule, this is an outcome of untreated subcutaneous felon
d. Infection penetrates in the bone through the cracks of skin
e. Typical localization is the nail phalanx

545. CM. Presence of bone sequesters in inflammation of hand fingers is observed in:
a. Pyogenic flexor tenosynovitis
b. Commissural phlegmon
c. Paronychia
d. Bone felon
e. Pandactylitis

546. CS. Radiographic changes at bone felon appear:


a. On the next day after the onset of disease
b. After 3-5 days after the onset of disease
c. After 5-7 days after the onset of disease
d. After 10-12 days after the onset of disease
e. After 1.5-2 months after the onset of disease

547. CM. Specify the compound components of bone felon treatment.


a. Administration of broad-spectrum antibiotics
b. Elevating and immobilizing the affected finger and hand
c. Dissection and excision of purulent fistula
d. Removal of bone sequesters
e. Bone felon is an absolute indication for amputation of finger

548. CS. Increasing of pain on percussion along the axis of finger is especially characteristic for:
a. Bone felon
b. Tenosynovitis
c. Articular felon
d. Subcutaneous felon
e. Paronychia

549. CS. The term “pandactylitis” refers to:


a. Purulent inflammation of all fingers of the hand
b. Purulent inflammation of all digits of the foot
c. Purulent inflammation of all tissues of one finger of the hand
d. Purulent inflammation of all tissues of one digit of the foot
e. Fusion of all fingers of hand or foot

550. CS. A forced amputation of the finger is most often necessary for:
a. Felon in the form of a hourglass
b. Pyogenic flexor tenosynovitis
c. Articular felon
d. Bone felon
e. Pandactylitis

551. CS. In surgical treatment of felon for wound drainage is most frequently is used:
a. Active drainage tube
b. Strips of rubber glove
c. Gauze turunda
d. Washing tubular drainage
e. Drainage in form of a “cigar”

552. CM. Specify the anatomical forms of deep palmar surface phlegmon.
a. Phlegmon of midpalmar space
b. Phlegmon of thenar
c. Subcutaneous phlegmon
d. Phlegmon of hypothenar
e. Subaponeurotic phlegmon

553. CM. Specify the anatomical forms of phlegmon of the dorsal hand surface.
a. Skin abscess
b. Phlegmon of midpalmar space
c. Interdigital (commissural) phlegmon
d. Subcutaneous phlegmon
e. Subaponeurotic phlegmon

554. CM. Specify the correct statements regarding the “forbidden zone” of hand.
a. “Forbidden zone” is located on the palmar surface of the thenar
b. “Forbidden zone” is located on the palmar surface of the hypothenar
c. “Forbidden zone” is located on the dorsal surface of the hand
d. In “forbidden zone” the motor branches of the median nerve pass
e. In “forbidden zone” the sensory branches of the radial nerve pass

555. CS. Through drainage by small incisions on the palmar and dorsal surfaces of hand in the areas of interdigital
folds is used for treatment of:
a. Phlegmon of thenar
b. Subcutaneous phlegmon of the dorsal surface of hand
c. U-shaped phlegmon of hand
d. Commissural phlegmon of hand
e. Phlegmon of hypothenar

556. CM. In phlegmon of the palmar surface the edema of the dorsal surface of hand:
a. Occurs extremely rare
b. Is due exceptionally to rupture of pus on the dorsal surface of hand
c. Occurs in most cases
d. Is due to features of lymph drainage
e. Appears early as a result of friable structure of subcutaneous tissue

557. CS. Why is weight deficit dangerous in surgical patients?


a) Major surgery cannot be performed in these patients
b) Weight deficit is not a danger to surgical patients
c) Additional weight loss associated with surgery is unacceptable in these patients
d) Postoperative complications and mortality are significantly increased in these patients
e) Weight deficit is a favorable factor, because it facilitates the technical aspects of surgical operation

558. CM. What are the main causes of malnutrition in surgical patients?
a) Interruption of normal food intake during preoperative diagnostic tests
b) A pathological process itself, that affects a patient
c) Administration of antibiotics, which reduces absorption of nutrients
d) Depressed state and loss of appetite in patients, who expects a surgery
e) Restriction of food intake after majority of surgical procedures

559. CM. Deficiencies of weight in patients with cancer is caused by:


a) Deficiencies of proteins
b) Deficiencies of calories
c) Deficiencies of trace elements
d) Deficiencies of fats
e) Deficiencies of vitamins

560. CM. What signs of malnutrition may be detected on the inspection of patient’s skin?
a) Decrease of elasticity
b) Bullae, filled with transparent fluid
c) Rash
d) Hyperemia
e) Malignant melanoma

561. CM. What signs of malnutrition may be detected on the examination of patient’s extremities?
a) Decreased arterial pulse
b) Reduction of muscle size and strength
c) Symmetrical pedal edema
d) Hyperemia
e) Tenderness on palpation

562. CS. What signs of malnutrition may be detected on the inspection of patient’s nails?
a) Ingrown nail
b) Frailty and deformities
c) "Hour glasses" type
d) Loss of nails
e) Paronychia

563. CM. What signs of malnutrition may be detected on the examination of patient’s eyes?
a) Exophthalmos
b) Subcorneal hematoma
c) Keratoconjunctivitis
d) Impairment of vision
e) Cataracts

564. CS. What signs of malnutrition may be detected on the inspection of patient’s tongue?
a) “Geographic” tongue
b) Leukoplakia of the tongue
c) Dry tongue with a dirty coating
d) Fissures on the surface of the tongue
e) Brightly red tongue, with prominent papillas (glossitis)

565. CM. What data that predispose to malnutrition, can be found on examination of the abdomen of patient?
a) Presence of intestinal stoma
b) Entero-cutaneous fistula
c) Incisional hernia
d) Distended bowel loops
e) Palpable abdominal tumors

566. CM. Specify the most frequent clinical reasons for decrease of serum electrolyte concentration.
a) Severe diarrhea
b) Overdosage of diuretics
c) Decreased excretion in renal dysfunction
d) Stenosing tumor of the colon with chronic constipation
e) Morbid obesity

567. CS. Which of the following laboratory parameters most closely correlates with body protein deficiency?
a) Serum albumin
b) Serum prothrombin
c) Serum globulin
d) Blood protein
e) Lymphocyte count in blood

568. CM. What alterations of immune function are characteristic for patients with malnutrition?
a) Delayed-type hypersensitivity
b) Decrease in total (absolute) lymphocyte count
c) Anaphylactic hypersensitivity
d) Decrease in the percentage of lymphocytes
e) Decrease in total (absolute) leucocytes count

569. CM. What methods of the nutritional status assessment are related to anthropometric ones?
a) Total (absolute) lymphocyte count in blood
b) Evaluation of dietary history
c) Correlation between the thickness of the triceps skin fold and the mid-arm muscle circumference
d) Calculation of body weight deficit
e) Calculation of body mass index

570. CM. The following indicators can be used for the assessment of body weight loss:
a) Insufficient body weight
b) Usual body weight
c) Actual body weight
d) Desirable body weight
e) Ideal body weight

571. CS. The calculation of the body mass index is done using the following formula:
a) % lymphocytes x WBC / 100
b) Weight (kg) / height (m2)
c) Actual weight (100) / ideal weight
d) 48.1 kg for height 152 cm plus 1.1 kg for every 1 cm over 152 cm
e) Weight (g) / height (cm2)

572. CS. Which of the following values of the body mass index correspond to normal weight?
a) 18.5-24.9
b) 25.0-29.9
c) 30.0-34.9
d) 35.0-39.9
e) 40 and more
573. CS. Which of the following values of the body mass index correspond to morbid obesity?
a) 18.5-24.9
b) 25.0-29.9
c) 30.0-34.9
d) 35.0-39.9
e) 40 and more

574. CS. Measurement of thickness of the triceps skin fold is used for assessment of:
a) Reserves of body trace elements
b) Reserves of body protein
c) Reserves of body carbohydrates
d) Reserves of body fat
e) Reserves of body vitamins

575. CS. Calculation of correlation between the thickness of the triceps skin fold and the mid-arm muscle
circumference is used to assess:
a) Reserves of trace elements in the body
b) Reserves of protein in the body
c) Reserves of carbohydrates in the body
d) Reserves of fat in the body
e) Reserves of vitamins in the body

576. CM. What methods are used to determine patients’ energy requirements?
a) Indirect calorimetric measurements (Weir formula)
b) Harris-Benedict equation
c) Calculation of body mass index
d) By the level of plasma albumin
e) The ration between thickness of the triceps skin fold and mid-humeral circumference

577. CS. In what surgical conditions the daily energy consumptions of patients are maximal?
a) After elective uncomplicated surgery
b) After polytrauma
c) In case of surgical sepsis
d) After major burns
e) In case of malignant tumor

578. CS. Enteral feeding is indicated for patients:


a) Who have functional digestive tract, but unable sustain an adequate oral intake
b) Who have upper entero-cutaneous fistulas
c) Who have intestinal obstruction
d) Who have upper gastrointestinal bleeding
e) Who have severe diarrhea

579. CM. Enteral feeding is contraindicated for patients:


a) Who have gastrointestinal bleeding
b) Who have severe diarrhea
c) Who have functional digestive tract, but who are unable to sustain an adequate oral intake
d) Who have upper entero-cutaneous fistulas
e) Who have intestinal obstruction

580. CM. What ways are used for enteral feedings administration?
a) Via nasogastric tube
b) Via gastrostomy
c) Via Sengstaken-Blakemore esophagogastric tube
d) Via rectal tube
e) Via subclavian venous catheter
581. CS. Energy value of standard solutions for enteral feeding is:
a) 5 kcal/mL
b) 10 kcal/mL
c) 0.5 kcal/mL
d) 3 kcal/mL
e) 1 kcal/mL

582. CM. What enteral feeding solutions (formulas) are available?


a) Modular formulas
b) Caloric enteral diets
c) Standard enteral diets
d) Blenderized tube feeding enteral diets
e) Chemically defined formulas (elemental diets)

583. CS. What enteral feeding solutions are prepared from conventional food that can be mixed?
a) Blenderized tube feeding enteral diets
b) Modular formulas
c) Standard enteral diets
d) Caloric enteral diets
e) Chemically defined formulas (elemental diets)

584. CS. What enteral feeding solutions are prepared for administration in specific clinical situations (pulmonary,
renal or hepatic failure, immune dysfunction)?
a) Chemically defined formulas (elemental diets)
b) Blenderized tube feeding enteral diets
c) Standard enteral diets
d) Caloric enteral diets
e) Modular formulas

585. CS. What enteral feeding solutions contain protein in the form of free amino acids?
a) Caloric enteral diets
b) Chemically defined formulas (elemental diets)
c) Standard enteral diets
d) Blenderized tube feeding enteral diets
e) Modular formulas

586. CM. What protocols of enteral feeding are used?


a) Continuous infusion
b) Partial feeding
c) Bolus (fractional) feeding
d) Mechanical feeding
e) Total feeding

587. CS. How frequently is recommended to introduce nutritional formulas in bolus (fractional) enteral feeding?
a) Every hour
b) Every 2 hours
c) Every 4 hours
d) Every 6 hours
e) Every 8 hours

588. CM. What complications are characteristic of enteral nutrition?


a) Metabolic
b) Diarrhea
c) Mechanical
d) Infectious
e) Tracheobronchial aspiration

589. CS. Hyperglycemia refers to the following group of enteral nutrition complications:
a) Mechanical
b) Laboratory
c) Chemical
d) Infectious
e) Metabolic

590. CM. Which patients have the greatest risk of tracheobronchial aspiration during the enteral feeding?
a) Patients with central nervous system abnormalities
b) Patients with diabetes mellitus
c) Medicamentous sedated patients
d) Patients with upper entero-cutaneous fistulas
e) Predominantly patients with bolus (fractional) enteral feeding

591. CM. Parenteral nutrition is indicated for patients:


a) After a massive bowel resection
b) With diabetes mellitus
c) In early postoperative period after partial gastrectomy
d) In early postoperative period after hemorrhoidectomy
e) In neurological coma

592. CM. Parenteral nutrition is indicated for patients:


a) With upper entero-cutaneous fistulas
b) With central nervous system abnormalities
c) With prolonged and severe diarrhea
d) With mechanical intestinal obstruction
e) With severe immunosuppression

593. CM. The types of parenteral nutrition are:


a) Continuous
b) Partial
c) Bolus (fractional)
d) Mechanical
e) Total

594. CM. Specify the components which are included in solution for parenteral nutrition.
a) Amino acids
b) Proteins
c) Lipid emulsion
d) Sucrose
e) Dextrose

595. CS. How should solutions for parenteral nutrition be administered?


a) Via nasogastric tube
b) Via gastrostomy
c) Intravenously
d) Intraarterially
e) Intramuscularly

596. CM. What complications of total parenteral nutrition are distinguished?


a) Mechanical
b) Chemical
c) Combined
d) Metabolic
e) Infectious

597. CM. What complications are common for morbid obesity?


a) Systemic hypertension
b) Type II diabetes
c) Osteomyelitis
d) Joint diseases
e) Cholelithiasis

598. CM. What complications are characteristic of morbid obesity?


a) Fat induced liver diseases
b) Thromboembolic disorders
c) Psychosocial problems
d) Endocrine dysfunction
e) Peptic ulcer disease

599. CS. Specify the most effective method of treatment for morbid obesity.
a) Jejuno-ileal bypass (anastomosis of proximal jejunum to terminal ileum)
b) Introduction of the balloon into the stomach in order to reduce its volume
c) Restrictive diets
d) Horizontal and vertical gastroplasty
e) Esophago-intestinal anastomosis

600. CS. The common principle of gastroplasty in surgical treatment of morbid obesity is:
a) To create anastomosis of proximal jejunum to terminal ileum
b) Endoscopic installation of special balloon into the stomach in order to decrease its volume
c) Administration of special reducing diets
d) To create a small (30-50 mL) proximal pouch across the proximal stomach and a small (1 cm) channel for the
passage of food
e) To create an oesophago-intestinal anastomosis with excluding of the stomach from the passage of food

601. CM. The local manifestations of wounds are the following:


a) Bleeding
b) Pain
c) Damage of inner organs
d) Dehiscence
e) Shock

602. CS. Why should damage to the liver in blunt abdominal trauma be considered as a rupture, rather than a
wound?
a) Because there is no pain
b) Because there is no breakup of cover tissues
c) Because there is no acute anemia and shock
d) Because there is no functional disturbance of damaged organ
e) Because there is no bleeding
603. CS. What is the main clinical symptom, which distinguishes a wound from contusion?
a) The presence of bleeding
b) It occurs due to external mechanical impact
c) Pain in the site of injury
d) Functional disturbances of damaged organ
e) Defect of cover tissues

604. CS. What is the mechanism of traumatic injury in open fracture of extremity, when the fragment of the broken
bone perforates the soft tissue?
a) External
b) Internal
c) Combined
d) Traumatic
e) Indirect

605. CM. Pain in the wound is causes:


a) By direct irritation of the nerve endings
b) By ischemia of damaged zone with thrombosis of small blood vessels
c) By palsy of nerve endings due to development of tissue acidosis
d) By compression of nerve endings due to edema
e) By rapid development of local inflammatory reaction

606. CM. Severity of pain in a wound depends on the following factors:


a) Duration of injure
b) Intensity of bleeding
c) Number of nerve endings in the tissues of damaged zone
d) Acuteness of traumatic agent
e) Power of immune response

607. CM. In what circumstances the pain at injury (in the wound) can be significantly reduced or absent?
a) When the surgical intervention is performed under local or general anesthesia
b) When the wound is produced by a sharp instrument, such as a scalpel
c) In injured patients with shock
d) In injured patients with severe alcohol intoxication
e) When the wound is penetrating just to a depth of subcutaneous tissue

608. CM. Intensity of bleeding from a wound is determined by:


a) State of systemic hemodynamics
b) Speed of damage
c) Depth of wound
d) State of coagulation system
e) Diameter and type (arterial or venous) of damaged vessel

609. CM. In what circumstances the injury, even small-caliber vessels may be accompanied by severe bleeding,
and to be life-threatening for patient?
a) In patients with liver cirrhosis
b) In patients with low blood pressure
c) In patients with hemophilia
d) In patients, receiving anticoagulants
e) In severe drunken patients

610. CM. What factors determine the degree of wound edges dehiscence?
a) Time passed since injury
b) Speed of damage
c) Grade of contamination of traumatic agent
d) Relationship between direction of wound and the lines of Langer
e) Size and depth of the wound

611. CM. What factors cause general clinical manifestations of wounds?


a) Degree of wound edges dehiscence
b) Acuteness of traumatic agent
c) Development of inflammatory processes
d) Direct injury of nerve endings
e) Anemia and shock

612. CM. What wounds are considered to be intentional?


a) Criminal wound
b) Battle wound
c) Accidental wound
d) Industrial wound
e) Surgical wound

613. CM. What wounds are distinguished depending on the nature of traumatic agent?
a) Chopped wound
b) Lacerated wound
c) Cut wound
d) Gunshot wound
e) Accidental wound

614. CM. What wounds are distinguished according to the nature of traumatic agent?
a) Stab wound
b) Bite wound
c) Compound wound
d) Surgical wound
e) Purulent wound

615. CS. What kind of damage does not refer to the classification of wounds by the nature of traumatic agent?
a) Industrial wound
b) Stub wound
c) Lacerated wound
d) Bite wound
e) Contusioned wound

616. CS. To what type of wounds according to the nature of traumatic agent should be attributed a surgical
incision?
a) Industrial wound
b) Stub wound
c) Compound wound
d) Cut wound
e) Accidental wound

617. CM. Specify the clinical features of stab wounds.


a) Dehiscence of wound borders is significant
b) High risk of injury of inner structures
c) Dehiscence of wound borders is not significant
d) External bleeding is not major
e) It occurs in animal bites

618. CS. What is the mechanism of contusioned wound?


a) Formed by action of a massive sharp object
b) Formed by action of a massive blunt object
c) Formed by gunshot injury
d) Formed due to bites of a large animals
e) Formed by action of a pointed object

619. CS. Specify the type of wounds, which is considered the most contaminated.
a) Chopped wound
b) Stab wound
c) Contusioned wound
d) Cut wound
e) Bite wound

620. CM. Specify clinical features of bite wounds.


a) Danger of rabies development
b) High risk of development of banal or anaerobic infection
c) Danger of inner organs damage
d) Presence of three zones of tissue alteration
e) Small zone of tissue alteration

621. CM. According to the classification by grade of contamination, wounds are divided into:
a) Contaminated
b) Aseptic
c) Necrotic
d) Putrid
e) Purulent

622. CS. Which wound is considered as a contaminated one?


a) A surgical wound, if during the operation the bowel lumen is opened
b) Every accidental wound
c) A wound, in which the concentration of microorganisms more than 103 (1.000) per 1 gram of tissue
d) A wound, in which purulent process has already developed
e) A wound, in which the concentration of microorganisms more than 105 (100.000) per 1 gram of tissue

623. CS. It is known that purulent process in the wound develops when the concentration of microorganisms is
more than:
a) 105 (100.000) of microorganisms per 1 gram of tissue
b) 104 (10.000) of microorganisms per 1 gram of tissue
c) 103 (1.000) of microorganisms per 1 gram of tissue
d) 102 (100) of microorganisms per 1 gram of tissue
e) 101 (10) of microorganisms per 1 gram of tissue

624. CM. What factors contribute to the development of infection in the wound?
a) Presence of blood clots and devitalized tissues in the wound
b) Decrease of human response to infection (anemia, shock, immunosuppression, diabetes)
c) Ischemia of damaged zone
d) High grade of bacterial contamination
e) Significant dehiscence of wound borders
625. CM. What wounds of the abdomen should be considered as penetrating?
a) Wounds, associated with injury of skin, subcutaneous adipose tissues, aponeurosis, parietal peritoneum, and
bowel
b) Wounds, associated with injury of skin and subcutaneous adipose tissues
c) Wounds, associated with injury of skin, subcutaneous adipose tissues, and aponeurosis
d) Wounds, associated with injury of skin, subcutaneous adipose tissues, aponeurosis, and muscles of the
anterior abdominal wall
e) Wounds, associated with injury of skin, subcutaneous adipose tissues, aponeurosis, muscles of the anterior
abdominal wall, and parietal peritoneum

626. CM. What zones of tissue alteration are distinguished in gunshot wounds?
a) Zone of primary traumatic necrosis
b) Zone of traumatic edema
c) Wound channel
d) Zone of molecular concussion
e) Zone of tertiary necrosis

627. CS. The zone of molecular concussion in gunshot wounds is characterizes:


a) By traumatic necrosis developing due to direct action of projectile kinetic energy
b) By penetration and rapid development of infection in the wound channel
c) By heterogeneous tissue edema along the wound channel
d) By affected tissue metabolism and damaged cellular structures
e) By complicated anatomical character of gunshot wound channel

628. CM. What features differentiate a gunshot wound?


a) Presence of three zones of tissue alteration
b) Heals by primary wound healing
c) Complicated anatomical character of wound channel
d) High grade of contamination
e) Primary surgical debridement (processing) of wound is finished by placement of sutures

629. CM. What characterizes the perforating (through-out) gunshot wound?


a) It has only an incoming hole
b) Only a part of kinetic energy of bullet converts into energy of tissue destruction
c) It has an incoming and outcoming holes
d) The whole kinetic energy of bullet converts into energy of tissue destruction
e) Only superficial tissues are damaged with no penetration inside of body

630. CM. Blind gunshot wound is characterizes by:


a) Only superficial tissues are damaged with no penetration inside the body
b) It has only an incoming hole
c) It has an incoming and outcoming holes
d) The whole kinetic energy of bullet converts into energy of tissue destruction
e) Only a part of kinetic energy of bullet converts into energy of tissue destruction

631. CM. Tangential gunshot wound is characterizes by:


a) Only superficial tissues are damaged with no penetration inside the body
b) It has only an incoming hole
c) It has an incoming and outcoming holes
d) The whole kinetic energy of bullet converts into energy of tissue destruction
e) It is not associated with damage of inner organs and structures
632. CM. Wound healing process has the following phases:
a) Secondary wound healing
b) Epithelization and reorganization of a scar
c) Proliferation
d) Primary wound healing
e) Inflammation

633. CS. What phase of wound healing process includes the period of angiogenesis and the period of wound
cleaning?
a) The phase of secondary wound healing
b) The phase of epithelization and reorganization of scar
c) The phase of proliferation
d) The phase of primary wound healing
e) The phase of inflammation

634. CM. Which periods includes the inflammation phase of wound healing process?
a) The period of hydration
b) The period of recanalization and growth of blood vessels
c) The period of dehydration
d) The period of wound cleaning from necrotic masses
e) The period of angiogenesis

635. CS. What is the approximate duration of the first phase of wound healing process (phase of inflammation)?
a) 6-14 days
b) 1-2 days
c) Over 1 month
d) 1-5 days
e) 14-21 days

636. CS. Which of the following phenomena does not refer to the first phase of wound healing process (phase of
inflammation)?
a) Short-term vasoconstriction, which subsequently replaces with prolonged vasodilatation
b) Thrombosis of capillaries and small veins
c) Tissue acidosis
d) Proliferation of granulation tissue
e) Migration of liquid in extracellular space and tissue edema

637. CM. Which of the following events refer to the first period (period of angiogenesis) of the inflammation phase
of wound healing process?
a) Recanalization and growth of blood vessels
b) Tissue edema
c) Phagocytosis
d) Increased permeability of vascular wall
e) Migration of leukocytes

638. CM. Which of the following events refer to the second period (period of wound cleaning from necrotic
masses) of the inflammation phase of wound healing process?
a) Recanalization and growth of blood vessels
b) Tissue edema
c) Phagocytosis
d) Collagen synthesis
e) Lysis of necrotic tissues
639. CM. Which of the following cells play a key role in the first phase of wound healing process (phase of
inflammation)?
a) Lymphocytes
b) Fibroblasts
c) Macrophages
d) Neutrophils
e) Erythrocytes

640. CS. What is the role of polymorphonuclear neutrophils in the first phase of wound healing process (phase of
inflammation)?
a) Phagocytosis of microorganisms and necrotic masses
b) Realization of immune response
c) Release of proteolytic enzymes
d) Synthesis of collagen
e) Release of prostaglandins and interleukins

641. CS. What is the role of macrophages in the first phase of wound healing process (phase of inflammation)?
a) Realization of immune response
b) Cross-linking of collagen
c) Release of proteolytic enzymes and phagocytosis of necrotic tissues
d) Synthesis of collagen
e) Release of prostaglandins and interleukins

642. CS. What is the role of lymphocytes in the first phase of wound healing process (phase of inflammation)?
a) Phagocytosis of microorganisms and necrotic masses
b) Realization of immune response
c) Release of proteolytic enzymes
d) Synthesis of collagen
e) Release of prostaglandins and interleukins

643. CM. Which of the following phenomena refers to the second phase of wound healing process (phase of
proliferation)?
a) Recanalization and proliferation of blood vessels
b) Thrombosis of capillaries and small veins
c) Tissue acidosis
d) Proliferation of granulation tissue
e) Synthesis of collagen

644. CS. Which of the following cells play the main role in the second phase of wound healing process (phase of
proliferation)?
a) Lymphocytes
b) Fibroblasts
c) Macrophages
d) Neutrophils
e) Platelets

645. CS. What is granulation tissue?


a) It is an epithelial tissue, which covers wound defect
b) It is a firm scar tissue
c) It is a necrotic tissue with high contents of microorganisms
d) It is a gentle connective tissue with newly formed capillaries
e) It is a dense clot in the wound, which is formed as a result of adhesion and aggregation of platelets and
thrombosis of capillaries and small veins
646. CM. What are physiological functions of granulation tissue?
a) Wound protection from subsequent bacterial contamination
b) Hemostasis
c) It contributes to restoration of skin integrity
d) Replacement of tissue defect
e) Sequestration and rejection of necrotic masses

647. CM. The third phase of wound healing process (phase of epithelization and reorganization of scar) is
characterizes by:
a) Tissue acidosis
b) Decrease of fibroblasts activity
c) Phenomenon of wound contraction
d) Cross-linking of collagen
e) Infiltration of tissues with leukocytes

648. CS. The phenomenon of wound contraction during its healing explained by:
a) Replacement of wound defect with granulation tissue
b) Disappearance of edema (dehydration of the wound)
c) Crosslinking of collagen fibers
d) Migration of fibroblasts into the wound, and synthesis of collagen and elastic fibers
e) Proliferation of capillary network in the wound

649. CS. Wound epithelization begins:


a) From the middle of wound
b) From the depth of wound
c) From the edges of wound
d) From the area of wound with the best blood supply
e) From the entire surface of wound simultaneously

650. CM. Types of wound healing include:


a) Healing under scab
b) Primary deferred wound healing
c) Secondary wound healing
d) Primary wound healing
e) Early secondary wound healing

651. CM. Specify conditions, necessary for wound healing by primary intention.
a) Absence of infection in the wound
b) Significant microbial contamination of the wound
c) Minimal dehiscence of the wound borders
d) Absence in the wound of hematomas, foreign bodies and necrotic tissues
e) The extensive skin defect of a complex shape

652. CS. Uncomplicated healing of surgical wound occurs:


a) By healing under scab
b) By secondary wound healing
c) By primary wound healing
d) By early secondary wound healing
e) By primary deferred wound healing

653. CS. Which of these wounds do not heal by secondary healing?


a) Surgical aseptic sutured wound
b) Gunshot wound
c) Bite wound
d) Purulent wound
e) Contusioned wound

654. CS. How do superficial wounds usually heal?


a) By secondary wound intention
b) By primary deferred wound intention
c) By healing under scab
d) By primary wound intention
e) By early secondary wound intention

655. CM. What complications occur in the first phase of wound healing process (phase of inflammation)?
a) Traumatic shock
b) Eventration
c) Dehiscence of wound edges
d) Formation of keloid scar
e) Bleeding

656. CS. To local complication of wounds refers:


a) Hemorrhagic shock
b) Traumatic shock
c) Sepsis
d) Suppuration of wound
e) Syndrome of intoxication

657. CM. What complications occur in the third phase of wound healing process (phase of epithelization and
reorganization of scar)?
a) Hemothorax
b) Eventration
c) Wound cachexy
d) Wound sepsis
e) Keloid scar

658. CM. Which of the folloing refers to the measures of first aid for wounds?
a) Immobilization of injured extremity
b) Primary surgical debridement of the wound
c) Application of aseptic bandage on the wound
d) Administration of antibiotics
e) Application of hemostatic tourniquet

659. CM. In case of cut wound of the lower third of the calf with an external arterial bleeding first medical aid
should include:
a) Application of aseptic bandage
b) Transport immobilization
c) Administration of analgesics
d) Application of hemostatic tourniquet on the middle third of the calf
e) Application of hemostatic tourniquet on the hip

660. CM. How can secondary wound contamination be prevented during the first medical aid?
a) Administration of antibiotics
b) Application of aseptic bandage
c) Processing with antiseptic of skin surrounding the wound
d) Drainage of wound by gauze, imbibed with hypertonic saline solution
e) Washing of wound

661. CS. In case of a large wound of the anterior abdomen with evisceration of inner organs, the first medical aid
consists in:
a) Antibiotics administration
b) Large aseptic dressing
c) Clean the skin surrounding the wound with any antiseptic
d) Immediate suturing of the abdominal wound
e) Clean the eviscerated inner organs with any antiseptic

662. CM. Describe the main components of the aseptic (surgical) wounds treatment.
a) Early and adequate primary debridement of the wound
b) Adequate anesthesia
c) Prevention of secondary contamination of the wound
d) Placing of secondary sutures on the wound
e) Placing of primary sutures on the wound

663. CS. In the treatment of surgical wounds pain is eliminated by:


a) Sterilization of surgical field
b) Suturing of wound
c) Wound drainage
d) Administration of antibiotics
e) Anesthesia

664. CS. In the treatment of surgical wounds dehiscence of borders is eliminated by:
a) Incision making along the lines of Langer
b) Use of sharp scalpel and scissors
c) Suturing of wound
d) Use of retractors
e) Very fast performing the operation

665. CM. Primary surgical debridement (processing) of contaminated wound includes:


a) Radiation of wound with ultraviolet rays
b) Drainage of wound
c) Excision of necrotic tissues
d) Removal of foreign bodies from the wound
e) Exploration of wound channel

666. CS. At what stage of the primary surgical processing of wound can be established, if the wound is penetrate
or does not penetrate in the body cavities?
a) During dissection of wound
b) During exploration of wound channel
c) During excision of wound edges and borders
d) During excision of wound bottom
e) During drainage of wound

667. CM. What are the options for the completion of primary surgical debridement (processing) of contaminated
wound?
a) Closure of the wound with free perforated skin graft
b) The wound is left opened
c) Suturing of the wound
d) Suturing and drainage of the wound
e) Closure of the wound with full-thickness skin graft on vascular pedicle, using microsurgical technique

668. CM. In what cases is recommended do not suture the wound after its primary surgical processing?
a) In patients with diabetes mellitus
b) In case of massive contamination of wound with soil
c) In case of localization of wound on the foot
d) In case of localization of wound on the back of the trunk
e) In patients with confirmed malignancy

669. CM. What types of wounds do not require the primary surgical processing?
a) Slash superficial wounds with smooth edges
b) Small facial wounds
c) Wounds with localization on the foot
d) Stab wounds
e) Accidental infected wounds in patients with diabetes mellitus

670. CM. What disadvantages are characteristic of secondary wound healing?


a) Loss of fluid, protein and electrolytes
b) Formation of deforming scar
c) Inhibition of granulation tissue growth
d) Frequent adherence of anaerobic infection
e) Long process of healing

671. CS. Primary sutures are placed on the wound:


a) Before formation of scar tissue
b) After formation of scar tissue
c) After development of granulation tissue
d) After cleaning of the wound from necrotic tissues
e) Before development of granulation tissue

672. CS. Primary deferred sutures are placed on the wound:


a) On 5-6 days after primary surgical debridement of the wound, prior to the growth of granulation tissue
b) Immediately after primary surgical debridement of the wound
c) After development of granulation, but prior to formation of scar tissue
d) After excision of granulation tissue in the wound
e) After development of scar tissue and phenomenon of wound contraction

673. CS. Early secondary sutures are placed on the wound:


a) Before development of granulation tissue
b) After primary surgical debridement of the wound
c) After development of granulation tissue, but before formation of scar tissue
d) After excision of granulation tissue, which is developed in the wound
e) After formation of scar tissue and phenomenon of wound contraction

674. CS. Late secondary sutures on the wound are placed:


a) Before development of granulating tissue
b) After primary surgical debridement of wound
c) After development of granulating tissue, but before formation of scar tissue
d) After secondary surgical debridement of wound
e) After formation of scar tissue and phenomenon of wound contraction
675. CS. What type of suture is applied after primary surgical debridement of gunshot wound?
a) Primary suture
b) Primary deferred suture
c) Early secondary suture
d) Late secondary suture
e) Sutures are not placed

676. CM. Specify principles of surgical debridement of purulent wounds.


a) Primary sutures to the wound are not placed
b) Placement of primary sutures to the wound
c) Excision of devitalized tissues
d) Excision of granulation tissue
e) Drainage of wound by gauze tampons

677. CM. Specify additional physical methods of purulent wound cleaning.


a) Pulsatile jet with antiseptics
b) Use of surgical laser
c) Ultrasound cavitations
d) Treatment in controlled abacterial environment
e) Secondary surgical debridement of wound

678. CM. Specify medications that are recommended for the treatment of purulent wounds in the first phase of
wound healing process.
a) Proteolytic enzymes
b) Hydrophilic water-soluble ointments
c) Fat-soluble ointments
d) Solution of Chlorhexidinum
e) Immunostimulating ointments

679. CS. What is the purpose of local application of bandages with hypertonic saline solution (10% solution of
NaCl) in the first phase of wound healing process?
a) It causes lysis of necrotic tissue
b) Acceleration of wound exudates outflow
c) Protection of granulating tissue from damage
d) Stimulation of regeneration
e) Antimicrobial action

680. CM. What are the advantages of local application of hydrophilic water-soluble ointments on polyethylenglycol
basis (Levosin, Levomikol) for the treatment of septic wounds?
a) Their therapeutic effect lasts for 20-24 hours, therefore only one dressing per day is enough
b) It causes lysis of necrotic tissue and accelerates the wound healing
c) Its composition contains antibiotics penetraiting easily into the wound
d) Their osmotic activity lasts for 4-8 hours
e) Their osmotic activity is 10-15 times higher, than that of hypertonic saline solution

681. CS. Which of the following is used for early lysis and removal of necrotic tissue from the wound:
a) Antibiotics
b) Solution of hydrogen peroxide
c) Water-soluble ointments (Levosin, Levomikol)
d) Proteolytic enzymes
e) Boric acid

682. CM. What is the purpose of local application of ointments in the second phase of wound healing process?
a) To protect granulation tissue from damage
b) To stimulate growth of granulation tissue
c) To activate lysis of necrotic tissues
d) To improve outflow of exudate from the wound
e) To decrease local pain

683. CM. Specify medications that are recommended for the treatment of purulent wounds in the second phase of
wound healing process.
a) Proteolytic enzymes
b) Hydrophilic water-soluble ointments
c) Fat-soluble ointments
d) Alcohol
e) Solcoseril and Actoveghin ointments

684. CS. Which of the following is used commonly for local treatment of wounds with already formed granulation
tissue?
a) Dressing with hypertonic saline solution
b) Dressing with ointments
c) Dressing with antibiotics
d) Proteolytic enzymes
e) Dressing with hydrogen peroxide solution

685. CM. Which of the following is done in the appearance of signs of wound suppuration?
a) Application of ice on the wound
b) Removal of sutures from the wound
c) Exploration of the wound
d) Drainage of the wound with gauze tampon, imbibed with hypertonic saline solution
e) Placement of additional sutures on the wound

686. CM. To acquired deformities of the chest refers:


a) Emphysematous chest
b) Paralytic chest
c) “Boat” chest
d) “Bird chest” (pectus carinatum)
e) Pectus excavatum

687. CM. To congenital deformities of the chest refers:


a) “Bird chest” (pectus carinatum)
b) Pectus excavatum
c) Emphysematous chest
d) Poland’s syndrome
e) Paralytic chest

688. CS. The most common congenital deformity of the chest wall is:
a) “Bird chest” (pectus carinatum)
b) “Boat” chest
c) Pectus excavatum
d) Poland’s syndrome
e) Sternal fissure

689. CM. What characterizes pectus excavatum?


a) Body of the sternum is displaced posteriorly to produce a funnel-shaped depression
b) The depression is centered at the xiphisternal junction
c) The protrusion deformities of the sternum
d) V-shaped cleft of the upper part of sternum
e) Unilateral absence of the breast and nipple

690. CS. In pectus excavatum the depression is centered commonly at the:


a) Xiphisternal junction
b) Upper third of the sternum
c) Manubrium of the sternum
d) Xiphoid process of the sternum
e) Middle third of the sternum

691. CM. What are the complaints of a patient with pectus excavatum?
a) Cosmetic deformity
b) Chest pain
c) Cough
d) Dyspnea
e) Arrhythmias

692. CS. In patients with pectus excavatum the severity of symptoms is determined by:
a) The distance between the sternum and xiphoid process
b) The angle between the sternum and xiphoid process
c) The level of asymmetry with a relatively large depression of the costal cartilages on the right and turn of the
sternum to the right
d) The distance between the sternum and the spine
e) The length of depression in centimeters

693. CM. What characterizes “bird chest” (pectus carinatum)?


a) Body of the sternum is displaced posteriorly to produce a funnel-shaped depression
b) Most often patients are asymptomatic
c) The protrusion deformities of the sternum
d) Is accompanied by chest pain, dyspnea and fatigue
e) The atrophy of chest muscles and asymmetry of clavicles and scapulas

694. CS. On side inspection of patients with “bird chest” (pectus carinatum) the deformity usually is maximal:
a) Above the nipple level
b) Below the nipple level
c) At the level of manubrium of the sternum
d) At the level of the second rib
e) At the level of the xiphisternal junction

695. CS. What are the complaints of a patient with “bird chest” (pectus carinatum)?
a) Dyspnea
b) Chest pain
c) Disturbances of feeding
d) Cosmetic deformity
e) Fatigue

696. CM. What forms of the sternal fissure are distinguished?


a) Superior sternal cleft
b) Middle sternal cleft
c) Inferior sternal cleft
d) Complete sternal cleft
e) Marginal sternal cleft

697. CS. In superior sternal fissure is revealed:


a) The total cleft of sternum along its entire length, with a large divergence of the edges in the upper third
b) The cleft is V-shaped and extends down to the level of the xiphisternal junction
c) The cleft is U-shaped and affects only manubrium of the sternum
d) The cleft is U- or V-shaped and extends down to the level of the fourth rib
e) The deformity and depression of the sternocostal cartilages
698. CM. What is the Poland’s syndrome?
a) Bilateral absence or hypoplasia of the major and minor pectoral muscles
b) Unilateral absence or hypoplasia of the major and minor pectoral muscles
c) Unilateral hypoplasia of soft fat tissue
d) Unilateral partial absence of the costal cartilages
e) Unilateral hypoplasia of the breast and nipple

699. CS. What is the Poland’s syndrome?


a) Congenital V-shaped defect of the sternum, which extends down to the level of the fourth rib
b) Congenital protrusion asymmetric deformities of the sternum and costal cartilages
c) Congenital unilateral hypoplasia of the pectoral muscles, soft fat tissue and breast
d) Acquired depression into the superior and medium part of the sternum, caused by syringomielia
e) Congenital posterior deviation of the body of sternum with depression and asymmetry of the chest

700. CS. What characterizes the Poland’s syndrome?


a) Congenital unilateral absence of the ribs and pectoral muscles
b) Congenital bilateral hypoplasia of the pectoral muscles, soft fat tissue and breast
c) Congenital unilateral atrophy of the chest muscles and asymmetry of clavicles and scapulas
d) Congenital bilateral absence of the breast and nipple in women
e) Congenital unilateral hypoplasia of the pectoral muscles, soft fat tissue and breast

701. CM. What characterizes the “barrel” chest?


a) Narrowing of the intercostals spaces
b) The neck muscles assist in inspiratory contraction
c) It is observed in patients with malnutrition
d) Extension of intercostal spaces
e) It is observed in persons with obesity

702. CM. What lung diseases can lead to the development of “barrel” chest?
a) Lung cancer
b) Chronic bronchitis
c) Pneumonia
d) Pneumosclerosis
e) Pulmonary tuberculosis

703. CM. What characterizes the paralytic chest?


a) It is observed in persons with obesity
b) It is observed in patients with lung tuberculosis
c) It is observed in patients with malnutrition
d) The clavicles and scapulas are situated asymmetrically
e) Atrophy of chest muscles is noted

704. CS. Emphysematous chest is also called:


a) Barrel chest
b) Cylindrical chest
c) Bird chest
d) Boat chest
e) Funnel-shaped chest

705. CS. How the inspection of patient is performed to determine lordosis and kyphosis of spine?
a) The patient is in standing position, the doctor is stands behind
b) The patient is in standing position, the doctor is at the side
c) The patient is in standing inclined forward, the doctor is stands behind
d) The patient is lying on his abdomen, the doctor stands on the right side
e) The patient is lying on his side, the doctor stands on the right side

706. CS. Concavity (anterior direction) of the spine is called:


a) Kyphosis
b) Lordosis
c) Kyphoscoliosis
d) Scoliosis
e) Gibbus

707. CS. Convexity (posterior direction) of the spine is called:


a) Kyphosis
b) Lordosis
c) Kyphoscoliosis
d) Scoliosis
e) Ankylosis

708. CM. Specify the physiological curvatures of the spine.


a) Cervical kyphosis
b) Thoracic lordosis
c) Lumar lordosis
d) Thoracic kiphosis
e) Cervical lordosis

709. CM. Specify the pathological anteroposterior spine deformities.


a) Scoliosis
b) Lumbar lordosis
c) Gibbus
d) Thoracic hyperkiphosis
e) Flattering of the spinal curvature – dorsum platum

710. CS. Flattering of the spinal curvature (dorsum platum) is often caused by:
a) Increase of the abdomen during pregnancy or obesity, as a compensatory measure
b) Osteoporosis developing with age
c) Protrusion of one or more vertebrae in their tuberculous lesions
d) Rotation of the vertebrae relative to each other
e) Muscle spasm in spinal disc herniation

711. CS. Lumbar hyperlordosis of the spine is caused by:


a) Osteoporosis developing with age
b) Increasing of the abdomen during pregnancy or obesity, as a compensatory measure
c) Muscle spasm in spinal disc herniation
d) Rotation of the vertebrae relative to each other
e) Protrusion of one or more vertebrae in their tuberculous lesions

712. CS. Thoracic hyperkyphosis of the spine is caused by:


a) Rotation of the vertebrae relative to each other
b) Protrusion of one or more vertebrae in their tuberculous lesions
c) Muscle spasm in spinal disc herniation
d) Osteoporosis developing with age particularly in women
e) Increase of the abdomen during pregnancy or obesity, as a compensatory measure

713. CS. What is gibbus?


a) Protrusion of one or more vertebrae
b) Rotation of the vertebrae relative to each other
c) Flattening of the normal line of the spine due to herniated disc
d) Accentuation of the thoracic spine convexity due to osteoporosis
e) Accentuation of normal lumbar lordosis

714. CM. What are the main causes of gibbus?


a) Muscle spasm in spinal disc herniation
b) Vertebral body fractures
c) Metastatic lesions of vertebrae
d) Tuberculosis of the spine (spondylitis)
e) Osteoporosis of the spine
715. CM. Specify the correct landmarks, which are identified on examination of patient with diseases of the spine
from behind.
a) On the level of the lower margins of scapula the spinous process of 7th cervical vertebra is situated
b) On the level of the upper margins of scapula the spinous process of 7th cervical vertebra is situated
c) A line drawn between the iliac crests crosses the spinal process of 2nd lumbar vertebra
d) A line drawn between the iliac crests crosses the spinal process of 4th lumbar vertebra
e) On the level of the upper margins of scapula is situated the spinous process of 3rd thoracic vertebra

716. CS. How is the lateral twist of spine called?


a) Ankylosis
b) Scoliosis
c) Kyphosis
d) Lordosis
e) Kyphoscoliosis

717. CM. How is the inspection of patient performed to determine the scoliosis?
a) The patient is in standing position, the doctor is stands behind
b) The patient is in standing position, the doctor is at the side
c) The patient is in standing inclined forward, the doctor is stands behind
d) The patient is lying on his abdomen, the doctor stands on the right side
e) The patient is lying on his side, the doctor stands on the right side

718. CM. Which of the following is characteristic of lateral tilt of the spine as a result of muscle spasm?
a) A vertical line dropped from the spinous process of the 1st thoracic vertebra passes through the gluteal cleft
b) A vertical line dropped from the spinous process of the 1st thoracic vertebra falls to one side of the gluteal
cleft
c) It appears in herniated disc of spine
d) It appears in shortening of one of the lower extremities
e) It usually occurs in children

719. CS. What are the causes of structural scoliosis?


a) Shortening of one of the lower extremities
b) Compression fracture of the spine
c) Lack of vitamin D in childhood
d) Spinal disc herniation
e) Intercostal neuralgia

720. CS. What is the most common cause of functional scoliosis?


a) Tuberculosis of the spine
b) Lack of vitamin D in childhood
c) Shortening of one of the lower extremities
d) Metastatic lesion of the spine
e) Spinal disc herniation

721. CM. Which of the following is characteristic for structural scoliosis?


a) Rotation of the vertebrae with deformity of the chest
b) With patient forward flexion scoliosis disappears
c) Scoliosis is identified better when the patient flexes forward
d) Is a compensatory measure in shortening of one of the lower limbs
e) If to compensate short leg with orthopedic shoes – scoliosis disappears

722. CM. For functional scoliosis is characteristic:


a) Rotation of the vertebrae with deformity of the chest
b) With patient forward flexion scoliosis disappears
c) Scoliosis is seen better when the patient flexes forward
d) Is a compensatory measure in shortening of one of the lower limbs
e) If to compensate short leg with orthopedic shoes – scoliosis disappears
723. CS. Palpation of the spine is performed:
a) With a palm
b) With a thumb
c) With a index finger
d) With all fingers of the hand, tight together
e) With a fist

724. CM. Specify some risk factors for breast cancer in women.
a) Late menopause
b) Absence of pregnancy
c) Gynecological dysfunction or pathology
d) Elderly age
e) Presence of tumor in patient’s mother or sisters

725. CM. Congenital anomalies of the breast are:


a) Amastia
b) Athelia
c) Polymastia
d) Gynecomastia
e) Mastopathy

726. CM. Inflammatory diseases of the breast include:


a) Acute nonlactational mastitis
b) Intraductal papilloma
c) Tuberculosis of the breast
d) Syphilis of the breast
e) Mastopathy (Reclus disease)

727. CM. Dyshormonal diseases of the breast include:


a) Aberrant breast
b) Mastopathy
c) Polythelia
d) Puerperal mastitis
e) Gynecomastia

728. CM. Which of the following diseases refers to anomalies of the breast?
a) Reclus disease
b) Polythelia, athelia
c) Polymastia, amastia
d) Macromastia (giant breast)
e) Aberrant breast

729. CM. What is characterized polythelia correctly?


a) Most often it is located in the axillae
b) It increases during lactation
c) Glandular tissue is absent
d) It is located along the “milk line”
e) Presence of one or more extra breasts with small nipple and areola

730. CM. Which of the following is characteristic of aberrant breast?


a) Most often it is located in the axillae
b) It increases during lactation
c) Glandular tissue is absent
d) It consists from functional glandular tissue
e) It is located along the “milk line”

731. CM. Inspection of the breast is performed in following positions of patient:


a) Lying on back, with her arms raised up
b) Lateral to doctor, with her arms at the sides
c) Face to doctor, with her hands pressed against the hips
d) Face to doctor, with her arms at the sides
e) Face to doctor, with her arms raised up

732. CS. When is it better to perform prophylactic physical examination of the breast?
a) Just before menses
b) During the 1-2 week after menses
c) During the menses
d) 1 week before menses
e) It is not important

733. CS. On examination of the breast the correct sequence is the following:
a) First, examine the patient in a sitting position with her arms raised over her head, then – with her arms on her
sides, and then – with her hands pressed against her hips
b) First, examine the patient in a sitting position with her hands pressed against her hips, then – with her arms
raised over her head, and then – with her arms on her sides
c) First, examine the patient in a sitting position with her arms at her sides, then – with her arms raised over her
head, an then – with her hands pressed against her hips
d) First, examine the patient in a sitting position with her hands pressed against her hips, then – with her arms
raised over her head, and then – with her arms on her sides
e) First, examine the patient in a lying position with her arms on her sides, then – with her arms raised over her
head, and then – with her hands pressed against her hips

734. CM. What is the purpose of breast examination, when woman press her hands against her hips?
a) When a cancer or its associated fibrous are attached to pectoral fascia, dumpling is become visible
b) Press of her hands against her hips makes more marked enlargement of the regional lymph nodes
c) Press of her hands against her hips causes the contraction of pectoral muscles
d) Press of her hands against her hips makes tumor immobile relative to the skin
e) If tumor grows into the mammary ducts, the bloody discharge from the nipple appear

735. CM. What signs can be noted during visual inspection of the breast?
a) Kőnig’s sign
b) “Orange peel” sign
c) Rashes or ulceration of the areola and nipple
d) Skin dimpling and nipple retraction
e) Skin color

736. CM. In which diseases during the inspection may be observed redness of the breast skin?
a) Acute mastitis
b) Inflammatory carcinoma (mastitis-like cancer)
c) Mastopathy
d) Paget’s disease
e) Fibroadenoma of the breast

737. CM. On inspection of the breast that is affected by inflammatory process, is noted:
a) Increased volume of the breast
b) Redness (hyperemia) of the skin
c) Cyanosis of the skin
d) Tuberosity of the skin
e) Retraction of the nipple

738. CM. Symptoms of contraction in case of the breast tumors include:


a) Unilateral flattering of the breast contour
b) Local skin dimpling
c) Unilateral depression of the nipple
d) Bilateral inversion of the nipples
e) Unilateral protrusion of the skin

739. CS. Unilateral nipple retraction is usually a sign of:


a) Adenocarcinoma of the breast
b) Congenital abnormality
c) Acute mastitis
d) Fibroadenoma (benign tumor) of the breast
e) Intraductal papilloma

740. CM. Adenocarcinoma of the breast is characterized by:


a) “Orange peel” sign
b) Bloody discharge from the nipple
c) Fluctuations
d) Skin dimpling and nipple retraction
e) Serous discharge from the nipple

741. CS. What should be suspected if the areola and nipple are covered with erosions and crusts?
a) Poland’s syndrome
b) Paget’s disease
c) Acute lactational mastitis
d) Reclus disease
e) Nonpuerperal galactorrhea

742. CM. Postpartum mastitis is also called:


a) Maternal mastitis
b) Non-puerperal mastitis
c) Lactational mastitis
d) Puerperal mastitis
e) Secondary mastitis

743. CS. Palpation of the breast is done in lying position of a patient with her arm rested over her head with all
fingers flat on the breast; compress the tissue gently in a rotary motion against the chest wall. This technique
is called:
a) The first moment of Kőnig’s sign
b) The second moment of Kőnig’s sign
c) The method of Velpeau
d) The method of Desault
e) The second moment of Velpeau’s method

744. CM. Palpation of the breast by method of Velpeau is performed according to the following rules:
a) The patient sits or stands with her arms down on her sides
b) Palpation is performed with all fingers flat on the breast
c) The patient in a lying position with her arm rested over her head
d) The breast tissue is compressed between two hands, which facilitates the identification of masses
e) Palpation is performed compressing the tissue in a rotary motion against the chest wall

745. CM. What are the purposes of palpation of the breast?


a) To identify “orange peel” sign
b) To establish tenderness
c) To reveal the consistency of tissue
d) To detect a mass
e) To detect hyperemia of the skin

746. CM. The purposes of palpation of the breast are:


a) To identify the size and symmetry of the breasts
b) To identify “orange peel” sign
c) To asses the consistency of tissue
d) To reveal changes of the breast contour
e) To detect subareolar nodules

747. CS. What pathological symptom can be identified both on inspection and palpation of the breast?
a) Mobility of mass
b) Eczema-like lesion of the areola
c) Skin dimpling
d) Flattering of the breast contour
e) Impairment of the normal consistency of tissue

748. CM. Specify the synonyms of the term “mastopathy”.


a) Cyst disease
b) Mastodynia
c) Schimmelbuch disease
d) Reclus disease
e) Paget’s disease

749. CM. The clinical presentation of mastopathy includes the following symptoms:
a) It is located more often in the lateral regions of the breast
b) Palpation reveals nodular consistency of the breast
c) Pain in the breast has a periodical character and increases 5-7 days before menses
d) Pain in the breast has a permanent character, disappearing only during menses
e) Palpation reveals multiple fluid collections, poor delimited with each other

750. CS. What occurs in case of mastopathy in the mammary gland?


a) Formation of scar tissue and deformation of the breast contour
b) Inflammatory tissue reaction, which has a continuous and progressive character
c) Increased lactic secretion, which leads to stagnation of milk in the ducts
d) Growth of firm connective tissue in the form of streaks
e) Appearance of cysts filled with transparent fluid

751. CS. What is the symptom of mastopathy, when palpable mass is in upper right position disappears in
repeated palpation in a lying position?
a) Velpeau sign
b) Kőnig sign
c) Paget sign
d) Schimmelbuch sign
e) Reclus sign

752. CS. How are called the sectors, into which the mammary gland is divided conventionally?
a) Regions
b) Segments
c) Zones
d) Quadrants
e) Departments

753. CM. Which characteristics are obligatory in the description of palpable breast mass?
a) Unusually prominent skin pores
b) Shape
c) Tenderness
d) Number of nodules (masses)
e) Delimitation in relationship with surrounding tissues

754. CM. Which characteristics are obligatory in the description of palpable breast mass?
a) Location
b) Consistency
c) Mobility
d) Defect of filling
e) Size in centimeters

755. CS. Breast tumor is attached to the ribs and intercostal muscles, if on palpation:
a) The mass is mobile with respect to surrounding tissues, but it is fixed in relation to skin
b) The mobile mass becomes fixed when the patient presses her hand against her hip
c) The mass is mobile on palpation in lying and upright position of patient
d) The mass remains immobile even with relaxed pectoral muscles
e) The signs of skin retraction become more evident

756. CS. Breast tumor is attached to the pectoral fascia, if on palpation:


a) The mass is mobile with respect to surrounding tissues, but is fixed in relation to skin
b) The mobile mass becomes fixed when the patient presses her hand against her hip
c) The mass is mobile on palpation in a lying and upright position of patient
d) The mass remains immobile even with relaxed pectoral muscles
e) The signs of skin retraction become more evident

757. CS. Sometimes palpation of the breast may detect a mass with its usual subareolar location, which is called:
a) Benign fibroadenoma
b) Adenocarcinoma
c) Fibrocystic mastopathy
d) Paget’s disease
e) Intraductal papilloma

758. CM. Nonpuerperal galactorrhea is characterized by the following features:


a) It is associated with hormonal disorders
b) Cystic extension of the milk ducts with surrounding inflammatory reaction
c) Milky discharge from the nipple, persisting for long periods after normal lactation
d) It is a malignant process
e) Milky discharge from the nipple unrelated to a prior pregnancy or lactation

759. CM. With the presence of bloody discharge from the nipples should be suspected:
a) Gangrenous form of mastitis
b) Intraductal papilloma
c) Malignant tumor
d) Benign fibroadenoma
e) Fibrocystic mastopathy

760. CS. In case of purulent discharge from the nipple should be suspected:
a) Acute mastitis
b) Intraductal papilloma
c) Inflammatory carcinoma of the breast
d) Tuberculosis of the breast
e) Fibrocystic mastopathy

761. CS. Central axillary lymph nodes are located:


a) Along the lateral border of the scapula and are felt deep in the posterior axillary fold
b) Along and posteriorly the border of the pectoral major muscle
c) In infraclavicular region on the same side
d) Along the upper humerus
e) In the axillae and midway between the anterior and posterior axillary folds

762. CM. Specify the optimal position of patient during palpation of central axillary lymph nodes.
a) Patient is in a seating position
b) Patient’s arm is folded down and completely relaxed
c) Patient's hand is slightly taken aside
d) Patient is in a lying position
e) Patient’s arms are raised up over her head

763. CM. If there is a suspicious on the breast mass, and the central axillary lymph nodes fell large, hard, or
tender, the following groups of lymph nodes should be detected:
a) Pectoral (anterior) lymph nodes
b) Subscapular (posterior) lymph nodes
c) Infraclavicular lymph nodes
d) Parasternal lymph nodes
e) Submandibular lymph nodes
764. CM. Malignant breast tumors metastasize to the:
a) Brain
b) Liver
c) Opposite breast
d) Spleen
e) Uterus

765. CM. Rules of the breast self examination are the following:
a) Women in menopause it should be performed monthly
b) It should include a systematic description of masses and nodules
c) In women of childbearing age is best done immediately after menstruation
d) It should include examination in a mirror
e) It should include palpation both in upright and supine positions

766. CM. Specify the imaging and instrumental methods in suspicious on breast diseases.
a) Mammography
b) Intraductal endoscopy
c) Thermography
d) Ultrasonography
e) Scintigraphy

767. CS. What diagnostic test is used for screening for early detection of breast cancer?
a) Mammography
b) Thermography
c) Ultrasonography
d) Determination of the oncological markers levels
e) Fine needle aspiration and biopsy

768. CS. Pathological enlargement of the breast in men is called:


a) Mastopathy
b) Macromastia (giant breast)
c) Asymmetry of the breast
d) Gynecomastia
e) Fibroadenomathosis of the breast

769. CS. What is gynecomastia?


a) Reduction of the breast in men
b) Enlargement of the breast in men
c) Absence of the breast in men
d) Presence of the additional breast in men
e) Presence of additional nipple and areola in men

770. CM. The main causes of gynecomastia are the following:


a) Klinefelter syndrome
b) Obesity
c) Pharmacologic
d) Idiopathic
e) Liver failure

771. CS. The method of choice for treatment of idiopathic gynecomastia is:
a) Diet therapy (correction of obesity)
b) Radiation therapy
c) Chemotherapy
d) Hormone therapy
e) Surgical removal

772. CM. In men breast cancer:


a) Is a genetically conditioned disease
b) Develops from rudimentary ductal elements
c) Develops in the presence of hormonal disorders
d) Develops from the glandular tissue
e) Develops because of liver failure

773. CM. Symptoms of breast cancer in men are the following:


a) Fluctuation
b) Retraction of the nipple and skin
c) History of surgery for gynecomastia
d) Tumor of the areola or nipple
e) Discharge from the nipple ulceration

774. CS. The term “acute abdomen” is:


a) Symptom
b) Final diagnosis
c) Syndrome
d) Postoperative diagnosis
e) All of the above are correct

775. CM. The diagnosis of “acute abdomen” is:


a) An indication for urgent surgery
b) An indication for admission to a therapeutic department
c) An indication for consultation of surgeon
d) A reason to use additional diagnostic measures
e) A reason for discharge of patients from hospital

776. CS. Which of the following statements is not true?


a) “Acute abdomen” is a general term, meaning a severe abdominal pain in a patient
b) “Acute abdomen” is a syndrome and at the same time - a preliminary diagnosis
c) “Acute abdomen” means the need for prompt diagnosis
d) “Acute abdomen” means the invariable need for surgery
e) “Acute abdomen” means the need for early treatment

777. CM. Which of the following diseases, given rise to the symptoms of “acute abdomen” have an inflammatory
nature?
a) Acute appendicitis
b) Strangulated hernia
c) Acute pancreatitis
d) Ruptured tubal gestation
e) Perforated ulcer

778. CM. Which of the following diseases, causing the clinical picture of “acute abdomen”, does not relate to
inflammatory ones?
a) Acute cholecystitis
b) Strangulated hernia
c) Acute pancreatitis
d) Thrombosis of mesenterial vessels
e) Acute appendicitis

779. CM. Which of the following conditions are accompanied by perforation of a hollow organ into the peritoneal
cavity?
a) Spontaneous rupture of abdominal esophagus
b) Duodenal perforated ulcer
c) Thrombosis of mesenterial vessels
d) Ruptured tubal gestation
e) Penetrated injury of small bowel

780. CM. Which of these conditions does not refer to perforation of a hollow organ into the peritoneal cavity?
a) Pathological communication between the stomach and colon
b) Pathological communication between the stomach and abdominal cavity
c) Pathological communication between the duodenum and retroperitoneal space
d) Pathological communication between the urinary bladder and abdominal cavity
e) Bleeding gastric ulcer

781. CM. Which of the diseases, causing the clinical picture of “acute abdomen”, is related to the syndrome of
“acute intestinal obstruction”?
a) Small bowel obstruction due to adhesions
b) Strangulated hernia
c) Intussusception
d) Pyloroduodenal stenosis due to peptic ulcer
e) Volvulus of sigmoid colon

782. CM. Which of the following diseases is accompanied by intraperitoneal hemorrhage?


a) Gastric ulcer bleeding
b) Ruptured aortic aneurysm
c) Ruptured tubal gestation
d) Uterine bleeding
e) Traumatic rupture of spleen

783. CM. Which of the following diseases is not accompanied by intraperitoneal hemorrhage?
a) Bleeding gastric ulcer
b) Ruptured aortic aneurysm
c) Ruptured tubal gestation
d) Uterine bleeding
e) Traumatic rupture of spleen

784. CM. Which of the following extraabdominal diseases can cause the clinical picture of “acute abdomen”?
a) Coarctation of aorta
b) Pneumonia
c) Systemic vasculitis
d) Noncompensated diabetes mellitus
e) Alimentary tract infections

785. CM. Specify the management options for patients, who presented with “acute abdomen”.
a) Observation at home (out-hospital setting)
b) Immediate operation
c) Preoperative preparation and then – operation
d) Conservative treatment
e) Observation in surgical department (in-hospital setting)

786. CS. Which of the following diseases is typical for children?


a) Perforated ulcer
b) Cancerous large bowel obstruction
c) Acute pancreatitis
d) Ruptured tubal gestation
e) Intussusception

787. CM. Which of the following diseases is not typical for pediatric patients?
a) Perforated ulcer
b) Cancerous large bowel obstruction
c) Acute pancreatitis
d) Acute appendicitis
e) Intussusception

788. CS. Which of the following diseases is typical for patients older than 60 years?
a) Cancerous large bowel obstruction
b) Ruptured tubal gestation
c) Acute pancreatitis
d) Intussusception
e) Perforated ulcer

789. CM. Which of the diseases is characteristic for middle-aged patients?


a) Acute pancreatitis
b) Cancerous large bowel obstruction
c) Perforated ulcer
d) Ruptured tubal gestation
e) Intussusception

790. CS. Which of the following diseases is characteristic for middle-aged patients?
a) Acute appendicitis, acute cholecystitis, intussusception
b) Cancerous large bowel obstruction, acute appendicitis, adnexitis
c) Strangulated hernia, perforated ulcer, cancerous large bowel obstruction
d) Ruptured Graafian follicle, intussusception, ruptured tubal gestation
e) Perforated ulcer, acute pancreatitis, ruptured tubal gestation

791. CS. In visceral pain irritation from the gastrointestinal organs is distributed by:
a) Parasympathetic nerve fibers
b) Perilymphatic nerve plexus
c) Parasympathetic and sympathetic nerve fibers
d) Perivenous nerve plexus
e) Celiac nerve fibers

792. CS. In somatic pain irritation from the gastrointestinal organs is distributed by:
a) Parasympathetic and sympathetic nerve fibers
b) Parasympathetic nerve fibers
c) Perivenous nerve plexus
d) Perilymphatic nerve plexus
e) Sympathetic nerve fibers

793. CM. What characterizes visceral pain in “acute abdomen”?


a) It is the result of distention or spasm of organ
b) It arises from the parietal peritoneum
c) It has a diffuse character
d) It can be accurately localized by patient
e) It can not be accurately localized by patient

794. CM. What is the character of visceral pain in “acute abdomen”?


a) It is diffuse
b) It is felt by patients in the form of compression, cramps and colic
c) It is intensive and continuous
d) It is strictly limited and localized
e) It increases with movements

795. CM. What characterizes somatic pain in “acute abdomen”?


a) It has an intensive and continuous character
b) It arises from the parietal peritoneum
c) It arises from the gastrointestinal organs
d) It is the result of distention or spasm of organ
e) It is strictly limited and localized

796. CM. What is the character of somatic pain in “acute abdomen”?


a) It increases with movements
b) It is felt by patients in the form of pressure, cramps and colic
c) It is strictly limited and localized
d) It is diffuse
e) It is intensive and continuous
797. CM. In which of the following conditions abdominal pain has a visceral nature?
a) Blumberg symptom
b) Kocher’s symptom
c) Acute cholecystitis with local peritonitis
d) Renal colic
e) Biliary colic

798. CS. For descriptive purposes the abdomen is divided into:


a) 2 parts, 6 regions
b) 3 parts, 9 regions
c) 3 parts, 6 regions
d) 2 parts, 9 regions
e) 3 parts, 3 regions

799. CM. The epigastric part of abdomen includes:


a) Epigastric region itself
b) Right hypochondrium
c) Left hypochondrium
d) Periumbilical region
e) Right iliac region

800. CM. The mezogastric part of abdomen includes:


a) Suprapubic region
b) Right flank
c) Left flank
d) Periumbilical region
e) Iliac region

801. CM. The hypogastric part of abdomen includes:


a) Epigastric region itself
b) Suprapubic region
c) Periumbilical region
d) Left iliac region
e) Right iliac region

802. CM. In which abdominal disease patients may indicate the exact time of the onset up to a minute?
a) Acute appendicitis
b) Strangulated hernia
c) Perforated ulcer
d) Ruptured tubal gestation
e) Acute adnexitis

803. CM. Which of the following diseases are characterized by a gradual increase of abdominal pain intensity?
a) Rupture of abdominal aortic aneurysm
b) Acute appendicitis
c) Perforated ulcer
d) Cancerous intestinal obstruction
e) Acute adnexitis

804. CS. Which of the following diseases usually starts after weightlifting or abrupt change of body position?
a) Rupture of abdominal aortic aneurysm
b) Cancerous intestinal obstruction
c) Perforated ulcer
d) Strangulated hernia
e) Rupture of spleen

805. CM. What variants of pain are typical for acute abdominal surgical diseases?
a) “Knife-like” pain
b) Constant and progressive pain
c) Nighttime pain
d) Gripping pain
e) Episodic pain

806. CS. Which of the following diseases is accompanied by a “knife-like” pain in the abdomen?
a) Invasion of parenchymatous organ tumor into the surrounding tissue
b) Perforated gastric ulcer
c) Thrombosis of mesenterial arteries
d) Necrotic process in the abdominal cavity
e) Severe alimentary tract infection

807. CM. Where is abdominal pain caused by disorders of the small intestine initially localized?
a) In the epigastric area
b) In the right hypochondrium
c) In the suprapubic region
d) In the periumbilical area
e) In the inguinal zone

808. CS. Shifting of pain from the epigastrium to the right iliac fossa suggests:
a) Acute adnexitis
b) Intestinal obstruction
c) Acute appendicitis
d) Strangulated hernia
e) Acute cholecystitis

809. CS. Abdominal cramps are typical for:


a) Pyelonephritis
b) Acute appendicitis
c) Acute pancreatitis
d) Intestinal obstruction
e) Acute adnexitis

810. CS. “Knife-like” pain in the abdomen is characteristic of:


a) Acute appendicitis
b) Dissecting abdominal aortic aneurysm
c) Intestinal obstruction
d) Acute cholecystitis
e) Perforated ulcer

811. CM. Radiation of pain in the inferior angle of the right scapula is characteristic in:
a) Traumatic rupture of spleen
b) Dissecting aortic abdominal aneurysm
c) Biliary colic
d) Acute cholecystitis
e) Perforated ulcer

812. CM. For disease of what organs is characteristic radiation of pain back into sacral area?
a) Rectum
b) Uterus
c) Small bowel
d) Pancreas
e) Kidneys and ureters

813. CM. Radiation of pain to the top of the shoulder on the side of lesion occurs in:
a) Traumatic rupture of spleen
b) Acute cholecystitis
c) Dissecting abdominal aortic aneurysm
d) Perforated ulcer
e) Ruptured pyosalpinx
814. CS. “Like a belt” radiation of abdominal pain is characteristic of:
a) Perforated ulcer
b) Dissecting abdominal aortic aneurysm
c) Intestinal obstruction
d) Acute cholecystitis
e) Acute pancreatitis

815. CS. In which pathology pain intensity is so high that the patient does not find a place (restlessness)?
a) Perforated ulcer
b) Renal colic
c) Acute appendicitis
d) Intestinal obstruction
e) Ruptured tubal gestation

816. CM. In which diseases abdominal pain will be increased on deep inspiration?
a) In subphrenic abscess
b) In acute cholecystitis
c) In pleuritis
d) In acute appendicitis
e) In renal colic

817. CM. In patients with “acute abdomen” vomiting is caused by:


a) Severe irritation of the nerves of the peritoneum and mesentery
b) Syndrome of increased intracranial pressure
c) Mechanical obstruction of intestine
d) Reflex mechanisms - in obstruction of an involuntary muscular tubes (the biliary ducts, ureter, intestine, or
vermiform appendix)
e) External factors

818. CS. What is the character of vomiting in acute appendicitis?


a) Frequent and copious vomiting
b) Single reflex vomiting
c) “Feculent” vomit
d) Copious, with stagnant gastric contents
e) “Coffee grounds” contents

819. CM. In which acute abdominal surgical diseases is absent vomiting?


a) Acute appendicitis
b) Acute cholecystitis
c) Ruptured tubal gestation
d) Small bowel obstruction
e) Perforated gastric and duodenal ulcer

820. CM. In which acute abdominal surgical diseases vomiting is not characteristic?
a) Spleen rupture
b) Large bowel obstruction
c) Small bowel obstruction
d) Acute pancreatitis
e) Acute appendicitis

821. CM. Which acute abdominal surgical diseases are accompanied by repeated vomiting?
a) Acute cholecystitis
b) Perforated gastroduodenal ulcer
c) Proximal small bowel obstruction
d) Acute pancreatitis
e) Acute appendicitis

822. CM. Frequent and bilious vomiting is typical for:


a) Acute cholecystitis
b) Large bowel obstruction
c) Biliary colic
d) Ruptured tubal gestation
e) Acute pancreatitis

823. CS. In what illness does “feculent” vomit occur?


a) Large bowel obstruction
b) Small bowel obstruction
c) Bleeding from colon polyps
d) Acute pancreatitis
e) Pyloroduodenal stenosis due to peptic ulcer

824. CS. The absence of passage of stool and gas is an early symptom of:
a) Large bowel obstruction
b) Small bowel obstruction
c) Advanced peritonitis
d) Acute pancreatitis
e) Ruptured tubal gestation

825. CS. In patients with “acute abdomen” thirst is a sign of:


a) Acute anemia
b) Hypotension
c) Dysbacteriosis
d) Portal hypertension
e) Dehydration

826. CM. In what acute surgical diseases of the abdominal organs are especially characteristic thirst and
xerostomia?
a) Advanced peritonitis
b) Severe acute pancreatitis
c) Appendiceal colic
d) Ruptured spleen with abdominal bleeding
e) Acute intestinal obstruction

827. CM. “Hippocratic face” in a patient with advanced peritonitis is characterized by a combination of the
following symptoms:
a) Gray color of skin
b) Sunken orbits
c) Pale-cyanotic color of skin
d) Accentuated features of his face
e) Skin covered with cold sweat

828. CS. Which of the following combinations of symptoms characterizes the “Hippocratic face” in a patient with
advanced peritonitis?
a) Pale color of skin, cold sweat, superficial respiration
b) Cyanotic color of skin, grimace of pain on his face, dry lips
c) Jaundice, cold sweat, edema of the face
d) Gray color of skin, restlessness, grimace of pain on his face
e) Gray color of skin, sunken orbits, accentuated features of his face

829. CS. Which of the given diseases is characterized by “tilting doll” symptom?
a) Ruptured spleen or liver
b) Advanced peritonitis
c) Acute pancreatitis
d) Acute intestinal strangulation
e) Ruptured tubal gestation

830. CS. When is the "tilting doll" symptom positive?


a) The patient lies on his back, attempts to turn over a patient on his side are associated with severe pain and
patient just holds previous position
b) The patient lies on his side, attempts to turn over a patient on his back are associated with severe pain and
patient just holds previous position
c) The patient prefers to sit or stand, trying to put the patient on his back are associated with severe pain, and
patient just holds previous position
d) The patient lies on his back, the appearance of cramping abdominal pain forces patient turn on his side
e) The patient lies on his side, the appearance of cramping abdominal pain forces patient turn on his back

831. CM. In what acute abdominal diseases in patients is often seen jaundice?
a) Ruptured tubal gestation
b) Acute cholecystitis
c) Acute pancreatitis
d) Cholangitis
e) Thrombosis of mesenteric vessels

832. CM. Symmetric distension of the abdomen usually occurs in:


a) Large abdominal tumor
b) Obesity
c) Intestinal obstruction
d) Ascites
e) Uniform intestinal distension by gas

833. CS. Asymmetric distension of the abdomen usually occurs in:


a) Inflammatory diseases of abdominal cavity
b) Obesity
c) Intestinal obstruction
d) Ascites
e) Alimentary tract infections

834. CS. In patients with tumors of the pancreatic head can be detected Courvoisier sign, which represents:
a) Increasing pain on deep palpation in the upper right part of abdomen
b) Sharp increasing of pain, when suddenly stop palpation of the upper right abdomen
c) Large-size tumor is contoured through the anterior abdominal wall in the epigastric region
d) Visible protrusion in the upper right abdomen, caused by an increasing volume of gallbladder
e) Attenuation of respiratory movements of the anterior abdominal wall in epigastric region

835. CM. Specify the correct position of patient during abdominal palpation.
a) Arms of patient are relaxed down along the sides of the body
b) Patient is lying on his back
c) Legs of patient are slightly flexed in knees
d) Arms of patient are raised above the head
e) Patient is lying on his side

836. CS. On abdominal palpation the patient's position should be:


a) Lying on his side, with his arms – relaxed down along the sides of the body, and legs – extended
b) Lying on his back, with his arms – raised above the head, and knees – slightly flexed
c) Lying on his back, with his arms – raised above the head, and legs – extended
d) Lying on his back, with his arms – relaxed down along the sides of the body, and legs – extended
e) Lying on his back, with his arms – relaxed down along the sides of the body, and knees – slightly flexed

837. CM. What is the purposes of superficial palpation of the abdomen?


a) To determine the peritoneal signs
b) To determine free fluid in the abdomen
c) To determine the lower border of liver
d) To determine muscular rigidity
e) To determine the site of maximum pain
838. CS. What can be revealed during superficial palpation of the abdomen?
a) Free fluid in the abdomen and peritoneal signs
b) The site of maximum pain and muscular rigidity
c) The site of maximum pain and size of hernial ring
d) The site of maximum pain and peritoneal signs
e) The presence of mass and peritoneal signs

839. CS. Palpation of the abdomen begins with:


a) Superficial palpation
b) Deep palpation
c) Determination of Blumberg’s symptom
d) Determination of muscular rigidity
e) Detection of hernias

840. CS. Superficial palpation of the abdomen should begin:


a) From the left hypochondrium
b) From the right iliac fossa
c) From the epigastric area
d) From the part that the most distant from the point of maximum pain
e) From the part of maximum pain

841. CS. In patients with “acute abdomen” muscular resistance develops due to:
a) Hydro-electrolyte imbalances
b) Status of psycho-emotional excitation
c) Irritation of parietal peritoneum
d) Irritation of parietal pleura
e) Effects of bacterial toxins on the central nervous system

842. CS. Muscular resistance of the anterior abdominal wall is not characteristic in patients with:
a) Perforated ulcer
b) Intestinal obstruction by tumor
c) Acute appendicitis
d) Traumatic rupture of hollow organ
e) Acute cholecystitis

843. CS. In which pathology muscular tenderness of the anterior abdominal wall is so firm, continuous and
extended, that it is defined as "board-like" abdomen?
a) Cancerous large bowel obstruction
b) Perforated ulcer
c) Thrombosis of mesenterial vessels
d) Ruptured abdominal aortic aneurysm
e) Ruptured tubal gestation

844. CM. What extraabdominal diseases may be accompanied by musculare resistance of the abdominal wall?
a) Pneumonia with pleurisy
b) Inferior ribs fracture
c) Renal colic
d) Epileptic seizure
e) Hypertensive crisis

845. CM. In which clinical situations muscular resistance may be very slight even in the presence of serious
peritonitis?
a) In patients with severe neuropsychic diseases
b) In obese patients with fat and flabby abdominal wall
c) In patients with traumatic or hemorrhagic shock
d) In patients with severe alcohol or narcotic toxemia
e) In elderly patients

846. CS. Positive Blumberg’s symptom in case of peritonitis is determined by:


a) Percussion in the examined area causes severe pain
b) Flexion of extended leg in the hip joint causes severe pain
c) Gently palpation of the examined area determines muscular resistance, which is accompanied by severe pain
d) Fingers are pressed gently and deeply into the abdominal wall over the examined area and then the pressure
is suddenly withdraws, that causes severe pain
e) Fingers are pressed deeply into the abdominal wall over the examined area, that causes severe pain

847. CM. Choose two correct versions of the determination of psoas-symptom, which is revealed in retroperitoneal
inflammation?
a) Percussion in the lumbar region causes severe pain
b) Bimanual palpation of the lumbar region from the affected side causes severe pain
c) Flexion of extended leg in the hip joint causes severe pain
d) Patient is placed on his affected side and extension of the leg in the hip joint causes severe pain
e) Bimanual palpation of the lumbar region on the affected side causes an involuntary flexion of leg

848. CM. In which of the following diseases of the abdominal cavity can be detected diffuse dullness on abdominal
percussion?
a) Acute cholecystitis
b) Acute diverticulitis
c) Acute appendicitis
d) Liver cirrhosis with ascites
e) Severe advanced peritonitis

849. CM. In which acute surgical diseases of abdominal organs disappearance of liver dullness can be detected?
a) Large bowel obstruction
b) Acute cholecystitis with empyema of the gallbladder
c) Traumatic rupture of the liver
d) Ruptured tubal gestation
e) Perforated ulcer

850. CS. The presence of free fluid in the abdominal cavity is determined clinically by:
a) Abdominal auscultation
b) Abdominal palpation
c) Abdominal percussion
d) X-ray examination
e) Ultrasonography

851. CM. “Plash sound” appears on auscultation and percussion of the abdomen as a result of:
a) Presence of free fluid in the abdominal cavity
b) Decompensated stenosis of the pyloric part of the stomach
c) Hemoperitoneum
d) Intestinal obstruction
e) Presence of free air in the abdominal cavity

852. CS. Peristalsis of the intestine should be heard:


a) In the mezogastric paraumbilical area
b) In the epigastric area and in the flanks of abdomen
c) In the point, located just above the umbilicus on the left
d) In the right and left iliac regions
e) All answers are correct

853. CS. Aortic bruits in abdominal aortic aneurysm should be heard in:
a) The inguinal areas
b) The epigastric region
c) On the median line, extending from the epigastric area to the suprapubic zone
d) The right and left iliac regions
e) The point, located just above the umbilicus on the left

854. CS. In which of the following conditions may loud borborygmi (peristalsis) be determined by auscultation?
a) Local peritonitis
b) Advanced peritonitis
c) Thrombosis of mesenterial vessels
d) Intestinal obstruction
e) Intraperitoneal hemorrhage

855. CS. In which of the following diseases the intestinal peristalsis on auscultation is absent?
a) Local peritonitis
b) Advanced peritonitis
c) Rupture of abdominal aortic aneurysm
d) Intestinal obstruction
e) Decompensated liver cirrhosis with tense ascites

856. CM. What are the most likely causes of the disease in patient whith the onset of pain in the abdomen
accompanied by temperature of 40-41˚C?
a) Advanced peritonitis
b) Lung pathology
c) Gastrointestinal bleeding
d) Massive intraperitoneal hemorrhage
e) Kidney disease

857. CS. On admission the temperature of patients with acute appendicitis is usually:
a) Subnormal (35-36˚С)
b) Normal (36.4-36.6˚С)
c) Subfebrile (37.3-37.5˚С)
d) Febrile (38˚С)
e) Hectic with chills (40-41˚С)

858. CM. In which of the following acute abdominal diseases body temperature of patient may be subnormal (to
35-36˚С)?
a) In perforated ulcer
b) In severe intraperitoneal hemorrhage
c) At the onset of an attack of acute pancreatitis
d) In perforation of appendix and progression of peritonitis
e) In intestinal obstruction with severe abdominal distention

859. CS. The term “limb ischemia” means:


a. Increased arterial inflow in extremity
b. Pathologic communication between artery and vein
c. Decreased arterial inflow in extremity
d. Disturbed venous outflow in extremity
e. Inflammation of sciatic nerve

860. CM. The syndrome of “limb ischemia” is classified in:


a. Acute ischemia
b. Subacute ischemia
c. Chronic ischemia
d. Recurrent ischemia
e. Transitory ischemia

861. CM. Choose the possible causes of the development of acute limb ischemia.
a. Arterial trauma
b. Stenosis of the vessel lumen with an atherosclerotic plaque
c. Arterial embolism
d. Arterial thrombosis
e. Thrombophlebitis

862. CS. Peripheral arterial embolism is manifested with syndrome of:


a. Chronic venous insufficiency
b. Chronic ischemia
c. Acute ischemia
d. Disseminated intravascular coagulation
e. Aneurysmal dilatation of artery

863. CS. Specify the most frequent cause of the development of chronic limb ischemia.
a. Atherosclerosis
b. Thrombangitis obliterans (Buerger’s disease)
c. Nonspecific aortoarteritis (Takayasu’s disease)
d. Arterial malformation
e. Aneurysm of peripheral artery

864. CS. The term “arterial aneurysm” means:


a. Disturbed auto-regulation of vascular tone
b. Vascular kinks and coils
c. Arterial narrowing
d. Arterial dilatation by less than 50%
e. Arterial dilatation by more than 50%

865. CM. Arterial aneurysms are divided according to cause of development into:
a. Primary
b. Secondary
c. True
d. False
e. Combined

866. CM. Patient suffered a knife wound at the region of femoral artery. Three months later the pulsatile mass was
determined at the medial part of the thigh. Indicate the correct variants of diagnosis.
a. Primary aneurysm of femoral artery
b. False aneurysm of femoral artery
c. Pulsatile hematoma of the hip
d. True aneurysm of femoral artery
e. Posttraumatic aneurysm of femoral artery

867. CM. Specify the typical complications of arterial aneurysms.


a. Acute limb ischemia
b. Chronic venous insufficiency
c. Abscess formation
d. Rupture and bleeding
e. Recurrent pulmonary embolism

868. CM. Choose the diseases that are manifested by the syndrome of chronic venous insufficiency.
a. Phlegmasia cerulea dolens
b. Varicose veins of lower limbs
c. Acute deep vein thrombosis
d. Atherosclerosis of peripheral arteries
e. Postthrombotic disease

869. CM. Specify pathogenic mechanisms of chronic venous insufficiency of the lower limbs.
a. Total or partial occlusion of deep veins
b. Trauma or surgical excision of superficial veins
c. Venous valves incompetence and venous reflux
d. Hemoconcentration
e. Thrombocytosis

870. CS. Limb lymphedema is classified into:


a. Primary and secondary
b. Erytematous and bullous
c. True and false
d. External and internal
e. Superficial and deep

871. CS. The patient complains of repeated leg pain that develops during walking some distance. The pain is
severe enough to force the patient to stop walking and it disappears after a short rest. What symptom is
described?
a. Blumberg symptom
b. Symptom of “duck walking”
c. Symptom of intermittent claudication
d. Symptom of calf muscles weakness
e. Symptom of radiculopathy

872. CS. The patient complains of repeated leg pain that develops during walking some distance. The pain is
severe enough to force the patient to stop walking and disappear after a short rest. What syndrome should be
suspected?
a. Syndrome of acute ischemia
b. Syndrome of chronic ischemia
c. Acute venous thrombosis
d. Chronic venous insufficiency
e. Lymphedema

873. CM. The typical position of the patient with critical lower limb ischemia is:
a. Recumbent position in the bed with one elevated limb
b. Recumbent position in the bed with one limb lowered down on the floor
c. Lateral position in the bed with both limbs bended to the abdomen
d. Recumbent position in the bed with face down
e. Sitting position in the chair

874. CM. Choose the characteristics of rest pain caused by chronic lower limb ischemia:
a. The pain increases in sitting position of the patient
b. The pain awakes the patient during the nighttime
c. The pain is resistant to analgesics
d. The pain appears periodically
e. The pain decreases in sitting position of the patient

875. CS. Sensorial disorders (paresthesia, hypo- or anesthesia) and motor deficit (paresis, paralysis) are
characteristic of:
a. Syndrome of acute ischemia
b. Syndrome of chronic ischemia
c. Acute venous thrombosis
d. Chronic venous insufficiency
e. Lymphedema

876. CS. The symptom of “heavy legs” which develops at the end of working day and disappears after night rest is
characteristic of:
a. Syndrome of acute ischemia
b. Syndrome of chronic ischemia
c. Acute venous thrombosis
d. Chronic venous insufficiency
e. Arterial aneurysm

877. CM. What data from patient’s history of life are important for the diagnosis of limb ischemia?
a. Heavy smoking
b. Atrial fibrillation
c. Heavy work in an upright position
d. Erysipelas in the past
e. Deep wounds localized near the big arteries
878. CM. What comorbidities (concomitant diseases) are typical for patients with acute limb ischemia caused by
arterial embolism?
a. Type II diabetes mellitus
b. Atrial fibrillation
c. Varicose veins of lower limbs
d. Myocardial infarction
e. Abdominal aortic aneurysm

879. CM. What events from the history of patient’s life are important for diagnosis of secondary lymphedema of
extremities?
a. Mastectomy for breast cancer
b. Radiotherapy for malignant tumor
c. Smoking
d. Recurrent episodes of erysipelas
e. Prolonged immobilization (bed rest)

880. CS. On examination of lower limb varicose veins the correct position of the patient is:
a. Dorsal (face up) recumbent position
b. Ventral (face down) recumbent position
c. Lateral recumbent position
d. Sitting position
e. Upright position

881. CM. What is characteristic of edema caused by acute limb ischemia?


a. Edema develops immediately at the onset of disease
b. Edema decreases in a recumbent position
c. Edema involves the foot and shin only
d. Edema involves the whole extremity
e. Edema develops in the final stage of disease

882. CM. What is characteristic of edema caused by proximal (iliofemoral) deep vein thrombosis of lower limb?
a. Edema develops immediately at the onset of disease
b. Edema decreases in a sitting position
c. Edema involves the foot and shin only
d. Edema involves the whole extremity
e. Edema develops in the final stage of disease

883. CM. What is characteristic of edema caused by chronic venous insufficiency of lower limb?
a. Edema develops suddenly
b. Edema develops gradually
c. Edema involves the hip
d. Edema decreases in the recumbent position
e. Edema is associated with severe pain

884. CM. Büerger-Ratschow symptom in case of chronic ischemia consists in:


a. Foot hyperemia after elevation of extremity
b. Foot paleness after elevation of extremity
c. Foot hyperemia after lowering of extremity
d. Foot paleness after lowering of extremity
e. Diminished pulsation of foot arteries

885. CS. “Phlegmasia cerulea dolens” is:


a. The synonym of severe chronic limb ischemia
b. The typical symptom of arterial embolism
c. A complication of varicose veins
d. The most severe form of deep veins thrombosis
e. The final stage of lymphedema

886. CM. Specify clinical signs, which are characteristic of the final stage of acute limb ischemia:
a. Significant dilatation of subcutaneous veins
b. Severe hip edema and foot hyperemia
c. Evident paleness and hypothermia of the foot
d. Moderate calf edema
e. Foot paralysis and flexion knee contracture

887. CM. The following clinical signs are characteristic of damp gangrene of extremity:
a. Fetid smell
b. Tissue edema
c. Presence of “line of demarcation”
d. Fast spreading
e. Absence of intoxication and general reaction of organism

888. CM. Thrombosis of superficial varicose veins of lower limbs (varicothrombophlebitis) is characterized by:
a. Severe edema of hip and shin
b. Localized skin hyperemia above the vein with thrombosis
c. Veins filled with thrombus do not collapse after limb elevation
d. Severe pain in the foot and shin
e. Positive Homans symptom

889. CM. Syndrome of chronic venous insufficiency of lower limbs is characterized by:
a. Atrophy of calf muscles
b. Skin hyperpigmentation in the lower third of the shin
c. Absence of pulse on limb arteries
d. Edema of ankle (paramaleolar) region
e. Tendency to develop shin eczema

890. CM. What symptoms are not characteristic of the syndrome of chronic venous insufficiency of lower limb?
a. Development of ulcer on the medial surface of the lower third of the shin
b. Development of small, very painful foot ulcers
c. Lipodermatosclerosis of the shin
d. Phenomenon of “white skin atrophy”
e. Severe pain in the affected extremity during the nighttime

891. CM. Palpation of pulse on the brachial artery may be performed in the:
a. Axillary region, along the anterior line of hair growth
b. Sulcus brahi medialis (medial brachial intermuscular depression)
c. Cubital region, medial of the biceps tendon
d. Lateral region of supraclavicular fossa
e. In the depth of the deltoid muscle

892. CS. Palpation of pulse on the femoral artery is performed in the:


a. Point between the superior and middle thirds of line which joins the navel (umbilicus) and spina iliaca anterior
superior
b. Region of Hunter channel
c. Point localized 5 cm up to the superior edge of patella
d. Point localized between the external and middle thirds of the inguinal ligament
e. Point localized between the internal and middle thirds of the inguinal ligament

893. CM. Palpation of pulse on the popliteal artery is performed:


a. In the inferior third of the hip
b. In the superior third of the shin
c. Deep in the popliteal fossa
d. With one hand
e. With both hands

894. CS. In approximately 10% of healthy persons the pulse can not be palpated upon:
a. Radial artery
b. Brachial artery
c. Dorsal foot artery
d. Popliteal artery
e. Femoral artery

895. CM. In arteriovenous fistula the following symptoms can be determined by palpation:
a. Homans symptom
b. Branham symptom
c. Systolic-diastolic quiver (“cat’s purr”)
d. Trendelenburg symptom
e. Büerger-Ratschow symptom

896. CS. The Branham symptom is considered positive if:


a. Acute pain develops in the extremity during testing
b. The frequency of respiration increases during testing
c. The heart rate decreases during testing
d. The heart rate increases during testing
e. Severe edema develops during testing

897. CS. The group of symptoms known as “6Р” is characteristic of:


a. Acute limb ischemia
b. Chronic limb ischemia
c. Arterial aneurysm
d. Acute vein thrombosis
e. Chronic venous insufficiency

898. CM. Choose the typical symptoms of acute ischemia of lower limb.
a. Sensory deficit (lack of sensation) at the level of the foot
b. Intermittent claudication
c. Significant thigh edema
d. Motor deficit (limitation or lack of movements) at the level of the foot
e. Systolic bruit over the arteries of the foot

899. CM. Choose the signs that do not refer to the “6Р” group of symptoms.
a. Paresthesia
b. Pallor
c. Polyuria
d. Poikilocytosis
e. Pain

900. CS. The main danger related to thrombosis of the great saphenous vein is:
a. Inevitable development of the postthrombotic syndrome
b. Risk of pulmonary embolism
c. Need to remove the whole length of affected vein
d. Major risk of septic complications development
e. Possible gangrene development

901. CM. Specify the symptoms characteristic of deep vein thrombosis of lower limbs.
a. Branham symptom
b. Mozes symptom
c. Kocher symptom
d. Büerger-Ratschow symptom
e. Homans symptom

902. CS. In vascular semiology acute pain in the calf muscles during passive dorsal flexion of the foot is
considered as a positive:
a. Branham symptom
b. Homans symptom
c. Kocher symptom
d. Büerger-Ratschow symptom
e. Mozes symptom

903. CM. What tests are used for clinical demonstration of venous reflux in the great saphenous vein in patients
with varicose veins disease?
a. Hackenbruch test
b. Trendelenburg test
c. Velpaux test
d. Homans test
e. Delbet-Perthes test

904. CS. What clinical test can be used for assessment of permeability of the deep veins of lower limb before
varicose veins surgery?
a. Hackenbruch test
b. Trendelenburg test
c. Velpaux test
d. Homans test
e. Delbet-Perthes test

905. CM. Systolic bruit above big arteries suggests:


a. Arterial hypertension
b. Acute arterial thrombosis
c. Arterial stenosis
d. Arterial occlusion
e. Arterial aneurysm

906. CS. Systolic-diastolic permanent bruit (“engine sound”) over the major artery is characteristic of:
a. Peripheral arterial embolism
b. Acute arterial thrombosis
c. Critical stenosis of major artery
d. Arterial occlusion
e. Arterio-venous fistula

907. CS. Systolic bruit in the popliteal fossa should raise the suspicion to:
a. Acute thrombosis of popliteal artery
b. Critical stenosis of popliteal artery
c. Popliteal artery aneurysm
d. Popliteal artery embolism
e. Arterio-venous fistula in popliteal fossa

908. CM. Auscultation of which of the following arteries is useless in case of chronic ischemia of lower limbs?
a. Anterior tibial artery
b. Peroneal artery
c. Iliac artery
d. Femoral artery
e. Posterior tibial artery

909. CM. Specify the correct statements, relating to traumatic injuries.


a) It is the leading cause of death among people between 1 and 50 years of age
b) It takes the first place in mortality structure
c) It is the leading cause of death among people of all ages
d) It takes the second place in mortality structure
e) Men are affected by trauma two folds more often, than women

910. CS. About half of all deaths occur within seconds or minutes of injury and are related to:
a) Infectious complications
b) Lacerations of the aorta, heart, and brain
c) Persistent hemorrhage
d) Multiple organ failure
e) Development of traumatic disease
911. CS. In patients with severe trauma the “golden hour” is called:
a) The first hour after admission of patient
b) The first hour after surgery
c) The first hour after trauma
d) The first hour after correction of shock and stabilization of hemodynamics
e) The first hour after stop of hemorrhage

912. CS. During the second mortality peak (first hours after injury) the percentage of death, caused by trauma, is:
a) 20%
b) 30%
c) 40%
d) 50%
e) 60%

913. CM. During the first “golden hour” after severe injury by early treatment can be prevented many of deaths,
related to:
a) Injuries of the central nervous system
b) Injuries of extremities and pelvis bones
c) Multiple organ failure
d) Persistent hemorrhage
e) Infectious complications

914. CM. Which of the following factors are of main importance in biomechanics of blunt trauma?
a) Combined (mechanical and thermal) action of etiological agent
b) Compression of tissues during impact
c) Separation of tissues along the path of the penetrating object
d) Massive external bleeding
e) Changes in speed motion of the body (acceleration or deceleration)

915. CS. The term “katatrauma” means:


a) Industrial injury
b) Car crash accident
c) Injury as a result of falling from a height
d) Sportive injury
e) Injury as a result of medical manipulation

916. CS. The term iatrogenic injury means:


a) Industrial injury
b) Car crash accident
c) Injury as a result of falling from a height
d) Sportive injury
e) Injury as a result of medical manipulation

917. CM. In accordance to character of damage agent traumas are divided into:
a) Sportive trauma
b) Psychic trauma
c) Electrical trauma
d) Mechanical trauma
e) Chemical trauma

918. CS. What type of trauma is classified as multiple?


a) Injuries of two and more organs from one anatomical system
b) Injuries caused by action of two or more etiological factors
c) Two or more injuries of one organ
d) Simultaneous injuries of organs from different anatomical systems
e) Injury of only one organ

919. CS. What type of trauma is classified as associated (polytrauma)?


a) Injuries of two and more organs from one anatomical system
b) Injury of only one organ
c) Two or more injuries of one organ
d) Simultaneous injuries of organs from different anatomical systems
e) Injuries caused by action of two or more etiological factors

920. CM. Which of the following injuries are related to isolated trauma?
a) Solitary injury of one organ
b) Multiple injuries of one organ
c) Solitary injuries of two organs from one anatomical system
d) Injuries of one isolated organ in different anatomical systems (lung, spleen, fractured tibia)
e) Solitary injuries of two organs from different anatomical systems

921. CM. Which of the given injuries are related to associated trauma (polytrauma)?
a) Wound of the stomach, liver and spleen
b) Fracture of hip and flame burns of lower extremities
c) Rupture of spleen and ribs fractures
d) Fracture of ribs, lung injury and pneumothorax
e) Hip fracture, concussion of brain and liver injury

922. CM. Which of the following injuries are related to multiple trauma?
a) Wound of the stomach, liver and spleen
b) Fracture of hip and flame burns of lower extremities
c) Rupture of spleen and ribs fractures
d) Fracture of ribs, lung injury and pneumothorax
e) Hip fracture, concussion of brain and liver injury

923. CS. Which of the following injuries relates to multiple trauma?


a) Injuries of two and more organs from one anatomical system
b) Injuries of two and more organs from different anatomical systems
c) Multiple injuries of one organ in one anatomical system
d) Injuries caused by action of two or more traumatic factors
e) All mentioned above

924. CM. What periods are distinguished in traumatic disease?


a) Period of traumatic shock
b) Period of hemodynamics stabilization
c) Period of early manifestations
d) Period of late disturbances
e) Period of convalescence

925. CM. Which of the following statements characterizes the period of traumatic shock in traumatic disease?
a) Development of dystrophic and sclerotic processes in the inner organs
b) It is caused by direct tissue damage and acute blood loss
c) It is caused by massive resorbtion into the blood stream of tissue and bacterial toxins
d) It lasts from several hours to 2 days
e) Development of acute cardiovascular failure

926. CM. Which of the following statements characterizes the period of early manifestations in traumatic disease?
a) It is caused by massive resorption of tissue and bacterial toxins into the blood stream
b) It is caused by direct tissue damage and acute blood loss
c) It lasts from several hours to 2 days
d) It lasts from 2 to 10 days
e) High fever over 38°С

927. CS. In what period of traumatic disease the traumatized patient may develop an acute toxemia and
septicopyemia?
a) In the period of traumatic shock
b) In the period of early manifestations
c) In the period of late disturbances
d) In the period of convalescence
e) In the period of degenerative and atrophic changes

928. CM. Which of the following statements characterizes the period of late disturbances in traumatic disease?
a) Development of dystrophic and sclerotic processes in the inner organs
b) It is caused by direct tissue damage and acute blood loss
c) It is caused by massive resorption of tissue and bacterial toxins into the blood stream
d) Development of contractures and ankylosis
e) Development of acute cardiovascular failure

929. CS. The length of convalescence period in traumatic disease is:


a) Up to 2 days
b) From 2 to 10 days
c) From 2 to 3 weeks
d) From 1 to 3 months
e) From a few months to several years

930. CM. Mechanisms of primary brain injury after trauma include:


a) Damage to vessels
b) Increased intracranial pressure
c) Hypoxia
d) Disruption of brain structures
e) Altered cellular biochemical processes

931. CM. Mechanisms of secondary brain injury after trauma include:


a) Damage to vessels
b) Increased intracranial pressure
c) Hypoxia
d) Disruption of brain structures
e) Altered cellular biochemical processes

932. CM. What local symptoms may suggest severe head trauma (eg, fractures of the skull base)?
a) Significant bleeding from head wound
b) Periorbital ecchymosis (“raccoon’s sign”)
c) Otorrhea
d) Ecchymosis over the mastoid process
e) Rhinorrhea

933. CM. Which of the following signs is related to general neurological symptoms after head trauma?
a) Hyperthermia
b) Hemiparesis
c) Anisocoria
d) Headache and dizziness
e) Nausea and vomiting

934. CM. Which of the following signs is related to focal neurological symptoms after head trauma?
a) Hyperthermia
b) Hemiparesis
c) Anisocoria
d) Headache and dizziness
e) Nausea and vomiting

935. CS. What is a unilateral hemiparesis in case of traumatic brain injury?


a) Loss of muscle function of arm and leg from one side of the body
b) Weakness of voluntary movements of both legs
c) Weakness of voluntary movements of arm and leg from one side of the body
d) Loss of muscle function of both legs
e) Weakness of voluntary movements of both arms
936. CS. What is a unilateral paralysis in case of traumatic brain injury?
a) Loss of muscle function of arm and leg from one side of the body
b) Weakness of voluntary movements of both legs
c) Weakness of voluntary movements of arm and leg from one side of the body
d) Loss of muscle function of both legs
e) Weakness of voluntary movements of both arms

937. CS. How is called the pupillary’s diameter difference due to traumatic brain injury?
a) Exophthalmos
b) Mydriasis
c) Anisocoria
d) Nystagmus
e) Diplopia

938. CS. Glasgow scale is used to determine:


a) Grade of obesity
b) Operating risk
c) Level of consciousness
d) Severity of blood loss
e) Prognosis of burns disease

939. CM. Which parameters in Glasgow scale are used to determine the level of consciousness in head trauma?
a) Eye opening
b) Pupillary’s light reflex
c) Verbal response
d) Motor response
e) Skin sensitivity

940. CM. Which of the following head injuries cause only general neurological symptoms?
a) Skull fractures
b) Cerebral contusion
c) Diffuse axonal injury
d) Brain concussion
e) Cerebral compression by hematoma

941. CM. Which of the following head injuries cause both general and focal neurological symptoms?
a) Skull fractures
b) Cerebral contusion
c) Diffuse axonal injury
d) Brain concussion
e) Cerebral compression by hematoma

942. CS. Specify the mildest form of brain injury.


a) Cerebral contusion
b) Diffuse axonal injury
c) Cerebral concussion
d) Cerebral compression
e) Subdural and epidural hematoma

943. CM. Which type of hematoma does not lead to cerebral compression in head injury?
a) Epidural hematoma
b) Subdural hematoma
c) Subfascial hematoma
d) Intracerebral hematoma
e) Subcutaneous hematoma

944. CM. Skull fractures are divided into:


a) “Green stick” fractures
b) Basal skull fractures
c) Linear fractures
d) Open fractures
e) Depressed fractures

945. CS. Which of the given symptoms does not occur in brain concussion?
a) Bradycardia
b) Loss of consciousness
c) Headache
d) Retrograde amnesia
e) Anisocoria

946. CM. Which of the following symptoms suggest cerebral compression by posttraumatic hematoma?
a) Anisocoria
b) Hemiparesis
c) Retrograde amnesia
d) Oliguria
e) Sleepiness

947. CM. What do neurological symptoms determine in intracranial hematomas?


a) Status of central hemodynamics
b) Quality of infusion therapy
c) Localization of hematoma
d) Status of blood coagulation
e) Size of hematoma

948. CS. What is the main method of treatment for intracerebral hematomas?
a) Haemostatic therapy
b) Infusion therapy (prevention of cerebral edema)
c) Anticonvulsants
d) Surgical (craniotomy, trepanation)
e) Endovascular intervention (embolization of the bleeding vessel)

949. CS. In case of the chest injury subcutaneous emphysema suggests:


a) Hemothorax
b) Pneumothorax
c) Hydrothorax
d) Pyothorax
e) Ribs fractures

950. CM. What symptoms are always determined in rib fractures?


a) Local tenderness on palpation
b) Subcutaneous hematoma
c) Subcutaneous emphysema
d) Pathological mobility and crepitus of rib fragments
e) Asymmetry of the chest

951. CM. Which of the following signs allows to determine clinically rib fracture?
a) Crepitus of fragments
b) Localized pain
c) Deformity
d) Subcutaneous emphysema
e) Diminished breath sounds on the affected side

952. CM. What are the clinical manifestations of floating rib fractures (flail chest)?
a) Abnormal instability of the segment
b) Unilateral impairment of the chest wall excursion on the side of injury
c) “Paradoxal” breathing
d) Crepitus
e) Presence of a large chest wall defect

953. CM. What characterizes the phenomenon of “paradoxical” breathing in case of flail chest (type of rib
fracture)?
a) With inspiration the affected chest wall segment moves outwardly
b) With expiration the affected chest wall segment moves inwardly
c) With inspiration the affected chest wall segment moves inwardly
d) With expiration the affected chest wall segment moves outwardly
e) With inspiration is heard the flow of air through the chest wound

954. CM. What symptoms are characteristic of flail chest?


a) The line of fracture is localized in the lower part of the rib cage
b) Unilateral impairment of the chest wall excursion on the side of injury
c) The chest wall segment moves outwardly with inspiration, and inwardly – with expiration
d) The segment of the chest wall does not have bony continuity to the rest of the rib cage
e) The chest wall segment moves inwardly with inspiration, and outwardly – with expiration

955. CM. Specify the correct statements, relating to sternal fractures.


a) Sternal fractures are usually transverse
b) They occur after a significant blow to the anterior chest
c) They are associated with collapsed lung and displaced mediastinum to the opposite side
d) Posterior displacement of the fractured sternum can impinge on the heart
e) Specific symptoms include severe chest or back pain, upper extremity hypertension, and asymmetry of
pulses in the upper and lower extremities

956. CS. What does the tympanic sound on percussion of the chest suggest in case of injury?
a) Hemothorax
b) Pneumothorax
c) Hydrothorax
d) Pyothorax
e) Ribs fractures

957. CS. In case of trauma dullness on percussion of the chest suggests:


a) Hemothorax
b) Cardiac contusion
c) Pneumothorax
d) Rib fractures
e) Sternal fractures

958. CM. Specify the symptoms of simple (closed) pneumothorax.


a) Dullness on percussion of the chest on the affected side
b) Decreased breath excursion of the affected side of chest
c) Hyperresonance on percussion of the chest on the affected side
d) Dyspnea
e) Breath sounds decreased or absent on the side of injury

959. CS. What treatment is recommended, if simple pneumothorax is large enough to be seen on plain chest
radiograph?
a) Thoracentesis with aspiration of air
b) Decompression of pleural cavity with a large-bore needle
c) Conservative therapy followed by radiological control
d) Tube thoracostomy
e) Thoracotomy with suturing of injured bronchus

960. CS. “Valve” pneumothorax is also called:


a) Simple pneumothorax
b) Open pneumothorax
c) Closed pneumothorax
d) Tension pneumothorax
e) Complicated pneumothorax

961. CM. What is characteristic of tension pneumothorax?


a) The affected lung is completely collapsed, and the mediastinum is displaced to the opposite side
b) Intrathoracic and ambient pressures are equilibrated, leading to collapse of the lung
c) Extensive interstitial hemorrhage and filling alveoli with blood and plasma, which leads to their collapse
d) Incoming into the pleural cavity air cannot exit out and progressively accumulates
e) Decrease of venous return and compromising ventilation of the other lung

962. CM. What symptoms are helpful to differentiate tension (valve) pneumothorax from a closed (simple)
pneumothorax?
a) Distended neck veins and cyanosis
b) Hyperresonance on percussion of the chest on the affected side
c) Dullness on percussion of the chest on the affected side
d) Complete collapse of the lung and displacement of the mediastinum to the opposite side
e) Dyspnea

963. CS. What emergency curative measure is indicated in tension pneumothorax?


a) Application of occlusive dressing
b) Decompression of pleural cavity with large-bore needle
c) Thoracotomy with suturing of defect in the lung
d) Fixation of rib fragments using intraosseous or extrafocal osteosynthesis
e) Bronchoscopy with occlusion of the appropriate bronchus

964. CM. Open pneumothorax is characterized by:


a) Intrathoracic and ambient pressures are equilibrated, leading to collapse of the lung
b) Incoming into the pleural cavity air cannot exit out and progressively accumulates
c) The affected lung is completely collapsed, and mediastinum and trachea are displaced to the opposite side
d) The presence of a large chest wall defect
e) Usually it is caused by shotgun wound

965. CM. Open pneumothorax is diagnosed based on the following signs:


a) The flow of air through the chest wound is heard with every breath
b) Distension of the neck veins
c) Large defect in the chest is determined
d) The affected lung is completely collapsed, and mediastinum and trachea are displaced to the opposite side
e) Dullness on percussion of the chest on the affected side

966. CM. What clinical symptoms are similar in hemothorax and pneumothorax?
a) Shock
b) Hyperresonance on percussion of the chest on the affected side
c) Dyspnea
d) Dullness on percussion of the chest on the affected side
e) Decreased breath sounds on the side of injury

967. CM. Which of the following clinical signs of hemothorax does not occur in pneumothorax?
a) Shock
b) Hyperresonance on percussion of the chest on the affected side
c) Presence of dyspnea
d) Dullness on percussion of the chest on the affected side
e) Decreased breath sounds on the side of injury

968. CM. Possible urgent surgical maneuvers in hemothorax include:


a) Thoracotomy
b) Application of hermetic impermeable rubber dressing
c) Angiography with embolization of a bleeding vessel
d) Tube thoracostomy
e) Fixation of rib fragments using intraosseous or extrafocal osteosynthesis
969. CM. Which of the following symptoms is characteristic of injuries of trachea and major bronchi?
a) Hemorrhagic shock
b) Pneumothorax
c) Hemoptysis
d) Pneumomediastinum (presence of air into the mediastinum)
e) Dullness on percussion over both pleural cavities

970. CM. Characteristic features of the cardiac tamponade are:


a) It leads to restriction of cardiac and impairs venous return
b) Rupture of diaphragm with displacement of abdominal organs into the pleural cavity, which is accompanied
by compression of the heart
c) It most often develops after penetrating injuries of the central area of the chest
d) Accumulation of blood into the pericardial sac
e) Pleural cavity is filled with clots, which leads to compression of the mediastinum

971. CM. The symptoms that constitute the triad of Beck in cardiac tamponade are:
a) Decline in arterial pressure
b) Tachycardia
c) Jugular venous distention
d) “Paradoxal” respiration
e) Muffled heart tones

972. CS. Specify the triad of Beck in cardiac tamponade.


a) Hypotension, diffuse enlargement of heart borders, blood on pericardial aspiration
b) Muffled heart tones, hypotension, jugular venous distention
c) High central venous pressure, diffuse enlargement of heart borders, blood on pericardial aspiration
d) Jugular venous distention, diffuse enlargement of heart borders, blood on pericardial aspiration
e) Muffled heart tones, hypotension, presence of wound in the central area of the chest

973. CM. Which three symptoms are combined into the triad of Beck in traumatic cardiac tamponade?
a) Muffled heart tones
b) Reduced cardiac output
c) Extension of topographic borders of the heart
d) Decline in arterial pressure
e) Jugular venous distention

974. CM. Symptoms of traumatic aortic rupture include:


a) Intolerable headache
b) Severe chest or back pain
c) Upper extremity hypertension
d) Asymmetry of pulses in the upper and lower extremities
e) Anuria

975. CS. Unilateral dullness on percussion with appearance of peristaltic sounds over the affected side of chest is
revealed in case of:
a) Tension pneumothorax
b) Caked or clotted hemothorax
c) Esophageal injury
d) Traumatic diaphragmatic hernia
e) Traumatic rupture of intestine

976. CM. Which of the following causes of esophageal traumatic injuries are iatrogenic by origin?
a) Esophagoscopy
b) Pressure injuries by Blakemore tube
c) Esophageal dilatations
d) Chemical burns of esophagus
e) Penetrating wounds of esophagus

977. CM. Esophageal injuries reveal the following symptoms:


a) Chest pain
b) Subcutaneous emphysema
c) Dysphagia
d) Asymmetry of pulses in the upper and lower extremities
e) Painful cough

978. CS. When is abdominal wound considered to be penetrating?


a) When the skin is damaged
b) When skin, subcutaneous tissue, aponeurosis and muscles of abdominal wall are damaged
c) When an abdominal organ is damaged
d) When the parietal peritoneum is damaged
e) When the visceral peritoneum is damaged

979. CS. What is the main purpose of examination of patients with abdominal trauma?
a) To determine the general status of patient and the presence of comorbidities
b) To determine the existence of intra-abdominal injuries
c) To determine, whether the injury is blunt or penetrating
d) To determine, what specific intra-abdominal organ is injured
e) To determine the exact circumstances of injury

980. CM. Which clinical syndromes are distinguished in abdominal trauma with inner organs damage?
a) Syndrome of intestinal obstruction
b) Hemorrhagic syndrome
c) Inflammatory syndrome
d) Peritoneal syndrome
e) Intoxication syndrome

981. CS. A successful kidney transplant from a living donor for the first time was performed by:
a) Alexis Carrell in Lyon
b) Joseph Murray in Boston
c) Theodor Billroth in Vienna
d) Ignaz Semmelweis in Budapest
e) Joseph Lister in Edinburgh

982. CM. What fundamental discoveries formed the basis for the development of transplantation?
a) Discovery of asepsis and antisepsis
b) Implementation of vascular anastomosis
c) Invention of device for artificial respiration
d) Elaboration of immunosuppressive drugs
e) Opening of histocompatibility laws

983. CM. What organs and tissues for transplantation can be extracted from a living donor?
a) Heart
b) Cornea
c) Kidney
d) Segment of small bowel
e) Liver lobe

984. CM. What organs and tissues for transplantation can be extracted only from a dead donor?
a) Bone tissue
b) Heart
c) Entire liver
d) Cornea
e) Segment of small bowel

985. CM. Specify the clinical signs of brainstem death, which are evaluated for possible organ transplantation.
a) Absence of motor response to painful stimuli applied to the lower extremities
b) Absence of motor response to painful stimuli applied to the head or face
c) Absence of pupillary and corneal reflexes
d) Absence of spontaneous breathing
e) Absence of consciousness within 14 days

986. CS. After extraction of organs for transplantation, the longest time in conditions of cold ischemia outside the
body can be maintained:
a) Kidney
b) Liver
c) Pancreas
d) Heart
e) Lungs

987. CS. The period of warm ischemia in organ transplantation begins:


a) After removing of the donor organ from a cold preservating solution
b) After the clamping of aorta and introduction of the ice cold water into the donor body
c) After beginning of dissection on the back-table, where the vessels of donar organ are prepared for
anastomosis
d) After connecting the vessels of the donor organ with the recipient blood vessels
e) After the organ is removed from the donor body

988. CM. In what types of transplantation does not develop a reaction of rejection?
a) Allogeneic
b) Syngeneic
c) Xenogeneic
d) Autogeneic
e) Exogenous

989. CS. Which of these types of transplantation is called orthotopic?


a) Kidney is transplanted from the Siamese twin of the patient
b) Donor kidney is transplanted into the iliac region, forming vascular anastomoses with the iliac vessels
c) To patient is transplanted a kidney from animal
d) To patient is transplanted a kidney from his/her mother
e) Donor kidney is transplanted into the bed, remaining after removing of the affected kidney

990. CS. Which of these types of transplantation is called heterotopic?


a) Kidney is transplanted from the Siamese twin of the patient
b) Donor kidney is transplanted into the iliac region, forming vascular anastomoses with the iliac vessels
c) To patient is transplanted a kidney from animal
d) To patient is transplanted a kidney from his/her mother
e) Donor kidney is transplanted into the bed, remaining after removing of the affected kidney

991. CS. Which of these options of organ transplantation refers to xenogeneic transplant?
a) To patient is transplanted an organ from his monozygotic twin
b) To patient is transplanted his own organ
c) To patient is transplanted an organ from animal
d) To patient is transplanted an organ from his/her brother or sister
e) To patient is transplanted an artificial organ

992. CS. Which of these options of organ transplantation refers to syngeneic transplant?
a) To patient is transplanted an organ from his monozygotic twin
b) To patient is transplanted an organ from his/her brother or sister
c) To patient is transplanted an organ from animal
d) To patient is transplanted an organ from donor, selected on the basis of tissue compatibility
e) To patient is transplanted an artificial organ

993. CS. Which of these options of organ transplantation refers to allogeneic transplant?
a) To patient is transplanted an organ from his monozygotic twin
b) To patients is transplanted his own organ
c) To patient is transplanted an organ from animal
d) To patient is transplanted an organ from donor, selected on the basis of tissue compatibility
e) To patient is transplanted an artificial organ

994. CS. The use of non organic or synthetic materials for replacement of tissues and organs is named:
a) Prosthetics
b) Heterotopic transplantation
c) Orthotopic transplantation
d) Replantation
e) Xenogeneic transplantation

995. CM. Specify the antigens of histocompatibility responsible for rejection of transplanted organs and tissues in
humans?
a) System of erythrocytes antigens ABO
b) Microbial antigens in case of violation of aseptic rules
c) System of human leukocyte antigens HLA
d) Minor histocompatibility antigens
e) Platelet antigens

996. CM. Specify the types of rejection reaction, occurring in transplantation.


a) Hyperacute rejection
b) Subacute rejection
c) Acute rejection
d) Chronic rejection
e) Recurrent rejection

997. CM. Specify the causes of hyperacute transplant rejection.


a) Incompatibility according to ABO antigens
b) T-lymphocyte immune response
c) Presence at recipient of antibodies to the donor leucocyte antigen system
d) Ischemia and fibrosis of transplanted organ
e) Violation of aseptic rules

998. CS. Specify the cause of acute transplant rejection.


a) Incompatibility according to ABO antigens
b) T-lymphocyte immune response
c) Presence at recipient of antibodies to the donor leucocyte antigen system
d) Ischemia and fibrosis of transplanted organ
e) Violation of aseptic rules

999. CS. Specify the cause of chronic transplant rejection.


a) Incompatibility according to ABO antigens
b) T-lymphocyte immune response
c) Presence at recipient of antibodies to the donor leucocyte antigen system
d) Ischemia and fibrosis of transplanted organ
e) Violation of aseptic rules

1000. CS. Prevention of transplant rejection in the postoperative period consists in:
a) Determining compatibility according the ABO system
b) Determining compatibility according HLA system
c) Repeated transplantation
d) Long time immune suppression
e) Treatment of bacterial and viral infections

1001. CS. What is sepsis?


a) The presence of bacteria in the bloodstream with purulent metastasis into the tissues and organs
b) A microbial phenomenon characterized by local inflammatory response to the presence of microorganisms
c) The presence and active multiplication of bacteria in the bloodstream
d) The presence of bacteria in the bloodstream
e) An infectious process accompanied by acute inflammatory reaction with systemic manifestations
1002. CM. Which of the following clinical phenomena is characteristic for sepsis?
a) Sustained bacteremia
b) Transient bacteremia
c) Symptomatic bacteremia
d) Septicemia
e) Septicopiemia

1003. CS. What is bacteremia?


a) The presence of bacteria in the bloodstream
b) The presence of bacteria in the bloodstream with purulent metastasis into the tissues and organs
c) The presence and active multiplication of bacteria in the bloodstream
d) A microbial phenomenon characterized by local inflammatory response to the presence of microorganisms
e) An infectious process accompanied by acute inflammatory reaction with systemic manifestations

1004. CS. What is septicemia?


a) An infectious process accompanied by acute inflammatory reaction with systemic manifestations
b) A microbial phenomenon characterized by local inflammatory response to the presence of microorganisms
c) The presence and active multiplication of bacteria in the bloodstream
d) The presence of bacteria in the bloodstream
e) The presence of bacteria in the bloodstream with purulent metastasis into the tissues and organs

1005. CS. What is septicopiemia?


a) The presence and active multiplication of bacteria in the bloodstream
b) An infectious process accompanied by acute inflammatory reaction with systemic manifestations
c) The presence of bacteria in the bloodstream with purulent metastasis into the tissues and organs
d) A microbial phenomenon characterized by local inflammatory response to the presence of microorganisms
e) The presence of bacteria in the bloodstream

1006. CM. Which of the following statements about sepsis is true?


a) Clinical, sepsis is manifested in the same way although the casual microorganisms are different
b) Sepsis does not have any incubation period
c) Clinical manifestations of sepsis depend on the casual microorganisms
d) Sepsis is systemic process
e) Incubation period of sepsis is short (<24 hours)

1007. CM. The following criteria are used to diagnose systemic inflammatory response syndrome (SIRS):
a) White blood cell (WBC) count
b) Systolic blood pressure
c) Temperature
d) Respiratory rate
e) Heart rate

1008. CM. Which of the following confirms the presence of systemic inflammatory response syndrome (SIRS)?
a) Heart rate < 90 beats/min
b) Heart rate > 90 beats/min
c) Temperature > 37,5°C
d) Temperature > 38°C
e) Temperature < 36°C

1009. CM. Which of the following confirms the presence of systemic inflammatory response syndrome (SIRS)?
a) White blood cell (WBC) count > 12.000/mm3
b) White blood cell (WBC) count < 4.000/mm3
c) Systolic blood pressure < 90 mm Hg
d) Respiratory rate < 14 breaths/min
e) Respiratory rate > 20 breaths/min

1010. CS. What combination of patients’ parameters is corresponding to presence of systemic inflammatory
response syndrome (SIRS)?
a) Temperature – 38,5°C; Heart rate – 90 beats/min; Respiratory rate – 16 breaths/min; White blood cell (WBC)
count – 14.000/ mm3
b) Temperature – 38,5°C; Heart rate – 88 beats/min; Respiratory rate – 18 breaths/min; White blood cell (WBC)
count – 11.000/ mm3
c) Temperature – 37,7°C; Heart rate – 100 beats/min; Respiratory rate – 17 breaths/min; White blood cell
(WBC) count – 9.000/ mm3
d) Temperature – 37,5°C; Heart rate – 84 beats/min; Respiratory rate – 22 breaths/min; White blood cell (WBC)
count – 10.000/ mm3
e) Temperature – 36,2°C; Heart rate – 70 beats/min; Respiratory rate – 14 breaths/min; White blood cell (WBC)
count – 3.000/ mm3

1011. CS. Choose the definition of “severe sepsis”?


a) It is systemic inflammatory response syndrome (SIRS) associated with organ dysfunction
b) It is systemic inflammatory response syndrome (SIRS) to documented infection
c) It is sepsis associated with organ dysfunction
d) It is systemic inflammatory response syndrome (SIRS)
e) It is sepsis associated with hypotension persisting despite adequate fluid resuscitation

1012. CM. Specify the reasons of increasing incidence of sepsis at present.


a) The increasing use of therapies that improve survival from underlying diseases but result in
immunosuppression
b) The use of invasive devices for monitoring and treatment of critically ill patients
c) Increasing antibiotic resistance of bacteria
d) Improving of diagnostic measures in sepsis
e) Genetic factors impact on the risk of development of sepsis or dying from infection

1013. CM. The increased risk of sepsis have:


a) Patients with diabetes mellitus
b) Patients with systemic lupus erythematosus
c) Men
d) Women
e) Patients with atherosclerosis

1014. CM. Specify the three cytokines of major importance in the pathogenesis of sepsis.
a) Interleukin 10 (IL10)
b) Platelet activating factor (PAF)
c) Tumor necrosis factor-alpha (TNF-alpha)
d) Interleukin 1 (IL1)
e) Prostaglandin Е (PGE)

1015. CM. Which of the following refers to generalized effects caused by cytokines in sepsis?
a) Endothelial injury
b) Neutrophil-endothelial cell adhesion
c) Coagulopathy with capillary leak and microthrombi
d) Release of blood from the depot
e) Paralysis of the respiratory center and heart failure

1016. CM. What are the main mechanisms of multiple organ failure in sepsis?
a) Intravascular coagulation and excessive fibrinolysis
b) Blood flow bypass capillary exchange vessels (a distributive defect)
c) Massive blood loss
d) Excessive release of endogenous cytokines
e) Translocation of intestinal bacteria and endotoxin to the portal and systemic circulations

1017. CS. What is the mechanism of bacterial translocation phenomenon that leads to sepsis and multiple organ
failure?
a) Impairment of intestinal barrier function
b) Capillary blood flow bypass
c) Endothelial injury by cytokines
d) Decreasing of phagocytic activity of neutrophils
e) Coagulopathy with microthrombi and cessation of oxygen delivery to the tissues

1018. CM. What classifications of sepsis are used?


a) Classification according to clinical evolution
b) Classification according to the type of bacteriological agent
c) Classification according to level of host immunosuppression
d) Classification according to localization of primary infectious focus
e) Classification according to severity of clinical manifestations

1019. CS. Specify the microbial agent, which is least likely to cause sepsis.
a) Bacteroides fragilis
b) Escherichia Coli
c) Klebsiella
d) Candida
e) Staphylococcus

1020. CS. Which of the following types of sepsis is observed most commonly in clinical practice?
a) Urological sepsis
b) Neonatal sepsis
c) Abdominal sepsis
d) Pulmonary sepsis
e) Soft tissue sepsis (phlegmon, gangrene)

1021. CM. The clinical diagnostics of severe sepsis is based on:


a) Identification of signs of multiple organ failure
b) Determination of serum cytokines level
c) Evidence of positive blood, urine or other suspect body fluids culture
d) Identification of classical local signs of surgical infection
e) Identification of positive criteria of systemic inflammatory response syndrome (SIRS)

1022. CM. The laboratory diagnosis of severe sepsis is based on:


a) Positive blood culture
b) Thrombocytopenia < 100.000/ mm3
c) Increased levels of blood cytokines
d) Twice increased levels of liver ferments
e) High level of serum procalcitonin

1023. CS. Which of the following results of investigations is crucial to confirm the diagnosis of sepsis?
a) Positive blood culture
b) Leukocytosis with a left shift
c) High level of serum cytokines (TNF)
d) Increased level of C-reactive protein
e) High contents of procalcitonin in the blood

1024. CS. The positive blood culture in sepsis with isolation of Bacteroides fragiles suggests, that source of
infection most likely is situated in:
a) Colon
b) Gallbladder or bile ducts
c) Urinary tract
d) Blood
e) Lungs

1025. CM. What characterizes the surgical treatment of primary purulent focus in sepsis of soft tissues?
a) Broad debridement and excision of necrotized tissues are necessary
b) Adequate drainage of abscesses is required
c) Surgical debridement has no effect on the evolution of sepsis
d) Closure of wounds with primary sutures is recommended
e) Wounds usually are treated in the open way
1026. CM. Specify the rules of antibiotic therapy administration in sepsis.
a) Broad-spectrum antibiotics should be administrated
b) Antibiotics should be modified by culture results
c) Antibiotic therapy should begin immediately after the diagnosis of sepsis
d) Antibiotic therapy should begin only after obtaining the result of positive blood culture according to antibiotic
sensitivity of isolated bacteria
e) Initially, antibiotics are administrated empirically

1027. CM. Specify the terms – synonyms of soft tissue anaerobic clostridial infection.
a) Anaerobic cellulitis
b) Tetanus
c) Gas gangrene
d) Necrotizing fasciitis
e) Anaerobic myonecrosis

1028. CS. The most frequent causative bacterial agent of gas gangrene is:
a) Clostridium hystoliticum
b) Clostridium oedomatiens
c) Clostridium tetani
d) Clostridium perfringens
e) Clostridium septicum

1029. CM. Which biologic effects are caused by alpha exotoxin of Clostridium perfringens?
a) Hemolysis or thrombosis
b) Myocardial suppression
c) Impairment of intestinal barrier function
d) Endothelial injury
e) Tissue necrosis

1030. CM. Forming of gas in the soft tissues is a classical characteristic of the following types of infection:
a) Anaerobic non-clostridial infection
b) Anaerobic clostridial infection
c) Tetanus
d) Pneumococcal infection
e) Specific inflammatory process caused by Mycobacterium tuberculosis

1031. CM. Specify the wounds with high risk of anaerobic clostridial infection of soft tissues (gas gangrene)
development.
a) Wounds with heavy soil contamination
b) Wounds with massive damage of muscles and bones
c) Wounds with profound and narrow wound channel (gunshot wound)
d) Wounds with ischemia of tissue caused by vessel injury, or a tourniquet placed for a long time
e) Cut wounds, closed tightly with primary sutures

1032. CM. What pathophysiological mechanisms contribute to the development of ischemia and create anaerobic
conditions in anaerobic clostridial infection of soft tissues (gas gangrene)?
a) Spread of gas along the fascial planes
b) Increased pressure in the fascial spaces due to edema
c) Disturbance of circulation due to thrombosis of peripheral blood vessels
d) Spontaneous thrombosis of large arteries
e) Rapid multiplication of microorganisms

1033. CM. Anaerobic clostridial infection of soft tissues (gas gangrene) is classified in the following forms:
a) Posttraumatic
b) Endogenous
c) Spontaneous
d) Postoperative
e) Generalized
1034. CS. Spontaneous anaerobic clostridial infection of soft tissues (gas gangrene) most commonly develops in
patients:
a) Who underwent recent surgery of the biliary tract
b) With small bite wounds
c) With occult malignancy
d) Who underwent recent surgery of the colon
e) With open fractures

1035. CM. Specify the typical clinical signs of soft tissue anaerobic clostridial infection (gas gangrene).
a) Persistent rigidity in the muscle group close to the injury site
b) Hemorrhagic bullae on the skin
c) Crepitus
d) Hyperemia of the affected area of skin, moderate edema
e) Severe pain in extremity and sensation of compression

1036. CS. What is Melnikov's test in anaerobic clostridial infection of soft tissues (gas gangrene)?
a) Crepitus defined on palpation
b) Timpanitis with “metallic” sound on percussion
c) Presence of hemorrhagic bullae on the skin
d) Sound of gas exit during evacuation of gauze dressing from wound channel
e) A thread is placed loosely around the extremity, and it penetrates into the skin in 20-30 min

1037. CS. Crepitus defined on palpation in anaerobic clostridial infection of soft tissue is associated with:
a) Penetration of air from the gastrointestinal tract into the wound in postoperative form of infection
b) Friction of bone fragments in the wounds received in open fractures
c) Production of a lot of exotoxins
d) Production of gas in tissue
e) Penetration of air from the external environment through the wound channel

1038. CS. In anaerobic clostridial infection of soft tissues (gas gangrene) affected muscles:
a) Edematous, but of normal color and consistency
b) Edematous, of normal color, on incision from them eliminate multiple gas bubbles
c) Rigid, due to persistent muscle spasm
d) Look like “boiled meat”, edematous, gray-colored
e) Edematous, of cyanotic or black color due to massive necrosis

1039. CM. Which of the following imaging and laboratory studies are the most informative for the diagnosis of
anaerobic clostridial infection of soft tissues (gas gangrene)?
a) Clostridial species growth from the blood cultures
b) Enzyme-linked immunosorbent assay (ELISA) of infected tissues
c) Gram stained microscopy of infected tissues
d) Ultrasound scan of the affected area
e) Radiographic examination of the affected area

1040. CS. In anaerobic clostridial infection (gas gangrene) identification of gas layers in soft tissues during
radiographic examination or CT scan is also called:
a) Sign of “boiled meat”
b) Krause's sign
c) Melnikov's sign
d) Sign of “champagne cork”
e) Vishnevsky’s sign

1041. CM. What are histopathological findings in anaerobic clostridial infection of soft tissues (gas gangrene)?
a) The presence of large gram-positive bacilli without neutrophils
b) Widespread myonecrosis
c) The presence of necrotic and nonviable tissue with a large accumulation of neutrophils
d) Areas of gas between the muscle fibers
e) Purulent metastases in remote tissues and organs
1042. CM. What is the aim of longitudinal incisions (fasciotomy) in anaerobic clostridial infection (gas gangrene)of
extremity?
a) To simplify the excision of necrotized and visual devitalized tissues
b) To accelerate the wound healing
c) To decrease compartment syndrome and tissue ischemia
d) To improve the access of air to wound channel
e) To prevent the multiplication of clostridia spores

1043. CS. Which of these additional methods is optimal for the treatment of anaerobic clostridial infection of soft
tissues (gas gangrene)?
a) Hemofiltration
b) Hyperbaric oxygen therapy
c) Plasmapheresis
d) Hemodialysis
e) Hemosorbtion

1044. CM. Specify additional non-surgical treatment methods of anaerobic clostridial infection of soft tissues.
a) Administration of antigangrenous serum
b) Hyperbaric oxygen therapy
c) Effective antibiotic therapy
d) Treatment in controlled abacterial environment
e) Administration of antigangrenous vaccine

1045. CM. Specify the microorganisms responsible for anaerobic non-clostridial infection of soft tissue.
a) Pseudomonas aeruginosa
b) Bacteroides fragiles
c) Clostridium perfringens
d) Peptococcus
e) Klebsiella

1046. CM. What are the typical clinical signs of anaerobic non-clostridial infection of soft tissue?
a) Slow evolution of pathological process
b) Wide, rapid, and progressive spreading of pathological process
c) Predominant damage of muscles and conjunctive tissues
d) Phlegmon with massive affection of subcutaneous adipose tissue
e) Hyperemia of the affected area of skin, moderate edema

1047. CM. Non-clostridial anaerobic infection can clinically manifest as phlegmon with extensive damage of the
following tissues:
a) Skin (dermatitis)
b) Subcutaneous adipose tissue (cellulitis)
c) Connective tissue (fasciitis)
d) Muscles (myositis)
e) Bone tissue (osteomyelitis)

1048. CM. Specify methods, which may be helpful for the diagnosis of anaerobic non-clostridial infection of soft
tissue.
a) Cultures and incubation of microorganisms under anaerobic conditions
b) Gram stain of the infected tissues or exudates
c) Biochemical tests for determination of the serum cytokines level
d) Detection of toxins in infected tissues by enzyme-linked immunosorbent assay (ELISA)
e) Gas-liquid chromatography of infected tissues or exudates

1049. CM. The complex treatment of patients with anaerobic non-clostridial infection includes:
a) Urgent surgical debridement of infectious focus with excision of all devitalized tissues
b) Administration of immunosuppressors and cytostatics
c) Hyperbaric oxygen therapy
d) Massive antimicrobial therapy
e) Administration of immunostimulatory medication

1050. CS. What is the most common source of infection in tetanus?


a) Infected chronic ulcers
b) Exposure via intravenous drug abuse
c) Surgery on gastrointestinal tract organs
d) Umbilical cord contamination during unsanitary delivery
e) Infected lacerations or puncture wounds

1051. CM. Which of the following statements are true characteristics of tetanus?
a) Although rare, tetanus can not been eradicated
b) Infection caused by anaerobic microorganisms with preferential affection of fat and conjunctive tissues
c) Neonatal tetanus accounts for 50% of tetanus-related deaths in developing countries
d) The disease is not transmitted from one person to another
e) Mortality rate from severe tetanus may be as high as 60%

1052. CS. Specify the correct classification of tetanus.


a) Generalized, local, cephalic, neonatal
b) Posttraumatic, postoperative, spontaneous
c) Myositis, fasciitis, combined
d) Fulminate, acute, subacute, chronic
e) Wound, intravenous, postoperative, idiopathic

1053. CS. What form of tetanus is less dangerous?


a) Generalized tetanus
b) Postinjection tetanus (after vaccine administration)
c) Local tetanus
d) Neonatal tetanus
e) Cephalic tetanus

1054. CM. Which symptoms are common for generalized tetanus?


a) Opisthotonus
b) Cranial nerve palsies
c) Trismus
d) Risus sardonicus
e) Anisocoria

1055. CS. What is trismus, which occurs in tetanus?


a) So-called “scornful” smile of tetanus, resulting from facial muscle involvement
b) Generalized muscle rigidity
c) Persistent rigidity in the muscle group close to injury site
d) Inability to open mouth secondary to masseter muscle spasm
e) Dysphagia and sore throat resulting from the spasm of diaphragm

1056. CS. Local tetanus is characterized by:


a) Inability to open mouth secondary to masseter muscle spasm
b) Persistent rigidity in the muscle group close to injury site
c) Dysphagia and sore throat resulting from the spasm of diaphragm
d) So-called “scornful” smile of tetanus, resulting from facial muscle involvement
e) Apnea resulting from the spasm of intercostal muscles and diaphragm

1057. CM. What characterizes the initial clinical manifestations of tetanus?


a) Body temperature is normal
b) Symptoms usually begin 8 days after contamination
c) First signs are headache and muscular stiffness in the jaw (lockjaw)
d) Symptoms usually begin 24-48 hours after contamination
e) Body temperature is hectic

1058. CS. Paraclinical diagnosis of tetanus is based on:


a) Laboratory findings and imaging studies are not diagnostically valuable
b) Positive culture from wound discharge placed under anaerobic conditions and be incubated for 48 hours
c) Identification of gram-positive bacilli on microscopy of wound exudate
d) Detection of tetanospasmin in infected tissues by enzyme immunoassay
e) Identification of tetanus bacilli metabolites during urgent gas-liquid chromatography

1059. CM. Specify the appropriate curative measures in patients with tetanus.
a) Administration of vaccine (tetanus toxoid)
b) Passive immunization with human tetanus immunoglobulin
c) Administration of broad-spectrum antibiotics
d) Surgical debridement of wound responsible for tetanus infection
e) Administration of pharmacologic agents that treat muscle spasms

1060. CM. Specify measures for tetanus prevention.


a) Administration of pharmacologic agents that treat muscle spasms
b) Administration of tetanus toxoid in combination with diphtheria toxoid and pertussis vaccine (DTP) to children
c) Revaccination is performed each 10 years
d) Passive immunization with human tetanus immunoglobulin
e) Administration of broad-spectrum antibiotics

1061. CM. Specify the correct associations of terms.


a) Diabetes mellitus type I or insulin-dependent
b) Diabetes mellitus type I or insulin-independent
c) Diabetes mellitus type II or insulin-dependent
d) Diabetes mellitus type II or insulin-independent
e) Diabetes mellitus type 0 or glucose-dependent

1062. CM. Choose the correct statements about diabetes mellitus.


a) Mortality among diabetics is 2 times higher than in non-diabetics
b) Vascular complications are the main cause of death in diabetes mellitus
c) Infection is the main cause of death in diabetes mellitus
d) Surgical complications are more frequent in type I diabetes mellitus
e) Surgical complications are more frequent in type II diabetes mellitus

1063. CM. Specify clinical conditions which correspond to diabetic foot syndrome definition provided by WHO.
a) Diabetic neuropathy without foot ulcer, gangrene or infection
b) Infected plantar wound in diabetic patient without neuro- and angiopathy
c) Foot ulcer associated with diabetic neuropathy
d) Toe gangrene associated with angiopathy in diabetic patient
e) Foot ulcer in non-diabetic patient with atherosclerosis

1064. CS. Foot ulcer develops in:


a) Less than 1% of patients with diabetes mellitus
b) Less than 5% of patients with diabetes mellitus
c) 10-15% of patients with diabetes mellitus
d) More than 50% of patients with diabetes mellitus
e) More than 50% of patients with diabetes mellitus

1065. CM. Most frequently lower limb amputations are performed in:
a) Patients with diabetes mellitus
b) Patients with diabetes mellitus and atherosclerosis
c) Patients with acute haematogenous osteomyelitis
d) Patients with vascular malformations
e) Patients with deep vein thrombosis

1066. CM. Pathogenetic classification of diabetic foot includes the following clinical forms of disease:
a) Neuropathic
b) Ischemic
c) Osteomyelitic
d) Neuroischemic
e) Gangrenous

1067. CS. Specify the most frequent form of diabetic foot.


a) Ischemic
b) Osteoarthropathic
c) Neuropathic
d) Neuroischemic
e) Osteomyelitic

1068. CS. Specify the form of diabetic foot which most frequently results in major amputation.
a) Ischemic
b) Osteoarthropathic
c) Neuropatic
d) Neuroischemic
e) Osteomyelitic

1069. CM. Somatic diabetic neuropathy is responsible for:


a) Plantar callus formation
b) Plantar muscles weakness
c) Osteoporosis
d) Occlusion of lower limb arteries
e) Hypoesthesia and anesthesia of the foot

1070. CM. Autonomic diabetic neuropathy is responsible for:


a) Plantar callus formation
b) Plantar muscles weakness
c) Osteoporosis
d) Decreased plantar sweating
e) Hypoesthesia and anesthesia of the foot

1071. CM. Plantar callus formation results in:


a) Spread of pressure out on a wider area
b) Focusing of pressure (concentration in one point)
c) Increased resistance of plantar skin to infection
d) Decreased risk of ulcer development
e) Increase of plantar pressure by more than 10 times

1072. CS. In diabetic foot syndrome critical pressure for ulcer development is equal to:
a) 0,7 kg/cm2
b) 1,5 kg/cm2
c) 3,0 kg/cm2
d) 5,0 kg/cm2
e) 7,0 kg/cm2

1073. CM. Screening for diabetic foot syndrome includes the following tests:
a) Foot inspection for ulcers, gangrene, infection
b) Determination of endogenous insulin level
c) Palpation of plantar pulses
d) Semmes-Weinstein test
e) Duplex ultrasound of arteries of lower limbs

1074. CM. Choose the rules which describe the correct technique of Semmes-Weinstein test.
a) 10 g monofilament is used
b) Bend monofilament for 1 second in each point
c) Test one point on the dorsal surface of the foot and another on the plantar one
d) Ask the patient if he/she feels each touch during the test
e) The absence of sensation only in one point is not an abnormality
1075. CM. Select the right descriptions of Semmes-Weinstein test technique.
a) Before start the test, patient should be instructed by doctor to inform about each touch with the filament which
he will percept
b) After each touch with the filament doctor should ask the patient if he feel the touch
c) Skin sensibility should be examined in 10 points on the dorsal surface of the foot
d) Pressure with filament to the skin should be maintained during the 10-15 minutes
e) Skin sensibility should be examined in 10 points on the plantar surface of the foot and toes

1076. CM. Neuropathic form of diabetic foot is characterized by:


a) The presence of painless plantar ulcers with hyperkeratosis
b) The presence of very painful necrosis of the toes
c) The absence of foot pulses
d) Warm and dry foot skin
e) Evident foot deformation

1077. CM. Ischemic form of diabetic foot is characterized by:


a) The presence of painless plantar ulcers with hyperkeratosis
b) The presence of very painful necrosis of the toes
c) The absence of foot pulses
d) Warm and dry foot skin
e) Systolic bruit at the femoral artery

1078. CM. For confirmation of circulatory disturb in ischemic form of diabetic foot the following methods are used:
a) Determination of ankle-brachial index
b) Semmes-Weinstein test
c) Transcutaneous determination of O2 pressure on the foot (ТсpO2)
d) Determination of toe systolic pressure by photoplethysmography
e) Foot X-ray

1079. CM. Specify the methods of vascular imaging used in diagnostic work-up of patients with ischemic form of
diabetic foot:
a) Photoplethysmography
b) Duplex ultrasound
c) CT-angiography
d) Isotope scintigraphy
e) MRI-angiography

1080. CS. The method of choice for diagnostics of osteomyelitis in diabetic foot syndrome is:
a) Foot X-ray
b) Bacteriologic study and culture of ulcer drainage
c) Magnetic resonance imaging (MRI)
d) Ulcer or wound probing
e) Photoplethysmography

1081. CM. What effects have the elevated glucose blood level upon immune system of patient?
a) Stimulation of leukocytes migration
b) Suppression of chemotaxis
c) Suppression of compliment system
d) Stimulation of monocytes function
e) Suppression of phagocytosis

1082. CS. What index reflects most exactly the grade of metabolic compensation in type II diabetes mellitus?
a) Blood glucose level
b) Hemoglobin
c) Glycolysated hemoglobin
d) Urine ketone bodies
e) Hematocrit

1083. CS. Choose the target level of HbA1C in patients with diabetic foot syndrome.
a) 20-30%
b) < 15%
c) 50-75%
d) < 7%
e) > 13%

1084. CS. If a probe reaches the bone during probing of plantar diabetic ulcer this suggests a high probability of:
a) Open foot fracture
b) Soft tissue infection
c) Ischemic nature of ulcer
d) Metatarsal dislocation
e) Foot osteomyelitis

1085. CM. Superficial infection of diabetic foot involves:


a) Skin
b) Adipose tissue
c) Muscles
d) Tendons
e) Bones

1086. CM. Deep infection of diabetic foot involves:


a) Blood vessels
b) Nerves
c) Muscles
d) Tendons
e) Bones

1087. CS. Diabetic foot infection associated with SIRS is classified according to the international classification as:
a) Grade 0 infection
b) Grade 1 infection
c) Grade 2 infection
d) Grade 3 infection
e) Grade 4 infection

1088. CM. Specify the clinical symptoms characteristic for grade 4 diabetic foot infection according to the
international classification.
a) Heart rate > 90 per min
b) Heart rate < 60 per min
c) Body temperature > 38°С
d) Body temperature < 36°С
e) Respiratory rate < 20 per min

1089. CM. Specify the abnormal changes in the laboratory tests, characteristic for diabetic foot infection.
a) Leukocytosis
b) Monocytosis
c) Hyperglycemia
d) Hypoglycemia
e) Eosinophilia

1090. CS. Superficial full-thickness diabetic ulcer (not extending through the subcutis) is classified according to
Wagner classification as:
a) Grade 1
b) Grade 2
c) Grade 3
d) Grade 4
e) Grade 5

1091. CS. Ulcer with exposed tendon or bone without osteomyelitis or abscess is classified according to Wagner
classification as:
a) Grade 1
b) Grade 2
c) Grade 3
d) Grade 4
e) Grade 5

1092. CS. Deep ulcer with osteomyelitis or abscess formation is classified according to Wagner classification as:
a) Grade 1
b) Grade 2
c) Grade 3
d) Grade 4
e) Grade 5

1093. CS. Localized gangrene of toes or the forefoot is classified according to Wagner classification as:
a) Grade 1
b) Grade 2
c) Grade 3
d) Grade 4
e) Grade 5

1094. CS. Foot with extensive gangrene is classified according to Wagner classification as:
a) Grade 1
b) Grade 2
c) Grade 3
d) Grade 4
e) Grade 5

1095. CM. Grade 0 diabetic foot according to Wagner classification means:


a) Superficial full-thickness ulcer (not extending through the subcutis)
b) Foot with extensive gangrene
c) Deep ulcer with osteomyelitis / abscess formation
d) Completely epithelialized ulcer
e) Absence of tissue defect

1096. CS. Grade 1 diabetic foot according to Wagner classification means:


a) Completely epithelialized ulcer
b) Deep ulcer with osteomyelitis / abscess formation
c) Superficial full-thickness ulcer (not extending through the subcutis)
d) Foot with extensive gangrene
e) Localized gangrene of the toes or forefoot

1097. CS. Grade 2 diabetic foot according to Wagner classification means:


a) Localized gangrene of the toes or forefoot
b) Foot with extensive gangrene
c) Deep ulcer with osteomyelitis / abscess formation
d) Completely epithelialized ulcer
e) Ulcer with exposed tendon or bone without osteomyelitis / abscess

1098. CS. Grade 3 diabetic foot according to Wagner classification means:


a) Deep ulcer with osteomyelitis / abscess formation
b) Superficial full-thickness ulcer (not extending through the subcutis)
c) Ulcer with exposed tendon or bone without osteomyelitis / abscess
d) Completely epithelialized ulcer
e) Localized gangrene of the toes or forefoot

1099. CM. Grade 4 diabetic foot according to Wagner classification means:


a) Gangrene of all digits
b) Completely epithelialized ulcer
c) Deep ulcer with osteomyelitis / abscess formation
d) Ulcer with exposed tendon or bone without osteomyelitis / abscess
e) Gangrene of the first toe

1100. CS. Grade 5 diabetic foot according to Wagner classification means:


a) Superficial full-thickness ulcer (not extending through the subcutis)
b) Foot with extensive gangrene
c) Deep ulcer with osteomyelitis / abscess formation
d) Completely epithelialized ulcer
e) Localized gangrene of the toes or forefoot

1101. CM. Choose the classifications of diabetic foot.


a) Wagner classification
b) Glasgow classification
c) University of Texas classification
d) SIRS classification
e) S(AD)SAD classification

1102. CM. Choose the drugs which are used for the treatment and prevention of vascular complications in patients
with diabetic foot syndrome:
a) Antispastic (papaverin, no-spa)
b) Antithrombotic (aspirin, clopidogrel, ticlid)
c) Group B vitamins
d) E1 prostanoids
e) Statine (Simvastatin, Atorvastatin)

1103. CM. The complex treatment of ischemic form of the diabetic foot Wagner grade IV should include:
a) Skin grafting of the foot ulcer
b) Surgical revascularization of affected limb
c) Necrectomy
d) Primary leg amputation at the level above the ankles
e) Remove of plantar callus

1104. CM. The complex treatment of neuropathic form of the diabetic foot Wagner grade III should include:
a) Surgical debridement of purulent focus
b) Surgical revascularization of affected limb
c) Extended antibacterial treatment
d) Primary above knee amputation
e) Off-load of affected foot

1105. CM. The most important factors which influence selection of correct curative strategy in patients with diabetic
foot are:
a) Presence and severity of ischemia
b) The possibility for surgical revascularization of affected limb
c) Patient age
d) Grade of tissue destruction
e) Presence of sensorial neuropathy

1106. CM. Specify the methods effective for treatment of ischemia in diabetic foot syndrome.
a) Fasciotomy
b) Surgical reconstruction of arteries
c) Endovascular percutaneous transluminal angioplasty with stent placement
d) Infusion of antispasmodic drugs, xanthine derivatives, vitamins B
e) Administration of heparin

1107. CS. The duration of antibacterial treatment in case of diabetic foot with osteomyelitis usually is:
a) 1-2 days
b) 5-7 days
c) 1-2 weeks
d) 2-4 weeks
e) 3-6 months

1108. CM. Choose the correct statements regarding VAC-therapy (Vacuum Assisted Closure) in diabetic foot
syndrome.
a) Method is used for the treatment of ulcers and wounds
b) Method is based on prolonged exposure of the wound to negative pressure
c) Method includes permanent washing of wound with antiseptics
d) Method is used for the treatment of neuropathy
e) Method requires introduction of polyurethane foam into the wound

1109. CS. Central venous access is performed preferably by:


a) Indirect venepuncture
b) Seldinger technique
c) Trendelenburg technique
d) Venesection
e) Guided venesection

1110. CM. Which of the following can be indications for central veins catheterization?
a) Inability to obtain a peripheral venous access
b) Hemodialysis
c) Severe coagulopathy in a patient with stable hemodynamics
d) Plasmaferesis
e) Lack of staff experience in a peripheral vein catheterization

1111. CM. Central venous access can be achieved through:


a) Internal jugular vein
b) Internal thoracic vein
c) Subclavian vein
d) Femoral vein
e) Common iliac vein

1112. CM. Placing the patient in the Trendelenburg position during jugular vein catheterisation:
a) Decrease svenous distension
b) Increases venous distension
c) Decreases the risk of thrombotic complications
d) Decreases the risk of air embolism
e) Decreases the risk of pneumothorax

1113. CS. Which kind of catheter is used to measure pressure in the pulmonary artery?
a) Seldinger catheter
b) Swan-Ganz catheter
c) Foley catheter
d) Biluminal catheter
e) Fogarty catheter

1114. CM. Which of the following condition can be an indications for installation of chest tube (thoracostomy)?
a) Pneumothorax
b) Haemotorax
c) Pulmonary edema
d) Pulmonary emphysema
e) Pleural empyema

1115. CM. Which of the given below refers to the complications of thoracentesis?
a) Pneumothorax
b) Haemopneumothorax
c) Acute respiratory distress syndrome
d) Emphysema
e) Pleural empyema
1116. CM. In case of pneumothorax thoracentesis is performed more frequently:
a) On the midclavicular line
b) On the midaxillary line
c) In the fifth intercostal space
d) In the second intercostal space
e) In the seventh intercostal space

1117. CM. In case of hydrothorax thoracentesis is frequently performed:


a) On the midaxillary line
b) On the midclavicular line
c) In the second intercostal space
d) In the XI intercostal space
e) In the seventh intercostal space

1118. CM. Choose the aims of installation of nasogastric tube:


a) Assessment of upper gastrointestinal bleeding (presence, volume)
b) Evacuation of gastric contents in the recent case of food intoxication
c) Decrease of gastric secretion
d) Increase of gastric secretion
e) Gastric decompression in cases of intestinal obstruction

1119. CM. Which kind of tubes is used for esophagogastric hemostatic balloon tamponade in variceal upper
digestive bleeding?
a) Sengstaken-Blakemore tube
b) Fogarty tube
c) Minnesota tube
d) Linton-Nachlas tube
e) Button tube

1120. CM. Among the complications of esophagogastric balloon tamponade in variceal upper digestive bleeding
may occur:
a) Bronchoalveolar aspiration
b) Perforation of the esophagus
c) Esophageal stricture
d) Reflux esophagitis
e) Esophageal diverticulum

1121. CM. In case of variceal upper digestive bleeding esophagogastric balloon tamponade is indicated in:
a) Bleeding from esophageal or gastric varices when conservative and endoscopic therapy is not effective
b) Bleeding from esophageal and gastric varices when endoscopic hemostasis is not possible
c) Bleeding from esophageal or gastric varices stopped endoscopically
d) Bleeding from esophageal or gastric varices stopped conservatively
e) Positive anamnesis of hemorrhage from esophageal or gastric varices in the last 2 weeks

1122. CS. In cases of upper digestive hemorrhage from varices with gastric localization haemostatic balloon
tamponade is posible with:
a) Nasogastric tube
b) Orogastric tube
c) Linton-Nachlas tube
d) Sengstaken-Blakemore tube
e) Nasogastroduodenal tube

1123. CS. In case of upper digestive bleeding from esophageal varices prolonged haemostatic balloon tamponade
can cause:
a) Reflux esophagitis
b) Mallory-Weiss syndrome
c) Rupture of esophageal balloon of tube
d) Esophageal mucosal necrosis and ulceration
e) Recurrence of bleeding
1124. CS. To prevent esophageal mucosa necrosis following prolonged haemostatic balloon tamponade with
Sengstaken-Blakemore tube it is recommended to:
a) Deflate gastric balloon over 24-48 hours after tube insertion
b) Deflate esophageal balloon over 24-48 hours after tube insertion
c) Inflate esophageal balloon over 24-48 hours after tube insertion
d) Withdraw temporary the tube over 24-48 hours after insertion
e) Change the tube every 24-48 hours

1125. CM. Pathological conditions that can be diagnosed by anoscopy are:


a) Hemorrhoidal disease
b) Pilonidal disease
c) Anal fissure
d) Anal itching
e) Perirectal fistula

1126. CS. Endoscopic examination with rigid sigmoidoscope allows the visualization of:
a) The whole sigmoid colon
b) The 50 cm of distal sigmoid colon
c) The 30 cm of distal sigmoid colon
d) The 50 cm of distal intestine
e) The 30-35 cm of distal intestine

1127. CS. Which of the following catheters is used for urethral catheterization?
a) Fogarty catheter
b) Foley catheter
c) Swan-Ganz catheter
d) Cantor catheter
e) Seldinger catheter

1128. CM. The complications of urethral catheterisation are:


a) Urethral trauma
b) Acute urinary retention
c) Urinary tract infection
d) Phimosis
e) Oliguria

1129. CM. Laparocentesis is indicated in the following conditions:


a) Suspected traumatic haemoperitoneum
b) Tension ascites
c) Suspected acute appendicitis
d) Acute abdominal intermittent pain
e) Acute abdominal continuing pain

Vous aimerez peut-être aussi